Càlcul infinitesimal per al batxilleraten el que las modernas tecnologías actuales permiten, por...

277
CÀLCUL INFINITESIMAL 2n Batxillerat Amb tots els problemes PAU 1998-2019 Gerard Romo Garrido

Transcript of Càlcul infinitesimal per al batxilleraten el que las modernas tecnologías actuales permiten, por...

CÀLCUL INFINITESIMAL

2n Batxillerat

Amb tots els problemes PAU 1998-2019

Gerard Romo Garrido

Toomates Cool·lección Los documentos de Toomates son materiales digitales y gratuitos. Son digitales porque están pensados para ser consultados

mediante un ordenador, tablet o móvil. Son gratuitos porque se ofrecen a la comunidad educativa sin coste alguno. Los libros de texto pueden ser digitales o en papel, gratuitos o en venta, y ninguna de estas opciones es necesariamente mejor o peor que las otras.

Es más: Suele suceder que los mejores docentes son los que piden a sus alumnos la compra de un libro de texto en papel, esto es un

hecho. Lo que no es aceptable, por inmoral y mezquino, es el modelo de las llamadas "licencias digitales" con las que las editoriales

pretenden cobrar a los estudiantes, una y otra vez, por acceder a los mismos contenidos (unos contenidos que, además, son de una

bajísima calidad). Este modelo de negocio es miserable, pues impide el compartir un mismo libro, incluso entre dos hermanos, pretende convertir a los estudiantes en un mercado cautivo, exige a los estudiantes y a las escuelas costosísimas líneas de Internet,

pretende pervertir el conocimiento, que es algo social, público, convirtiéndolo en un producto de propiedad privada, accesible solo a

aquellos que se lo puedan permitir, y solo de una manera encapsulada, fragmentada, impidiendo el derecho del alumno de poseer todo el libro, de acceder a todo el libro, de moverse libremente por todo el libro.

Nadie puede pretender ser neutral ante esto: Mirar para otro lado y aceptar el modelo de licencias digitales es admitir un mundo más

injusto, es participar en la denegación del acceso al conocimiento a aquellos que no disponen de medios económicos, en un mundo en el que las modernas tecnologías actuales permiten, por primera vez en la historia de la Humanidad, poder compartir el

conocimiento sin coste alguno, con algo tan simple como es un archivo "pdf".

El conocimiento no es una mercancía.

El proyecto Toomates tiene como objetivo la promoción y difusión entre el profesorado y el colectivo de estudiantes de unos

materiales didácticos libres, gratuitos y de calidad, que fuerce a las editoriales a competir ofreciendo alternativas de pago atractivas aumentando la calidad de unos libros de texto que actualmente son muy mediocres, y no mediante retorcidas técnicas comerciales.

Este documento se comparte bajo una licencia “Creative Commons”: Se permite, se promueve y se fomenta cualquier uso, reproducción y edición de todos estos materiales siempre que sea sin ánimo de lucro y se cite su procedencia. Todos los documentos

se ofrecen en dos versiones: En formato “pdf” para una cómoda lectura y en el formato “doc” de MSWord para permitir y facilitar

su edición y generar versiones parcial o totalmente modificadas. Se agradecerá cualquier observación, comentario o colaboración a

[email protected]

Actualmente, Toomates Cool·lección consta de los siguientes libros:

GA Geometría Axiomática pdf 1 2 ... 23 portada

PG Problemas de Geometría pdf 1 2 3 4 5 6 7

AG Àlgebra (Llibre de text) CAT pdf 1 2 3 4

PA Álgebra (Problem-solving) (en preparación)

pdf doc

AR Teoría de números (Problem-solving) pdf doc

GN Geometria analítica (Llibre de text) CAT pdf doc

TR Trigonometria (Llibre de text) CAT pdf doc

PT Trigonometría (Problem-solving) pdf doc

CO Nombres complexos (Llibre de text) CAT pdf doc

PC Números complejos (Problem-solving) pdf doc

DE Desigualdades pdf doc

AL Àlgebra Lineal 2n batxillerat CAT pdf doc

GL Geometria Lineal 2n batxillerat CAT pdf doc

CI Càlcul Infinitesimal 2n batxillerat CAT pdf 1 2

PL Programació Lineal 2n batxillerat CAT pdf doc

PT Compendium PAU TEC 1998-2019 CAT pdf

PS Compendium PAU CCSS 1998-2019 CAT

pdf

PM Problemas de Matemáticas pdf doc

Versión de este documento: 18/01/2020

www.toomates.net

Índex

1 Límits i indeterminacions. →

1.1 Elements notables d’una funció.

1.2 Concepte de límit.

1.3 Indeterminació / amb funcions racionals.

1.4 Indeterminació / amb ordre de creixement.

1.5 Indeterminació 0/0 amb funcions racionals..

1.6 Indeterminació 0/0 amb arrels quadrades.

1.7 Indeterminació 0/k amb funcions racionals.

1.8 Indeterminació 0∞.

2 Funcions racionals sense derivació. →

2.1 Taula de l’estudi d’una funció racional.

2.2 Exercicis resolts d’estudi de funcions racionals.

2.3 Problemes PAU CCSS amb funcions racionals sense derivació.

3 Continuïtat. →

3.1 Definició.

3.2 Continuïtat de funcions definides a trossos.

3.3 Problemes PAU CCSS amb continuïtat de funcions definides a trossos.

3.4 Continuïtat de funcions definides a trossos amb paràmetres.

3.5 Teorema de Bolzano i aplicacions. Teorema de Weirestrass.

4 Derivada d’una funció en un punt. →

4.1 Taxa de variació mitjana.

4.2 Concepte de derivada d’una funció en un punt.

4.3 Creixement, decreixement i extrems relatius.

4.4 Propietats de les funcions derivables.

4.5 Derivabilitat amb funcions definides a trossos.

4.6 Curvatura.

5 Funcions polinòmiques amb derivació. →

5.1 Primera derivada de funcions polinòmiques.

5.2 Estudi de funcions polinòmiques mitjançant derivació.

5.3 Problemes PAU TEC amb funcions polinòmiques.

5.4 Problemes PAU CCSS amb funcions polinòmiques.

6 Funcions racionals amb derivació. → 6.1 Derivada del producte, potència i quocient de funcions.

6.2 Estudi de funcions racionals (sense segona derivada).

6.3 Estudi de funcions racionals (amb segona derivada).

6.4 Problemes PAU TEC amb funcions racionals i derivació.

6.5 Problemes PAU CCSS amb funcions racionals i derivació.

7 Tècniques de derivació. →

7.1 Taula de derivades de funcions elementals.

7.2 Àlgebra de derivades.

7.3 Regla de la cadena.

7.4 Pràctica amb derivades.

7.5 Derivació de polinomis i funcions racionals amb la Regla de la Cadena.

7.6 Derivació de funcions radicals.

8 Recta tangent.→

8.1 Concepte de recta tangent.

8.2 Determinació de la recta tangent.

8.3 Determinació del punt de tangència.

8.4 Recta normal a la gràfica d’una funció.

9 Funcions exponencials i logarítmiques. →

9.1 Extrems relatius de funcions exponencials i logaritmes.

9.2 Problemes PAU TEC amb funcions exponencials i logarítmiques.

9.3 Problemes PAU CCSS amb funcions exponencials i logarítmiques.

10 Funcions trigonomètriques. → 10.1 Pràctica derivant funcions trigonomètriques.

10.2 Recta tangent amb funcions trigonomètriques.

11 Problemes d’optimització. →

11.1 Resolució de problemes d’optimització.

11.2 Problemes PAU TEC d’optimització.

11.3 Problemes PAU CCSS d’optimització.

12 Regles de L’Hôpital. →

12.1 Regla de L’Hôpital fonamental.

12.2 Altres indeterminacions que es poden resoldre amb L’Hôpital.

13 Integral indefinida, definida i àrea. →

13.1 Integral indefinida.

13.2 Integral definida.

13.3 Teorema fonamental del càlcul. Regla de Barrow.

13.4 Determinació d’àrees amb funcions polinòmiques.

14 Tècniques d’integració. →

14.1 Integrals immediates.

14.2 Integració per substitució.

14.3 Integració per parts.

14.4 Integració de funcions racionals.

14.5 Substitucions trigonomètriques.

14.6 Determinació d’àrees amb funcions en general.

15 Llistes d’exercicis de repàs general. →

Les 21 funcions fonamentals. →

Solucions. →

Links d’interès. →

1 Límits i indeterminacions.

1.1 Elements notables d’una funció.

1.1.1 Definició. Funció.

Una funció entre dos conjunts és una relació que assigna a cada element del primer un

sol element del segon. Aquest últim element rep el nom d’imatge de l’element del

primer. S’acostuma a designar la funció amb una lletra, per exemple f , i si un element

del primer conjunt és x, la seva imatge es designa per )(xf .

El primer conjunt s’anomena domini de la funció. Quan una funció entre nombres ve

donada per una regla i no s’explicita quin és el domini, s’entén que és el conjunt més

gran en el qual la regla es pot aplicar.

El subconjunt del segon conjunt format per totes les imatges del domini rep el nom

d’imatge o recorregut de la funció.

La gràfica de la funció serà el conjunt de punts del pla del tipus )(, xfx

1.1.2 Domini de definició.

Les funcions IRIRf : no estan definides necessàriament per a tots els valors de x.

Direm que f no està definida per a px quan no existeix )( pf . En cas contrari

direm que la funció està definida per a px . El domini de definició d’una funció és el

conjunt de punts )( fDom per als quals la funció està definida.

1.1.3 Proposició. Punts de no definició de les funcions fonamentals.

Trobarem punts de no definició, és a dir, punts fora del domini, als següents casos:

a) Funcions racionals: On s’anul·la el denominador:

)(

)()(

xg

xhxf i 0)( xg

Recorda: No està definida encara que també s’anul·li el numerador. No existeix 0/0.

b) Arrel quadrada: Per a valors d’x negatius.

xxf )( i 0x

i en general qualsevol arrel d’índex parell: n xxf )( per a 0x

c) Logarítmiques: Per a valors d’x negatius o zero.

)(log)( xxf a i 0x

Recorda: Existeix 00 però no existeix )0log(

d) Tangent: Per a 2

x .

)tan()( xxf i ,...2

5,

2

3,

2

x

1.1.3 Punt de tall amb l’eix Y.

És el possible punt de tall de la gràfica de la funció amb l’eix d’ordenades (el vertical).

Per trobar-lo hem d’avaluar la funció: )0(f .

1.1.4 Punts de tall amb l’eix X.

Són els possibles punts de tall de la gràfica de la funció amb l’eix d’abscisses

(l’horitzontal)

Per trobar-los hem de resoldre l’equació 0)( xf .

1.1.5 Simetria respecte de l’eix d’ordenades.

Això vol dir que per a qualsevol x del domini de la funció )(xf la imatge de x i del seu

oposat x ha de ser la mateixa:

)()( xfxf

Aquestes funcions es diuen funcions parelles.

1.1.6 Simetria respecte de l’origen de coordenades.

Això vol dir que per a qualsevol x del domini de la funció )(xf la imatge de x i del seu

oposat –x han de ser oposades:

)()( xfxf

Aquestes funcions es diuen funcions senars.

1.2 Concepte de límit.

1.2.1 Definició tècnica. Límit d’una funció en un punt.

Límit finit en un punt.

Sigui :)(xf . Direm que el límit d’una funció quan x s’apropa a a és b si

bxfxabxfax

)(0|0,0)(lim

Intuïtivament, f(x) s’acosta a b en la mesura que x s’acosta a a.

En altres mots, es pot fer f(x) tant prop de b com es vulgui sobre un interval prou petit al

voltant de a.

Límits infinits en un punt.

MxfaxMxfax

)(0|0,0)(lim

MxfaxMxfax

)(0|0,0)(lim

En altres mots, es pot fer f(x) tant a proa de com es vulgui, sobre un interval proa

petit al voltant de a.

Límit d’una funció a l’infinit.

bxfMxMbxfx

)(|0,0)(lim

És a dir, es pot fer f(x) tant a prop de l com es vulgui, a partir d’un valor M prou gran.

Límit infinit d’una funció a l’infinit.

NxfMxMNxfx

)(|0,0)(lim

)(lim xfx

si per a valors molt grans de x, els termes corresponents de f(x) es fan

més grans que qualsevol nombre prefixat.

NxfMxMNxfx

)(|0,0)(lim

)(lim xfx

si per a valors molts grans de x, els termes corresponents de f(x) es fan

més grans en negatiu que qualsevol nombre prefixat.

De la mateixa manera es defineixen

)(lim xfx

i

)(lim xfx

1.2.2 Àlgebra de límits.

Si 1)(lim lxfax

i 2)(lim lxgax

llavors

a) 21)()(lim llxgxfax

b) 21)()(lim llxgxfax

c) 0)(

)(lim 2

2

1

lsil

l

xg

xf

ax

d) 1)(lim lkxfkax

e) realnombreunéslsilxfllxg

ax

22

11

)()(lim

1.2.3 Les indeterminacions.

Al 99.9% dels casos: )()(lim afxfax

, és a dir, per determinar un límit només cal

avaluar la funció en aquest punt.

Però al 0.1% dels casos.... no podem fer-lo d’aquesta manera, són les anomenades

indeterminacions:

1) 2) 0 3) 0

0 4)

5) 1 6) 0 7) 00

1.2.4 Observació. Truco per calcular aproximadament un límit.

En general, el valor de )(lim xfax

, en ax serà aproximadament el valor de la funció

per a un valor 'ax molt pròxim, per exemple, a una centèsima:

)01.0()(lim

afxfax

Exemple:

Ens demanen calcular x

x

x

sinlim

0.

Ens trobem davant d’una indeterminació 0/0 i suposem que no coneixem cap tècnica per

resoldre-la (Hopital). Podem prendre un punt molt pròxim a 0 i avaluar:

101.0

)01.0sin(sinlim

0

x

x

x (Recorda que has de tenir la calculadora en Radians)

1.3 Indeterminació / amb funcions racionals.

1.3.1 Esquema principal.

Donada una funció racional )(

)()(

xq

xpxf , amb p(x) i q(x) polinomis,

1. Si )()( pgradqgrad , llavors 0)(lim

xfx

Exemple: 733

4452)(

25

23

xx

xxxxf , 0)(lim

xf

x

2. Si )()( pgradqgrad , llavors kxfx

)(lim ,

on k és el quocient entre el coeficient del monomi de grau màxim del numerador

i el coeficient del monomi de grau màxim del denominador.

Exemple: 33

124)(

2

2

x

xxxf ,

3

4)(lim

xf

x

Tant en el cas 1 com en el 2, parlarem d’asímptotes horitzontals.

3. Si )()( qgradpgrad , llavors

)(lim xfx

En particular, si 1)()( qgradpgrad , tindrem una asímptota obliqua:

quan x creix, els valors de la funció tendeixen als valors d’una recta baxy ,

1.3.2 Determinació d’una asímptota obliqua.

Recordem que si )(

)()(

xq

xpxf , amb p(x) i q(x) polinomis, i 1)()( qgradpgrad ,

llavors

)(lim xfx

en la forma baxy .

Determinació mitjançant límits:

x

xfa

x

)(lim

, i axxfb

x

)(lim

Determina l'asímptota obliqua de la funció: 2

12)(

2

x

xxf

22

12lim

)(lim

2

2

xx

x

x

xfa

xx

42

14lim2

2

12lim)(lim

2

x

xx

x

xxaxfb

xxx

Per tant, la funció f(x) té com a asímptota obliqua la recta 42 xy

Determinar l’asímptota obliqua de la funció 32

1)(

2

x

xxf

5.02

1

32

1lim

)32(

1lim

)(lim

2

22

xx

x

xx

x

x

xfa

xxx

75.04

3

2

23

32

12

3

lim2

1

32

1lim)(lim

*2

x

x

xx

xaxxfb

xxx

32

12

3

32

2

3

2

21

32

322

11

32

322

1

32

1

2

1

32

1*

22222

x

x

x

xxx

x

xxx

x

xx

x

xx

x

x

Per tant, l’asímptota obliqua és 75.05.04

3

2

1 xxy

Determinació mitjançant una divisió de polinomis.

Una forma alternativa de trobar-la és fent la divisió del numerador entre el

denominador.

Determineu l’asímptota obliqua de la funció 3

22)(

2

x

xxxf

x2 -2x +2 x -3

x2 -3x x +1

/ x +2

x -3

/ 5

Per tant , l’asímptota obliqua de la funció és 1 xy

Determineu l’asímptota obliqua de la funció 1

573)(

2

x

xxxf

3x2 +7x -5 x + 1

3x2 +3x 3x + 4

4x -5

4x 4

-9

La funció té l’asímptota obliqua y = 3x + 4

1.4 Indeterminació / amb ordre de creixement.

Direm que l’infinit )(xf quan ax és d’ordre superior a l’infinit )(xg quan

ax si

)(

)(lim

xg

xf

ax

i escriurem )()( xgxf quan ax

Direm que l’infinit )(xf quan ax és d’ordre inferior a l’infinit )(xg quan ax

si

0)(

)(lim xg

xf

ax

i escriurem )()( xgxf quan ax

Direm que l’infinit )(xf quan ax i l’infinit )(xg quan ax són del mateix

ordre si

0)(

)(lim

kxg

xf

ax

i escriurem )()( xgxf quan ax

Infinits d’ordre k.

Direm que l’infinit )(xf quan ax és d’ordre k si és equivalent al polinomi kxxg )(

És molt fàcil comprovar que qualsevol polinomi de grau k és d’ordre k.

Exemple: 53)( xxf i xxg )( són infinits del mateix ordre quan x perquè

35

3lim)/53(

lim53

lim)(

)(lim

xx

xx

x

x

xg

xf

xxxx

Comparació d’infinits exponencial, potencial, radical i logarítmic.

Quan x , Tota funció exponencial xa amb base 1a és d’ordre superior a

qualsevol funció potencial x n amb 0n , i tota funció potencial és d’ordre superior a

tota funció logarítmica xblog amb 1b . Aquesta comparativa també inclou les

funcions radicals n xxf )( , només cal escriure el radical en forma d’exponent

fraccionari: nn xxxf /1)( :

xxxa bmnx log

1.5 Indeterminació 0/0 amb funcions racionals.

Una funció racional és una funció donada com a quocient de dos polinomis:

)(

)()(

xQ

xPxf , on P(x) i Q(x) són polinomis.

Si per a ax tenim que 0)(0)( aQiaP ens trobem en un cas d’indeterminació del

tipus 0

0.

Per a resoldre aquesta indeterminació, factoritzem numerador i denominador,

simplifiquem i calculem el límit de la expressió simplificada:

axsiaxxQ

axxP

xQ

xPxf

)/()(

)/()(

)(

)()(

El resultat pot ser una discontinuïtat evitable:

kxfxfxx

)(lim)(lim

o una asímptota vertical (vegeu apartat 1.7)

)(lim xfax

Exercicis:

a) xx

xxx

x 4

4lim

2

23

0

b)

xx

x

x 3

9lim

2

2

3

1.6 Indeterminació 0/0 amb arrels quadrades.

Per resoldre una indeterminació 0

0 amb arrels quadrades hem de multiplicar pel

conjugat.

Calcula: xx

x

x 2

35lim

2

2

2

Observem que ens trobem davant d'una indeterminació 0/0:

0

0

222

352

2

35lim

2

2

2

2

2

xx

x

x

Multipliquem i dividim pel conjugat:

3

1

12

4

35

2

352

22

352

4

352

35

352

35

352

3535

2

35

22

222

2

22

22

22

22

2

22

22

2

2

xxx

x

xxx

xx

xxx

x

xxx

x

xxx

x

xxx

xx

xx

x

Per tant 3

1

2

35lim

2

2

2

xx

x

x

Exercicis.

1.

a) 1

1lim

2

1

x

x

x b)

xx

xx

x

20

22lim c)

22

2lim

2

x

x

x

d) x

x

x

82

53lim

2 e)

22

11lim

2

x

x

x

2.

a) 2

2lim

2

x

x

x b)

3

3lim

3

x

x

x c)

x

x

x

22lim

0

d)

xx

x

x 2lim

0

e) 5

411lim

5

x

x

x f)

x

x

x

2

4lim

2

2 g)

x

x

x

2

16lim

2

4

1.7 Indeterminació 0/k amb funcions racionals.

En general, si a una funció racional )(

)()(

xQ

xPxf , i ax es compleix que

0)( kaP i 0)( aQ , llavors

)(lim xfax

.

Cal diferenciar els quatre possibles casos:

Per determinar a quin cas correspon, avaluarem la funció en dos punts propers a ax ,

un per la dreta i un altre per l’esquerra.

A)

)(lim xfax

)(lim xfax

B)

)(lim xfax

)(lim xfax

C)

)(lim xfax

)(lim xfax

D)

)(lim xfax

)(lim xfax

1.8 Indeterminació 0∞.

1.8.1 Mètode de la fórmula.

Si 1)(lim

xfx

i

)(lim xgx

, es verifica que

)()1)((lim)()(lim

xgxfxg

x

xexf

Calcula:

13

2

2

5

3lim

x

x x

x

15

3)(

2

2

x

x

xxf

x

xxg 13)(

Tenim, doncs, una indeterminació 1∞. Apliquem la fórmula:

5

2

5

53

5

53

5

5

5

31

5

31)(

222

22

2

2

2

2

2

2

xxx

xx

x

x

x

x

x

xxf

05

2613

5

2131)(

22

x

x

xx

xxxf

15

3lim 0

13

2

2

e

x

xx

x

1.8.2 Mètode de les plantilles.

Es tracta d’adaptar les expressions a una de les dues plantilles següents:

(A) k

u

ue

u

k

1lim (B) kv

vevk

/1

01lim

Calcula 2

12

52

32lim

x

x

x x

x

Estem davant d’una indeterminació 1 , que resoldrem reescrivint l’expressió de la

forma A:

52

81

52

8

52

52

52

852

52

32

xxx

x

x

x

x

x

52

152

52

52

52

81

52

81

52

32

xxx

x

xxx

x

42/82/182

1

52

152

2

1

52

152

2

1

2

2

2

52

81

52

81

52

32

eeex

xx

x

xx

x

xx

x

x

xxx

x

ja que 2

1

102

1

2

1

52

12

22

x

xx

x

x

x

x

Calcula x

xx

sin/1

031lim

Estem davant d’una indeterminació 1 , que resoldrem reescrivint l’expressió de la

forma B:

xxxxxxx

/1/313131

313sin//1)sin/1(/1

sin/1/1sin/1

31313131 eexxxxxxxxxxx

xxxx

ja que 1sin

0

x

x

x

2 Funcions racionals sense derivació.

Una funció racional és una funció donada com a quocient de dos polinomis:

)(

)()(

xQ

xPxf , on P(x) i Q(x) són polinomis.

2.1 Taula de l’estudi d’una funció racional.

a) Domini de definició.

Una funció racional està definida sempre que no s’anul·li el denominador.

0)(|)( xQxfDom

b) Indeterminacions 0/0.

Si per a ax tenim que 0)(0)( aQiaP ens trobem en un cas

d’indeterminació del tipus 0

0. Vegeu apartat 1.5.

c) Punt de tall amb l’eix Y.

Avaluem la funció per a )0(0 fx

d) Punts de tall amb l’eix X.

Una funció racional )(

)()(

xQ

xPxf té un zero (punt de tall amb l’eix X) en

ax si 0)(0)( aQiaP , és a dir, s’anul·la el numerador però no el

denominador en a.

La funció es perfectament contínua en aquest punt.

e) Asímptotes verticals.

Una funció racional )(

)()(

xQ

xPxf té una asímptota vertical en ax si

0)(0)( aQiaP , és a dir, s’anul·la el denominador però no el numerador.

També poden aparèixer asímptotes verticals a algunes de les indeterminacions

0/0.

En particular, la funció no està definida en x = a, i per tant té una discontinuïtat

en aquest punt.

Vegeu apartat 1.5.

f) Comportament d’una funció racional en l’infinit.

Podem trobar infinits, asímptotes horitzontals i asímptotes obliqües. Vegeu

apartat 1.3.

Nota importat:

La gràfica de la funció mai pot creuar una asímptota vertical. Però pot creuar tantes

vegades com vulgui una asímptota obliqua o una asímptota horitzontal.

g) Gràfica.

2.4 Exercicis resolts d’estudi de funcions racionals.

a) Punts de tall amb l’eix X.

b) Punt de tall amb l’eix Y.

c) Asímptotes verticals.

d) Comportament de la funció en l’infinit.

e) Gràfica (esquemàtica) de la funció amb la informació dels apartats anteriors.

1. 54

3)(

2

xx

xxf

2. 54

86)(

2

2

xx

xxxf

3. 1

)3()(

2

x

xxf

4. 6

652)(

2

23

xx

xxxxf

5. 3 2

3 2

7 20 96( )

9 14 24

x x xf x

x x x

6. 2

3 2

11 28( )

6 30

x xf x

x x x

7. 3 2

2

3 25 21( )

3 10

x x xf x

x x

2.3 Problemes PAU CCSS amb funcions racionals sense derivació.

1. Sabem que la funció 1

)(

cx

baxxf passa pel punt (2, –5) i que les rectes x = 1 i

y = 2 en són les asímptotes vertical i horitzontal, respectivament. Calculeu a, b i c.

Solució PAU CAT CCSS SET 2018 3.6

2. Una empresa ven un producte a un preu de p euros. El nombre d’unitats venudes

depèn del preu que fixem segons la funció

p

ppV

1030)(

a) Demostreu que, en augmentar els preus, les vendes disminueixen.

b) És possible que l’empresa vengui 20 unitats del producte? Si el preu augmenta

indefinidament, què passarà amb les vendes?

Solució PAU CAT CCSS SET 2016 1.2

3. S’ha observat que el nombre d’entrades que es venen al cinema d’un poble està lligat

al sou mitjà x de la població, expressat en milers d’euros, segons la funció

1

50)(

2

x

xxN .

a) Determineu el sou mitjà de la població que correspon a la màxima venda d’entrades i

justifiqueu la resposta.

b) Si suposem que els sous de la població creixen indefinidament, com incidiria aquest

fet en la venda d’entrades del cinema?

Solució PAU CAT CCSS SET 2015 5.4

4. Donada la funció següent:

1)(

2

2

x

xxf

a) Determineu-ne el domini i els valors de x per als quals el signe de la funció f és

negatiu.

b) Determineu les asímptotes horitzontals i verticals de la funció f.

Solució PAU CAT CCSS SET 2010 2.6

5. Trobeu les equacions de les asímptotes verticals i obliqües de la funció

182

5)(

2

23

x

xxxg

PAU CAT CCSS SET 1998 2.4

3 Continuïtat.

3.1 Concepte de continuïtat d’una funció en un punt.

3.1.1 Definició. Continuïtat d’una funció (Interpretació visual).

Una funció té una discontinuïtat en un punt quan la seva gràfica està trencada. Si la

gràfica fos una mànega d'aigua, en aquest punt tindria una fuita. Les discontinuïtats

poden ser de tres tipus: evitables, de salt i asímptotes verticals.

Discontinuïtat evitable Discontinuïtat de salt

Si una funció no té discontinuïtats, direm que és contínua.

3.1.2 Definició. Continuïtat d’una funció (Definició formal).

Direm que una funció )(xfy és contínua en el punt “a” si existeixen i són iguals els

següents valors:

a) )(lim xfax

b) )(lim xfax

c) )(af

Els valors )(lim xfax

i )(lim xfax

s'anomenen límits laterals.

3.1.3 Alguns tipus de discontinuïtats notables.

1. Discontinuïtat de salt: )(lim)(lim xfxfaxax

Exemple:

32

3)(

xsi

xsixxf té una discontinuïtat de salt en x = 3.

2. Discontinuïtat evitable: )()(lim)(lim afxfxfaxax

Exemple:

53

52)(

xsi

xsixf té una discontinuïtat d’aquest tipus a x = 5

També les trobem en algunes indeterminacions 0/0.

Exemple: 2

2)(

2

x

xxxf té una discontinuïtat d’aquest tipus a x = 2

3. Asímptotes verticals:

)(lim xfax

o bé

)(lim xfax

Normalment trobem aquest tipus de discontinuïtat en funcions racionals, quan el

denominador val zero per a x=a i el numerador no.

Exemple: 3

1)(

xxf té una discontinuïtat d’aquest tipus a x = 3

3.1.4 Continuïtat de les funcions fonamentals.

1. La funció constant kxf )( és contínua en ),(

2. La funció potencial nxxf )( , amb n>0 és contínua en ),(

3. Les funcions irracionals n xxf )( són contínues en el seu domini.

,0)(n xDom si n és parell

),()( n xDom si n és senar

4. La funció exponencial xaxf )( , 0a , 1a és contínua en ),(

5. La funció logarítmica xxf alog)( , 0a , 1a són contínues al seu domini de

definició: ),0(

6. Les funcions trigonomètriques )sin()( xxf i )cos()( xxf són contínues en

),( .

La funció )tan()( xxf és contínua en tot el seu domini

kxDom2

),(,...2

5,

2

3,

2),())(tan(

7. Les funcions trigonomètriques inverses )arcsin()( xxf i )arccos()( xxf són

contínues en el seu domini [ -1 , 1 ].

3.1.5 Continuïtat amb les operacions amb funcions.

Si les funcions )(xf i )(xg són contínues en ax , llavors també són contínues en

ax les següents funcions:

1. La funció suma: )()( xgxf

2. La funció diferència: )()( xgxf

3. La funció producte: )()( xgxf

4. La funció quocient: )(

)(

xg

xf quan 0)( ag

5. Si )(xg és contínua en ax i )(xf és contínua en )(ag , llavors la funció composta

))(()( xgfxgf també és contínua en ax

Una conseqüència dels resultats anteriors és que qualsevol funció polinòmica és

contínua en tota la recta ),( .

3.2 Continuïtat de funcions definides a trossos.

3.2.1 Mètode per determinar la continuïtat de funcions definides a trossos.

Objectiu : Estudiar la continuïtat d’una funció definida a trossos al voltant del seu punt

frontera ax :

axsi)(

axsi)()(

xh

xgxf

i suposem que )(xg i )(xh són funcions contínues en ax .

Procediment:

Hem d’estudiar tres valors :

)(lim xfAax

)(lim xfBax

)(afC

i comprovar que CBA .

El valor C no ofereix dificultat. Només cal veure quina de les dues funcions )(xg

o )(xh determina la funció en ax .

Anem a estudiar A:

A l’esquerra del punt a, tenim que )()( xgxf , i per tant

)(lim)(lim xgxfAaxax

Si a més a més, la funció )(xg és contínua en ax , es complirà, per definició de

continuïtat, que:

)()(lim)(lim agxgxgaxax

Per tant

)()(lim)(lim agxgxfAaxax

.

El valor B s’obté de la mateixa manera:

Si )(xh és contínua en ax , tenim que

)()(lim)(lim ahxhxfBaxax

Així doncs, si )(xh i )(xg són dues funcions contínues en ax , la determinació dels

límits laterals es redueix a l’avaluació de les funcions al punt frontera:

f és contínua en ax )()( ahag

3.2.2 Exercicis i problemes de continuïtat de funcions definides a trossos.

1. La funció f(x) està definida de la següent manera:

15

121

23

)(

xsix

xsi

xsix

xf

a) Dibuixeu la gràfica de la funció.

b) Determina els punts de tall de la funció amb els eixos.

c) Estudieu la continuïtat de la funció.

2. La funció f(x) està definida de la següent manera:

023

022)(

2 xsixx

xsixxf

a) Dibuixeu la gràfica de la funció.

b) Determina els punts de tall de la funció amb els eixos.

c) Estudieu la continuïtat de la funció.

PAU TEC 2006 3.4

3. Representa gràficament i estudia la continuïtat i les següents funcions:

0

0)(

xsie

xxxf

x

4. Representa gràficament i estudia la continuïtat de la següent funció

122

11)(

2

xx

xxxf

5. Representa gràficament i estudia la continuïtat de la següent funció:

95

934

312

)(

xx

x

xx

xf

3.3 Problemes PAU CCSS amb continuïtat de funcions definides a trossos.

1. Considereu la funció

2

21

132

)(

2 xsix

xsibax

xsix

xf

Trobeu el valor de a i b perquè la funció sigui contínua per a tots els nombres reals.

Solució PAU CAT CCSS SET 2018 3.1

2. Considereu la funció següent :

01

02)(

2

xsie

xsibxxxf

x

a) Determineu el valor de b perquè la funció f sigui contínua en el punt x = 0.

Justifiqueu si f pot ser discontínua en algun altre punt.

a) b) Justifiqueu si, per a valors positius de x, la funció f és creixent o decreixent.

Solució PAU CAT CCSS JUNY 2010 1.2

3. Considereu la funció següent:

43

421

23

)(

xsi

xsix

xsi

xf

a) Feu-ne la representació gràfica.

b) Digueu en quins punts és discontínua i quin tipus de discontinuïtat té.

Solució PAU CAT CCSS JUNY 2009 3.1

4. Un hivernacle està destinat al cultiu de tomàquets. Se sap que les tomaqueres només

produeixen fruits si la temperatura dins l’hivernacle està entre 15 °C i 40 °C. En la

gràfica següent es mostra la producció de tomàquets en kilograms, segons la

temperatura que es manté a l’hivernacle.

a) Si la temperatura està entre 15 ºC i 29 ºC, digueu quina variació experimenta la

producció en augmentar la temperatura 1 °C. Calculeu aquesta variació quan la

temperatura està entre 30 °C i 39 °C.

b) Definiu una funció a trossos que expressi la producció segons la temperatura.

c) Trobeu les temperatures per a les quals s’obté el 75 % de la producció màxima.

Solució PAU CAT CCSS JUNY 2008 5.6

5. Considerem la funció definida a trossos següent:

xsibx

xsix

xsiax

xf

13

125

24

)( 2

a) Calculeu els valors de a i de b per tal que f(x) sigui contínua per a tot x.

b) Feu un gràfic de la funció obtinguda en l’apartat anterior.

Solució PAU CAT CCSS JUNY 2007 2.2

6. Considereu la funció :

023

022)(

2 xsixx

xsixxf

b) Dibuixeu la gràfica.

c) Estudieu-ne la continuïtat.

d) Determineu els extrems relatius. Solució PAU CAT CCSS JUNY 2006 3.4

7. Considereu la funció:

023

02)(

2

3

xsixx

xsixxxf

a) Estudieu-ne la continuïtat.

e) Determineu els intervals de creixement i decreixement de la funció.

f) Feu un gràfic aproximat de la funció.

g) Trobeu els extrems relatius i absoluts en l’interval [-2,2].

Solució PAU CAT CCSS SET 2006 4.5

8. Sigui

1)(

1)(

2

1

xsiax

xsiexf

x

Per a quins valors del paràmetre a la funció és contínua?

Solució PAU CAT CCSS JUNY 2004 3.3

9. Considereu la funció

0

3

1

04

)(

2

xx

xx

xf .

a) Feu-ne una representació gràfica aproximada. Justifiqueu per a quins valors de x la

funció és discontínua.

b) Calculeu l’equació de la recta tangent a la gràfica de f en el punt d’abscissa x=4.

Solució PAU CAT CCSS JUNY 2011 4.4

3.4 Continuïtat de funcions definides a trossos amb paràmetres.

La funció f(x) està definida de la següent manera:

23

21)(

2

xax

xaxxxf

Determina el valor del paràmetre a per al qual aquesta funció és contínua.

Per a valors 2x aquesta funció és contínua, perquè és polinòmica (de grau 2 a

l’esquerra de 2 i de grau 1 a la dreta del 2)

Per a 2x , hem d’aplicar la definició de continuïtat:

)2()(lim)(lim22

fxfxfxx

1241221lim)(lim 22

22

aaaxxxf

xx

aaxxfxx

)2(33lim)(lim22

124122)2( 2 aaf

Per tant, la condició de continuïtat en 2x equival a l’equació

aa 23124

aaaaaaaa 112614612423124

Donada la funció

21

2123)(

2

2

xxbxa

xxxxf

Determineu els valors d’a i b de forma que la funció sigui contínua i la seva gràfica

passi pel punt )1,3( .

)(xf contínua en 2x

9124

1241221)2(

912223123)(lim

1241221)(lim

2

2

2

2

2

2

2

2

2

2

2

ba

bababxaxf

xxxf

bababxaxxf

x

xx

xx

La gràfica passa pel punt )1,3( 113911331)3( 2 babaf

El sistema

1139

9124

ba

ba té una única solució que és 12,4 ba .

3.4.2 Exercicis.

1. Troba el valor de k perquè la funció següent sigui contínua:

a)

062

0)(

2 xsikxx

xkxxf

b)

1)ln(

1)(

xsikx

xxxf

c)

232

22)(

xsix

xkxxf

2. Determina els valors dels paràmetres a i b perquè la funció sigui contínua:

a)

xsibx

xsix

xax

xf

12

111

12

)( 2 b)

xsix

xsibax

xx

xf

14

12

21

)( 2

3. Determina els valors dels paràmetres a i b perquè la funció sigui contínua:

a)

1

102

04sin

)(

2 xabxx

xbabx

xx

xf x b)

11

10333

0cos

)(

2 xabxx

xbax

xax

xf x

4. Determina els valors a i b perquè la següent funció sigui contínua:

274

2132

113

)(

2

xx

xbax

xx

xf

5. Determina el valor de h perquè la funció següent sigui contínua:

352

313)(

2 xhxx

xxhxf

6. Determina el valor del paràmetre a perquè la funció següent sigui contínua:

12

11

12

)( 2

xx

x

xax

xx

x

xf

3.5 Teorema de Bolzano i aplicacions. Teorema de Weirestrass.

Teorema. Teorema de Bolzano.

Si una funció és contínua en un interval tancat ba, i 0)()( bfaf (vol dir que les

dues imatges tenen signe distint), existeix un punt c de l’interval obert ),( ba en el qual

la funció s’anul·la. Simbòlicament:

Si ba,en contínua f i suposant que

)(af i )(bf tenen signes diferents,

llavors existeix un ba,c tal que

0)( cf

La propietat és totalment intuïtiva: Si la funció és contínua i la seva gràfica ha d’unir els

punts )(, afa i )(, bfb , que són en costats diferents de l’eix d’abscisses, sense

aixecar el llapis del paper, a la força ha de tallar l’eix d’abscisses entre a i b.

Bernardo Bolzano (1781-1848), sacerdot catòlic que va fer

aportacions importants a les Matemàtiques en la primera meitat del

segle XIX. Va ser un dels primers en reconèixer que moltes de les

propietats sobre funcions contínues que semblaven òbvies exigien

una demostració. Les seves demostracions referents a la continuïtat

van ser publicades al 1850 en la seva obra pòstuma "Paradoxes de

l'infinit". Un dels seus resultats més coneguts porta el seu nom: "el

Teorema de Bolzano":

Observacions:

1. És indispensable la continuïtat en l’interval tancat. Si la funció és discontínua, la tesi

del teorema de Bolzano no es verifica necessàriament.

Un exemple pot ser x

xf1

)( , amb 1)1( f i 1)1( f , però no existeix cap c tal

que 0)( cf .

2. El teorema de Bolzano es pot usar per buscar les arrels o zeros d’una funció per

aproximacions successives. Si tenim una funció contínua en un interval tancat en els

extrems del qual té signe diferent, segons el teorema té un zero. Si subdividim aquest

interval en n parts (per exemple, en dues), o bé un dels nous extrems és un zero o la

funció canvia de signe en algun dels subintervals. Podem repetir l’operació tantes

vegades com vulguem segons l’error que ens puguem permetre en els nostres càlculs

(aquests mètodes són ideals per als ordinadors).

Problemes PAU on apareix el Teorema de Bolzano.

1. Considereu la funció x

xxxf

1

452)(

3

.

a) Calculeu-ne el domini i estudieu-ne la continuïtat. Té cap asímptota vertical?

b) Observeu que 3

2)2( f , f(0) = 4 i f(2) = –10. Raoneu si, a partir d’aquesta

informació, podem deduir que l’interval (–2, 0) conté un zero de la funció. Podem

deduir-ho per a l’interval (0, 2)? Trobeu un interval determinat per dos enters

consecutius que contingui, com a mínim, un zero d’aquesta funció.

PAU CAT TEC JUNY 2019 1.4 (Solució: "Compendium Tec", Pàg. 533)

2. Sigui la funció 2)( xxxf

a) Comproveu que la funció f (x) compleix l’enunciat del teorema de Bolzano a

l’interval [0, 2] i que, per tant, l’equació f (x) = 0 té alguna solució a l’interval (0, 2).

Comproveu que x = 1 és una solució de l’equació f (x) = 0 i raoneu, tenint en compte el

signe de f '(x), que la solució és única.

b) A partir del resultat final de l’apartat anterior, trobeu l’àrea limitada per la gràfica de

la funció f (x), l’eix de les abscisses i les rectes x = 0 i x = 1.

Solució PAU CAT TEC JUNY 2018 1.5

3. a) Donada la funció f(x) = x3 – 3x, calculeu els punts de tall amb els eixos i els

extrems relatius (si en té), i feu un esbós de la gràfica de la funció.

b) Basant-se en el gràfic anterior, i sense cap més càlcul, raoneu que la funció g(x) = x3

– 3x – 10 talla l'eix de les x en un sol punt.

c) Indiqueu, raonadament, un interval de longitud 1 en el qual es troba la solució real de

l'equació x3 –3x –10 = 0.

Solució PAU CAT CCSS JUNY 2001 5.5

Corol·lari. Teorema dels valors intermedis (Darboux).

Si )(xf és contínua en ba, , aleshores pren tots els valors intermedis entre )(af i

)(bf . Simbòlicament:

kcfbacbfafk )(|,)(),(

Demostració.

És una conseqüència immediata del teorema de Bolzano.

Corol·lari.

Si )(xf i )(xg són funcions contínues en ba, i )()( agaf i )()( bgbf , aleshores

existeix un nombre ),( bac tal que )()( cgcf .

Definició. Màxims i mínims de funcions.

Una funció )(xf té un màxim en el punt c si les imatges de tots els altres punts del

domini són menors o no són superiors. Simbòlicament:

)()(Dom(f) cen màximun té cfxfxf

De la mateixa manera definiríem el que és un mínim d’una funció. Els màxims i els

mínims s’anomenen extrems.

Teorema. Teorema de Weierstrass.

Si )(xf és una funció contínua en un interval tancat ba, , la funció té un màxim i un

mínim absoluts en aquest interval. És a dir, existeixen baMm ,, per als quals es

compleix que

)()()(, Mfxfmfbax

Demostració.

Òbviament, si hem d’unir )(, afa i )(, bfb amb una corda contínua tindrem un punt

més alt i un punt més baix.

Teorema. Teorema de Bolzano generalitzat.

La imatge contínua d’un interval tancat també ho és. Més concretament, si f és contínua

en l’interval ba, , f té un màxim en c, Mcf )( , i un mínim en d, mdf )( , la

imatge de ba, per f és Mm, i la funció és injectiva si i solament si és estrictament

monòtona.

4 Derivada d’una funció en un punt.

4.1 Taxa de variació mitjana (TVM).

Donada una funció )(xf , un punt x i un increment h, definim la taxa de variació

mitjana de la funció en el punt x, en relació a l’increment h com

h

xfhxfhxfTVM

)()(),,(

Per trigonometria, la TVM és la tangent de l’angle :

)tan(TVM o )arctan(TVM

4.2 Concepte de derivada d’una funció en un punt.

4.2.1 Definició. Derivada d’una funció en un punt.

Diem que la funció )(xf és derivable en el punt ax si existeix el límit

h

afhafhafTVMaf

hh

)()(lim),,(lim)('

00

A aquest límit, si existeix, li direm derivada de la funció en a.

4.2.2 Exemple. Derivada de la funció arrel quadrada.

Sigui xxf )( i volem determinar la seva derivada mitjançant la definició anterior.

)1(lim)()(

lim)('00

h

aha

h

afhafaf

hh

Ens trobem davant d’una indeterminació 0/0 amb arrels quadrades. Per resoldre-la,

multipliquem numerador i denominador pel conjugat:

aaaahaahah

h

ahah

aha

ahah

aha

ahah

ahaaha

hhh

hh

2

111limlimlim

limlim)1(

000

22

00

Per tant, si x

xfxxf2

1)(')(

4.2.3 Exemples. Derivada de funcions potencials mitjançant la definició.

1. kxf )(

0)('00lim0

limlim)()(

lim)('0000

xfhh

kk

h

xfhxfxf

hhhh

2. xkxf )(

kxfkkh

hk

h

kxhkxk

h

kxhxk

h

xfhxfxf

hh

hhh

)('limlim

lim)(

lim)()(

lim)('

00

000

3. 2)( xxf

xxfxhxh

xfhxfxf

hxh

hxh

h

hxh

h

xfhxf

hxhxhxhxxfhxf

hxhxhxhxf

hh2)('22lim

)()(lim)('

2)2(2)()(

22)()(

2)()(

00

2

2222

222

4. 3)( xxf

2222

00

2222322

32233223

32233

3)('333lim)()(

lim)('

33)33(33)()(

3333)()(

33)()(

xxfxhxhxh

xfhxfxf

hxhxh

hxhxh

h

hxhxh

h

xfhxf

hxhxhxhxhxhxxfhxf

hxhxhxhxhxf

hh

Apunts d’història del Càlcul Diferencial.

La invenció del càlcul infinitesimal, que compren el càlcul diferencial i el càlcul integral

s'atribueix a Leibniz juntament amb Isaac Newton.

Leibniz Newton

Leibniz començà a treballar en el càlcul l’any 1674. Les seves contribucions

matemàtiques més importants foren publicades entre 1682 i 1692. Generalment

aparegueren a Acta Eruditorum, revista que jugà un paper clau en la seva reputació com

a matemàtic i contribuí a la seva rellevància com a diplomàtic, historiador, teòleg i

filòsof.

Si bé trobem implícita la noció matemàtica de funció en la trigonometria i en les taules

de logaritmes, existents en el temps de Leibniz, aquest fou el primer, entre 1692 i 1694,

que les emprà explícitament per a denotar alguns conceptes geomètrics derivats del de

corba, com per exemple els d’abscissa, d’ordenada, de tangent, de corda i de

perpendicular o normal. Posteriorment, en el segle XVIII, el concepte de funció es

deslligaria d’associacions geomètriques. Leibniz s’avançà també a la consideració de

sistemes d’equacions lineals i al seu tractament pel mètode que després serà conegut

com a eliminació gaussiana.

D’acord amb els quaderns de Leibniz, el dia 11 de novembre de 1675 tingué lloc per

primera vegada l’ús del càlcul integral per a l’obtenció de l’àrea limitada per la corda

definida per una funció, . En relació amb aquest fet, Leibniz introduí diverses notacions

que encara són emprades en l’actualitat: el signe d’integració, , representat mitjançant

una S allargada (la inicial llatina de summa), i el signe de diferencial, d (la inicial llatina

de differentia). A més, Leibniz fou el primer a utilitzar el terme analysis situs, que en el

segle XVIII serà emprat per Euler, en el seu famós treball de 1736 en què resol el

problema dels ponts de Königsberg, i a finals del XIX per Poincaré, en un no menys

famós treball de 1895 al qual donava títol i que fou iniciador de la branca matemàtica

coneguda avui amb el nom de topologia.

Font: Arrels germàniques de la matemàtica contemporània: amb una antologia de... Per Bayer, Pilar,

Guàrdia, Jordi , Travesa, Artur, Secció de Ciències i Tecnologia.

4.3 Creixement, decreixement i extrems relatius.

4.3.1 Definició. Creixement i decreixement d’una funció.

Direm que )(xf és creixent en px si existeix un entorn ),( apap tal que

)()(, 2121 xfxfapapxx

Direm que )(xf és estrictament creixent en px si existeix un entorn ),( apap

tal que

)()(, 2121 xfxfapapxx

Si una funció és creixent, direm que és monòtona.

Anàlogament es defineix decreixent i estrictament decreixent en px .

4.3.2 Proposició.

Si )(xf és derivable

)(xf creixent en px 0)(' pf

)(xf decreixent en px 0)(' pf

Demostració.

Suposem que )(xf és derivable i creixent en px .

Si )()(0 pfhpfphph per definició de funció creixent, i per tant

0)()(

lim)('0)()(

0

h

pfhpfpf

h

pfhpf

h

perquè el límit d’una funció que pren valors positius és positiu o nul.

Un raonament similar serveix per a demostrar el cas de funció decreixent.

4.3.3 Definició. Màxims i mínims relatius.

Direm que )(xf té un màxim relatiu en px si existeix un entorn ),( apap tal

que si )()(, pfxfapapx .

Anàlogament es defineix un mínim relatiu.

Proposició. Condició necessària per ser extrem relatiu.

Si )(xf és derivable en px i hi té un mínim o un màxim relatiu, llavors 0)(' pf .

Demostració. Suposem que la funció té un màxim en px .

Si 0h , per definició de màxim 0)()()()( pfhpfpfhpf .

Per tant 0)()(

lim)('0)()(

0

h

pfhpfpf

h

pfhpf

h

Si 0h , de nou 0)()()()( pfhpfpfhpf .

Per tant 0)()(

lim)('0)()(

0

h

pfhpfpf

h

pfhpf

h

Si la funció és derivable en px llavors 0)(')(')(')(' pfpfpfpf .

Un raonament similar demostra el resultat quan f té un mínim relatiu en px .

Observació. Derivada igual a zero no implica extrem relatiu.

0)(' af no implica necessàriament que sigui un extrem relatiu. Per exemple, la

funció 3)( xxf compleix 23)(' xxf , i la seva primera derivada s’anul·la en 0x ,

però no és cap extrem relatiu, aquesta funció és sempre creixent:

Per garantir que es tracta realment d’un extrem relatiu, hem d’analitzar el signe de la

derivada en les proximitats del punt, a la dreta i a l’esquerra.

4.3.4 Criteri de la primera derivada per a determinar màxims i mínims relatius.

Una funció presenta un extrem relatiu en ax si, en aquest punt, la funció canvia de

creixent a decreixent (màxim relatiu) o de decreixent a creixent (mínim relatiu).

Observació: En 4.6.3 veurem un criteri alternatiu mitjançant la segona derivada.

4.3.5 Observació. Diferència entre )(xf i )(' xf .

Entre )(xf i )(' xf només canvia una tilde, però representen conceptes totalment diferents:

)(xf

Sense tilde ens informa de la altura de la

gràfica al punt x respecte de l’eix X

)(' xf

Amb tilde ens informa de la inclinació de

la gràfica al punt x

0)( xf

Indica que la gràfica de la funció està per

sobre de l’eix X

0)(' xf

Indica que la gràfica de la funció és

creixent

0)( xf

Indica que la gràfica de la funció està per

sota de l’eix X

0)(' xf

Indica que la gràfica de la funció és

decreixent:

0)( xf

Indica que la gràfica de la funció està just a

l’eix X

0)(' xf

Indica que la gràfica de la funció és

horitzontal:

4.3.6 Problemes PAU amb el concepte de derivada.

1. La gràfica corresponent a la derivada d’una funció f(x) és la següent:

a) Expliqueu raonadament quins valors de x corresponen a màxims o a mínims relatius

de f(x).

b) Determineu els intervals de creixement i decreixement de la funció f(x).

Solució CAT TEC JUNY 2011 4.3

2. En la figura següent es representen dues funcions. L’una és la derivada de l’altra.

Decidiu si la funció f(x) és la derivada de la funció g(x) o és a l’inrevés, estudiant què

passa en els punts x = a , x = b i x = c.

Solució PAU CAT TEC JUNY 2010 4.5

3. Teniu una funció f(x) definida per a )2,2(x , sabeu que el gràfic de f '(x) és de la

forma

(on f'(–1) = 0, f'(0) = –1, f'(1) = 1) i que f(0) = 2.

Dibuixeu un gràfic aproximat de f(x) indicant en quins punts hi ha extrems relatius. Solució PAU CAT TEC JUNY 2001 5.2

4. Considereu una funció tal que la seva representació gràfica a l’interval (-3,3) és la

següent:

a) Determineu les abscisses dels punts extrems (màxims i mínims) relatius.

b) Estudieu el creixement i decreixement de la funció a l’interval (-3,3)

c) Feu un esbós de la gràfica de la derivada d’aquesta funció.

d) Sabent que la funció és de la forma cbxaxxf 24)( , trobeu de quina funció es

tracta. Solució PAU CAT TEC JUNY 2008 2.5

5. La gràfica següent correspon a una funció f:[2 , 6] →R derivable i amb derivada

contínua. Feu un esbós de la gràfica de f’:(2 , 6) →R i justifiqueu-ne el perquè.

Solució PAU CAT TEC SET 2004 5.2

6. El dibuix representa les gràfiques de les tres funcions A(x), B(x) i C(x) i de les seves

derivades P(x), Q(x) i R(x), no necessàriament en el mateix ordre.

Associeu cada funció A(x), B(x) i C(x) amb la seva respectiva funció derivada P(x),

Q(x) o R(x). Raoneu la resposta. Solució PAU CAT CCSS JUNY 2004 4.3

7. Observeu la gràfica següent de la funció f(x) i digueu quin valor tenen

(aproximadament)

a) f (0) b) x si f(x) = 0 c) f’(0) d) f’(–2)

Solució PAU CAT CCSS JUNY 2004 3.4

4.4 Propietats de les funcions derivables.

4.4.1 Teorema. Derivabilitat implica continuïtat.

Si una funció )(xf és derivable en un punt, és contínua en aquest mateix punt.

Demostració.

Sigui )(xf una funció derivable en ax .

00)('lim)()(

lim)()(

lim)()(lim

afax

ax

afxfax

ax

afxfafxf

axaxaxax

i per tant )()(lim afxfax

.

4.4.2 Teorema. Teorema de Rolle.

Si f és contínua en ba, , derivable en ba, i )()( bfaf , llavors existeix un bac ,

tal que 0)(' cf .

Demostració.

Si )(xf és constant, la seva derivada és nul·la en tot l’interval ),( ba i el teorema queda

demostrat.

Si )(xf no és constant, en ser contínua, segons el teorema de Weierstrass, tindrà

màxim i mínim i els dos no podran tenir com a imatge el valor )()( bfaf .

Sigui bac , l’extrem tal que )()( afcf . Segons la proposició 4.3.4, 0)(' cf ,

com volíem demostrar.

4.4.3 Teorema. Teorema dels increments finits, o de Lagrange.

Si )(xf és contínua en ba, i derivable en ba, , llavors existeix un bac , tal que

))((')()( abcfafbf

Geomètricament aquest teorema vol dir que existeix un punt de l’interval ba, en el

qual la tangent a la gràfica és paral·lela a la secant per )(, afa i )(, bfb .

Demostració.

Considerem la funció abxfafbfxxg )()()()(

que en ser combinació lineal de dues funcions contínues en ba, i derivables en ba,

també ho és, i a més a més )()( bgag com és fàcil de comprovar.

Pel teorema de Rolle, existirà un bac , tal que 0)(' cg , on

)(')()()()(' xfabafbfxg

és a dir:

)(')()(

)(')()()(0)(')()()()('

cfab

afbf

cfabafbfcfabafbfcg

4.4.4 Corol·lari.

Si )(xf és contínua en ba, i derivable en ba,

a) Si babaxxf ,en creixent és f,0)(' .

b) Si babaxxf ,en decreixent és f,0)(' .

c) Si babaxxf ,en constant és f,0)(' .

Demostració.

a) Siguin baxx ,21 . La funció )(xf serà contínua en 21, xx i derivable en

21, xx , per tant, aplicant el teorema de Lagrange (4.4.2), existirà un 21, xxc tal que

))((')()( 1212 xxcfxfxf

i com que 0)(' cf per hipòtesi, tenim que )()(0)()( 1212 xfxfxfxf .

Les demostracions de (b) i (c) són anàlogues.

4.4.5 Corol·lari.

Si )(xf és contínua en ba, i derivable en ba,

a) Si babaxxf ,en creixent nt estrictame és f,0)(' .

b) Si babaxxf ,en decreixentnt estrictame és f,0)(' .

Demostració.

És el mateix raonament que hem fet servir a la demostració anterior.

4.5 Derivabilitat amb funcions definides a trossos.

4.5.1 Definició. Derivades laterals.

Definim la derivada lateral per la dreta d’una funció )(xf en ax com

h

afhafaf

h

)()(lim)('

0

De la mateixa forma definim la derivada lateral per l’esquerra de la funció )(xf en

ax

h

afhafaf

h

)()(lim)('

0

4.5.2. Proposició.

Perquè existeixi la derivada d’una funció contínua en un punt, les seves derivades

laterals han d’existir i han de ser iguals. El seu valor serà el valor de la derivada en

aquest punt:

)(')(')(' afafaf

4.5.3 Derivabilitat de funcions definides “a trossos”.

Si una funció f(x) està definida “a trossos” per dos funcions h(x) i g(x) derivables en

ax

axsixg

axsixhxf

)(

)()(

Llavors

)(')()(

lim

)(')()(

lim)()(

lim)('

0

0

0

agh

afhaf

ahh

afhaf

h

afhafaf

h

h

h

per tant:

f(x) serà derivable en x = a si )()( agah i )(')(' agah

Es a dir, la funció és contínua en x = a i les derivades laterals coincideixen.

Demostra que la funció xxf )( no és derivable en 0x .

Recordem que

0

0)(

xx

xxxxf , és a dir, tècnicament és una funció definida a

trossos.

)2()0()0(

lim

)1()0()0(

lim)0()0(

lim)0('

0

0

0

h

fhf

h

fhf

h

fhff

h

h

h

(1) 11limlim0

lim0

lim)0()0(

lim00000

hhhhh h

h

h

h

h

hh

h

fhf

(2) 11limlim0

lim0

lim)0()0(

lim00000

hhhhh h

h

h

h

h

hh

h

fhf

Així doncs, els límits laterals d’aquesta funció al voltant del 0 no coincideixen. Això

queda reflectit a la seva gràfica amb un punt “que punxa”. A la dreta del 0x té

pendent 1 (45º) i a l’esquerra del 0x té pendent –1 (-45º). La inclinació canvia de

forma brusca al voltant de 0x .

55017

53)(

2

2

xsixx

xsixxxf

És contínua i derivable en x = 5

Segon exemple:

5253

53)(

2

xsix

xsixxxf

És contínua però no és derivable en x = 5

Visualment, en x = a la gràfica de la funció és suau si és derivable i punxa si no ho és.

Donada la funció

21611

21

10

)( 3

xsix

xsibxax

xsi

xf

a) Trobeu a i b per als què la funció sigui contínua en tot x real.

b) Estudieu la derivabilitat d’aquesta funció.

a) Fora dels punts frontera 1x i 2x , la funció són polinomis, i per tant

perfectament contínua.

Continuïtat en 1x :

0

1)1(

00)(lim

)(lim

11

1

3

1

ba

f

xf

babxaxxf

xx

xx

Continuïtat en 2x :

34628

6)2(

28)(lim

61611)(lim

2

3

2

22

baba

f

babxaxxf

xxf

xx

xx

1,134

0

ba

ba

ba

b) Fora dels punts frontera 1x i 2x , la funció són polinomis, i per tant

perfectament derivable.

Derivabilitat en 1x

0200)1('

213)1('

1

1

3

x

x

f

xfLa funció no és derivable en 1x

Derivabilitat en 2x

11111113)2('

1111)2('

2

2

2

x

x

xf

fLa funció sí és derivable en 2x

4.5.6 Extrems relatius amb funcions definides a trossos.

Estudia la continuïtat i determina els extrems relatius de la funció

1104

133)(

2

23

xxx

xxxxxf

La funció té un extrem relatiu a x=2, i també té un extrem relatiu al punt frontera x=1.

Observem que no té cap extrem relatiu al punt x=-1 tot i què en aquest punt s’anul·la la

derivada.

4.5.7 Exercicis PAU TEC sobre continuïtat i derivació amb funcions definides a

trossos.

1. Trobeu els valors dels paràmetres a i b per tal que la funció següent sigui contínua i

derivable en x = 2.

25

232)(

3

2

xsixbx

xsixaxxf

Solució PAU CAT TEC JUNY 2008 5.1

2. Considereu la funció

0

0)(

2

xsiea

xsibxxxf

xb

on a i b són nombres reals.

a) Quina condició han de complir a i b per tal que f sigui contínua a tot ?

b) Trobeu els valors de a i b per als quals f sigui contínua però no derivable a tot .

c) Per a a = 1 i b = 1, calculeu 1

1)( dxxf

Solució PAU CAT TEC SET 2005 3.6

3. Considereu la funció f(x) de la figura definida a l’interval 2,0 .

a) Calculeu la funció derivada )(' xf a l’interval (0,2).

b) Hi ha algun punt de (0,2) en el qual )(' xf no existeixi?

c) Calculeu 2

0)( dxxf .

Solució PAU CAT TEC JUNY 2004 4.4

4. Considereu la funció definida per

012

0)(

xsix

xsiexf

xa

on a és un nombre real.

a) Calculeu )(lim0

xfx

i comproveu que f(x) és contínua en x = 0.

b) Per a quin valor del paràmetre a la funció f(x) és derivable en x = 0 ? Solució PAU CAT TEC SET 2001 4.3

5. D’una funció )(xfy sabem

a) El seu domini de definició és tot .

b) La seva funció derivada és

11

12)('

xsi

xsixf

c) f(x) és contínua en tot punt i )1(f = 2.

Determineu el valor de )1(f i dibuixeu la gràfica de la funció f(x).

Solució PAU CAT TEC SET 2000 6.1

6. La gràfica d’una funció és la que hi ha en el dibuix següent. Quina és la gràfica de la

funció derivada? En quins punts és discontínua la derivada ?

PAU CAT TEC SET 1999 2.1

7. La funció derivada )(' xf de certa funció contínua :f és una funció a trossos

formada per les semirectes del dibuix.

a) Digueu si f(x) és derivable en tots els punts de i per què.

b) Estudieu el creixement i el decreixement de f(x).

c) Trobeu si f(x) té algun extrem relatiu i, si és així, per a quin valor de x i de quin tipus.

d) Sabent que f(0)=1, calculeu el valor de f(1).

Justifiqueu totes les respostes.

Solució PAU CAT TEC JUNY 2007

8. Trobeu els valors dels paràmetres a i b per tal que la funció següent sigui contínua i

derivable en x= 2

25

232)(

3

2

xsibxx

xsixaxxf

Solució PAU CAT TEC JUNY 2008 5.2

4.5.8 Exercicis PAU CCSS sobre continuïtat i derivació amb funcions definides a

trossos.

1. Considereu la funció següent:

01

02)(

2

xsie

xsibxxxf

x

a) Determineu el valor de b perquè la funció f sigui contínua en el punt x= 0. Justifiqueu

si f pot ser discontínua en algun altre punt.

b) Justifiqueu si, per a valors positius de x, la funció f és creixent o decreixent.

Solució PAU CAT CCSS JUNY 2010 1.2

2. Considereu la funció:

023

022)(

2 xsixx

xsixxf

a) Dibuixeu la gràfica.

b) Estudieu-ne la continuïtat.

c) Determineu els extrems relatius.

Solució PAU CAT CCSS JUNY 2006 3.4

4.6 Curvatura.

4.6.1 Definició visual.

Una funció és còncava en un punt ax si la gràfica de la funció al voltant d’aquest

punt està doblegada cap avall.

Una funció és convexa en un punt ax si la gràfica de la funció al voltant d’aquest

punt està doblegada cap amunt.

Equivalentment, una funció és convexa quan la recta que uneix dos dels seus punts

passa sempre per sobre de la gràfica.

Equivalentment, una funció és convexa quan la recta tangent associada a qualsevol punt

passa per sota de la seva gràfica.

Definició formal.

Una funció és convexa en un punt ax si la segona derivada de la funció en aquest

punt és positiva: 0)('' af

Una funció és còncava en un punt ax si la segona derivada de la funció en aquest

punt és negativa: 0)('' af

Una funció presenta un punt d’inflexió en ax si, en aquest punt, la funció canvia la

seva curvatura, és a dir, passa de ser còncava a convexa, o viceversa.

4.6.2 Determinació dels punts d’inflexió.

Si una funció presenta un punt d’inflexió en ax , es compleix que 0)('' af . Per

garantir que es tracta realment d’un punt d’inflexió, analitzarem el sentit de la

concavitat en les proximitats del punt.

Atenció! Segona derivada igual a zero no implica necessàriament punt d’inflexió. Per

exemple, la funció 4)( xxf compleix 212)('' xxf , i la seva segona derivada

s’anul·la en 0x , però no és un punt d’inflexió, aquesta funció és sempre convexa:

Criteri de la tercera derivada.

Una funció té un punt d’inflexió en ax si la segona derivada s’anul·la i la tercera

derivada és diferent de zero en aquest punt.

0)('''0)('' afiaf la funció té un punt d’inflexió en ax

Com determinem la curvatura d’una funció.

1. Calculem la seva segona derivada )('' xf

2. Resolem l’equació 0)('' xf

3. Fem un estudi per intervals:

Marquem els punts x trobats al punt anterior en una recta.

Atenció! També hem d’afegir els punts de discontinuïtat de la funció!

Agafem valors auxiliars a cada interval i els avaluem amb la segona derivada.

Si la segona derivada és positiva, la funció serà convexa a tot l’interval

Si la segona derivada és negativa, la funció serà còncava a tot l’interval

Els punts d’inflexió seran els punts on canvia la curvatura.

4. Calculem els valors Y dels punts d’inflexió per poder representar-los a la gràfica.

4.6.3 Criteri de la segona derivada per determinar màxims i mínims relatius.

Una funció és còncava al voltant d’un màxim relatiu i convexa al voltant d’un mínim

relatiu, per tant:

Si 0)(' pf i 0)('' pf llavors la funció té un mínim relatiu en px .

Si 0)(' pf i 0)('' pf llavors la funció té un màxim relatiu en px .

Controvèrsia entre què és còncava i què es convexa.

No hi ha un criteri universal sobre la denominació “funció còncava” i “funció convexa”.

La Gran Enciclopèdia Catalana les defineix de la forma convencional, és a dir, bé:

còncava = corbada cap a vall , convexa = corbada cap a dalt

Però podem trobar alguns llibres de text de 2n de batxillerat les defineixen mal:

Editorial Barcanova Editorial Santillana

5 Funcions polinòmiques amb derivació.

5.1 Primera derivada de funcions polinòmiques.

1)(')( n

n

n

n xanxfxaxf

1

2

1

1

01

1

1 ...)1()('...)( axanxnaxfaxaxaxaxf n

n

n

n

n

n

n

n

1. Determina l’angle de la funció per al valor de x indicat:

a) 4)( 2 xxxf , 9.1x

b) 45)( 2 xxxf , 5.3x

c) 334)( 23 xxxxf , 7.2x

d) 32)( 23 xxxf , 5.1x

2. Determina el valor de x donat el valor f’(x) per a aquest valor.

a) 32)( 2 xxxf , 6.2)(' xf

b) 25)( xxxf , 6.1)(' xf

c) 3452)( 23 xxxxf ,

5.1)(' xf , ( 0x )

d) 132)( 23 xxxxf ,

7.2)(' xf , ( 0x )

3. Determina l’extrem relatiu P=(x,y) de la següent funció

a) 252)( 2 xxxf

b) 4)( 2 xxxf

c) 15)( 23 xxxxf

d) 35)( 23 xxxxf

5.2 Estudi de funcions polinòmiques mitjançant derivació.

5.2.1 Exemples resolts.

Estudi de la funció 23)( 3 xxxf

a) Domini de la funció: Tot IR, perquè és polinòmica.

b) Punt de tall amb l’eix Y: 2)0( f

c) Punts de tall amb l’eix X: 1,20230)( 3 xxxxxf (Per Ruffini)

d) Creixement, decreixement i extrems relatius.

1,10)('

33)(' 2

xxxf

xxf

Fem un estudi per intervals:

)1,( 1x )1,1( 1x ),1(

Agafem, per

exemple, 2x

09)2(' f

Agafem, per

exemple, 0x

03)0(' f

Agafem, per

exemple, 2x

09)2(' f

Creixent Decreixent Creixent

De la taula anterior deduïm que la funció té un màxim relatiu en 1x , 4)1( fy

i un mínim relatiu en )0,1( , 0)1( fy

e) Gràfica.

Estudi i representació de la funció 35 53)( xxxf

Solució:

a) )( fDom , una funció polinòmica no dóna problemes de domini.

b) Signe i zeros de la funció:

3

5,

3

5,005353)( 2335 xxxxxxxxf

c) Creixement i decreixement:

1,1,00)('1515)(' 24 xxxxfxxxf

creixent f0)('1, xf decreixent f0)('1,1 xf

creixent f0)(',1 xf

La funció té un mínim en (1,-2) i un màxim en (-1,2).

d) Concavitat-convexitat:

xxxf 3060)('' 3 2

1,

2

1,00)('' xxxxf

còncava f0)(''2

1,

xf convexa f0)(''0,

2

1

xf

convexa f0)(''2

1,0

xf còncava f0)('',

2

1

xf

La funció té punts d’inflexió a 0,0 ,

24

7,

2

1 i

24

7,

2

1

e) Gràfica

5.2.2 Exercici.

Estudi de les següents funcions (Punts de tall amb els eixos, monotonia, extrems

relatius, curvatura i punts d’inflexió):

1. xxxxf 240848)( 23

2. 78)( 24 xxxf

3. xxxf 4)( 3

4. 924228)( 234 xxxxxf

5. 33)( xxxf

6. 424)( xxxf

7. 34 2)( xxxf

8. 36)( xxxf

5.3 Problemes PAU TEC amb funcions polinòmiques.

1. Donada la funció cbxaxxxf 23)(

a) Determineu la relació que han de complir els paràmetres a, b i c perquè f(x) tingui un

extrem relatiu en el punt d’abscissa 1x .

b) Calculeu el valor del paràmetre a perquè hi hagi un punt d’inflexió de la funció f(x)

en el punt d’abscissa 0x .

c) Determineu la relació entre els paràmetres a, b i c sabent que la gràfica de f(x) talla

l’eix OX en el punt d’abscissa 2x .

d) Calculeu el valor dels paràmetres a, b i c perquè es compleixin les tres propietats

anteriors alhora. Solució PAU CAT TEC SET 2011 2.3

2. Sigui cbxaxxP 2)( un polinomi qualsevol de segon grau.

a) Trobeu la relació existent entre els paràmetres a, b i c sabent que es compleix que

0)1( P i 0)2( P

b) Quan es compleix la condició anterior, indiqueu quins valors pot tenir )2/3('P

Solució PAU CAT TEC JUNY 2010 4.3

3. Busqueu els extrems relatius i els punts de tall amb els eixos, i feu una representació

aproximada de la corba d’equació y=x4–x

2. A continuació, calculeu l’àrea del

recinte tancat per aquesta corba i l’eix d’abscisses. Solució PAU CAT TEC JUNY 2007 1.3

4. Trobeu els màxims i mínims relatius de la funció f(x)= 6x5 –15x

4 +10x

3

Solució PAU CAT TEC JUNY 2005 4.3

5. Considereu la funció polinòmica de tercer grau, f(x)= ax3

+bx2

+cx +d , (a≠0).

a) Trobeu els valors de a, b, c i d per als quals f(x) talla l’eix OX en els punts x = 0 i x =

1 i presenta un mínim relatiu en el punt x = 0.

b) Feu un esbós de la gràfica de la funció que heu trobat, i cabeu de calcular els

elements necessaris per dibuixar-la. Solució PAU CAT TEC SET 2004 5.5

6. Sabent que la funció y = (x + a)(x2

– 4), on a és un nombre real, té un màxim i un

mínim relatius, i que el màxim relatiu s'assoleix en el punt 3

1x , trobeu l'abscissa del

mínim relatiu. Solució PAU CAT TEC JUNY 2002 2.2

7. a) Trobeu els extrems relatius de la funció polinòmica 362

3)( 23 xxxxf

i calculeu els valors de f(x) en aquests punts. A partir d'aquestes dades, feu un dibuix

aproximat de la seva gràfica.

b) Demostreu que l'equació 0362

3 23 xxx té, exactament, tres solucions reals.

PAU CAT TEC 2000 2.1

5.4 Problemes PAU CCSS amb funcions polinòmiques.

1. Una botiga obre al públic des de les 10 hores fins a les 21 hores. Sabem que els

ingressos per vendes, en funció de l’hora del dia, venen donats per la funció:

I(t) = –5(m – t)2 + n,

per a 10 ≤ t ≤ 21.

a) Trobeu el valor de m sabent que els ingressos màxims es produeixen a les 18 hores.

b) Trobeu el valor de n sabent que a les 21 hores hi ha uns ingressos de 500 €.

PAU CAT CCSS JUNY 2019 1.6 (Solució: "Compendium CCSS", Pàg. 429)

2. Considereu una funció f(x) que té com a primera derivada 42)(' 2 bxxxf , en

què b és un paràmetre real.

a) Determineu el valor de b perquè f(x) tingui un extrem relatiu en x = –1 i raoneu si es

tracta d’un màxim o d’un mínim.

b) Si sabem que la gràfica de la funció f(x) passa pel punt (0, 3), trobeu l’equació de la

recta tangent a f(x) en aquest punt.

PAU CAT CCSS SET 2019 5.5 (Solució: "Compendium CCSS", Pàg. 437)

3. Una companyia de mòbils va presentar fa un any un telèfon intel·ligent al preu de 750

€. Recentment, un estudi de mercat ha arribat a la conclusió que, amb aquest preu,

compren el telèfon 2.000 clients al mes, i que la relació entre aquestes dues variables és

lineal, de manera que per cada 10 € que s’incrementa el preu del mòbil, el compren 100

clients menys, i a l’inrevés: per cada 10 € de descompte sobre el preu inicial de 750 €, el

compren 100 clients més.

a) Deduïu que la funció que determina els ingressos mensuals de la companyia segons

el preu del mòbil és I(p) = –10p2 + 9.500p.

b) Trobeu quin ha de ser el preu del mòbil per a obtenir ingressos, el preu del mòbil que

dona els ingressos mensuals més elevats i el valor d’aquests ingressos màxims.

Solució PAU CAT CCSS SET 2018 1.5

4. Sigui y = f(x) una paràbola que té el vèrtex en el punt V = (0, −4) i talla l’eix de les

abscisses en els punts (−2, 0) i (2, 0).

a) Determineu-ne l’equació.

b) Sigui una funció g tal que g '(x) = f(x). Estudieu el creixement de la funció g,

determineu-ne les abscisses dels extrems relatius i classifiqueu-los.

Solució PAU CAT CCSS JUNY 2018 5.3

5. Un gimnàs cobra una quota de 42 euros mensuals i té 2.000 usuaris. Un estudi de

mercat afirma que per cada euro que s’apuja (o s’abaixa) la quota es perden (o es

guanyen) 20 usuaris.

a) Expresseu el nombre d’usuaris del gimnàs en funció de la quota, tenint en compte

que la relació entre les dues variables és lineal. Per a quin valor de la quota el gimnàs es

quedaria sense usuaris?

b) Determineu en quin preu cal fixar la quota per a obtenir un benefici mensual màxim.

Quin seria aquest benefici i quants usuaris tindria el gimnàs en aquest cas?

Solució PAU CAT CCSS SET 2017 2.2

6. El vèrtex d’una paràbola és el punt (1, 2).

a) Si la paràbola talla l’eix de les abscisses pel punt

0,

2

1, quin serà l’altre punt de

tall de la paràbola amb l’eix de les abscisses?

b) Trobeu l’equació de la paràbola.

Solució PAU CAT CCSS SET 2017 2.6

7. D’una funció y = f(x) sabem que la seva derivada és f ’(x) = x3 – 4x.

a) Determineu els intervals de creixement i de decreixement de la funció y = f(x).

b) Determineu les abscisses dels seus extrems relatius i classifiqueu-los.

Solució PAU CAT CCSS JUNY 2017 1.1

8. Des d’una barca es dispara una bengala de salvament marítim que s’apaga al cap de 4

minuts. En aquest interval de temps, es comprova que la intensitat lumínica de la

bengala en funció del temps, mesurada en percentatges del 0 % al 100 %, queda

perfectament descrita per l’expressió L(t) = 25 · t · (4 – t), en què el temps t varia entre

0 i 4 minuts.

a) Calculeu per a quin valor de t el percentatge d’intensitat lumínica serà màxim.

b) Si des de la costa la bengala només és visible quan la seva intensitat lumínica és

superior al 75 %, quin és l’interval de temps en què serà visible des de la costa i, per

tant, serà més factible el salvament?

Solució PAU CAT CCSS JUNY 2017 1.2

9. Determineu els valors de a, b i c que fan que la funció f(x) = x3 +ax

2 +b +c passi pel

punt (0, 4) i tingui extrems relatius en els punts d’abscissa x = 1 i x = 3. Classifiqueu

aquests extrems.

Solució PAU CAT CCSS SET 2016 5.3

10. La fotografia matemàtica següent sembla indicar que les branques de les ulleres

formen una paràbola. Tanmateix, no totes les corbes en forma de «U» són paràboles.

Hem marcat sobre uns eixos de coordenades alguns dels punts: (0, 2,5), (1, 0), (3, –1) i

(5, 0).

Justifiqueu si la gràfica correspon a una paràbola o no.

Solució PAU CAT CCSS SET 2016 1.3

11. Un hotel cobra 45 € per habitació i nit. Per aquest preu, té ocupades 165 habitacions

cada nit. S’ha fet un estudi a partir del qual s’ha deduït que, per cada euro que s’apugi el

preu de l’habitació, se n’ocuparà una menys cada nit.

a) Si x és la quantitat que s’apuja el preu de l’habitació per sobre dels 45 € inicials,

determineu la funció que dóna els ingressos diaris de l’hotel segons el valor de x.

Indiqueu també els ingressos màxims que pot obtenir l’hotel.

b) Indiqueu entre quins preus obtindrà ingressos l’hotel. Solució PAU CAT CCSS SET 2016 5.2

12. Una fàbrica de mobles de cuina ven 1.000 unitats mensuals d’un model d’armari a

200 € per unitat. Per tal de reduir-ne l’estoc, fa una oferta als compradors i estima que,

per cada euro de reducció del preu, les vendes mensuals del producte s’incrementaran

en 100 unitats.

a) Quantes unitats caldrà vendre per a obtenir el màxim d’ingressos mensuals?

b) A quant pujaran aquests ingressos?

Solució PAU CAT CCSS JUNY 2016 3.1

13. Un arbre té un volum de 30 m3 i, per la qualitat de la seva fusta, es ven a 50 € per

metre cúbic. Cada any l’arbre augmenta el volum en 5 m3. Alhora, la qualitat de la fusta

de l’arbre disminueix, i també el preu, que cada any és un euro per metre cúbic més

barat. D’aquí a quants anys aconseguirem el màxim d’ingressos per la venda de la fusta

de l’arbre? Quins seran aquests ingressos?

Solució PAU CAT CCSS JUNY 2016 2.1

14. Una cadena de televisió decideix emetre un nou programa en la franja horària de les

17.00h a les 21.00h. El percentatge d’audiència P de la primera emissió en funció del

temps t, mesurat en hores, és definit per la funció

21173690760495

1)( 23 tttttP .

Els directius de la cadena acorden que el programa se seguirà emetent si en algun

moment s’aconsegueix un percentatge d’audiència superior al 20%.

a) Expliqueu raonadament en quins intervals de temps l’audiència del programa va

augmentar i en quins intervals va disminuir.

b) En vista dels resultats, se seguirà emetent el programa? Justifiqueu la resposta.

Solució PAU CAT CCSS JUNY 2014 3.3

15. Els dos darrers anys, el valor de les accions en borsa d’una empresa ha baixat un

20% anual.

a) Aquest any, en canvi, les accions han pujat un 30 %. Quin és el percentatge global de

pèrdua en aquests tres anys?

b) Quin hauria de ser el percentatge de guanys d’aquest tercer any si el balanç global

dels tres anys acaba sent equilibrat, és a dir, sense pèrdues ni guanys?

Solució PAU CAT CCSS SET 2013 1.3

16. Una empresa agrícola ha recollit un total de 40 tones de fruita que produeixen un

benefici de 0,80 €/kg. Cada setmana que passa es produeix una pèrdua de 400 kg de

fruita, però el benefici augmenta en un cèntim per cada kilogram.

a) Quin benefici s’obté si es ven la fruita al cap de nou setmanes? Quin percentatge de

fruita s’ha hagut de llençar?

b) Quina setmana de venda serà la que obté un benefici màxim?

Solució PAU CAT CCSS JUNY 2013 3.4

17. Donades les funcions xkxxxf )3(5)( 23 i kxxxg 2)( .

a) Determineu les abscisses dels punts de tall de les dues corbes.

b) Determineu k perquè la paràbola donada per la funció g tingui el vèrtex en el punt

d’abscissa x= 2, i determineu-ne l’ordenada. Solució PAU CAT CCSS JUNY 2013 3.2

18. Els beneficis d’una companyia de transport de viatgers són donats per la funció

cbxaxxB 2)( , on x és el preu que la companyia cobra per cada viatge. Sabem que

si cobren 40 € per viatge, els beneficis són 19.000 €. A més, si augmentem el preu un 25

%, el benefici que s’obté és el màxim, 20.000 €. Tenint en compte aquestes dades,

determineu els valor de a, b i c.

Solució PAU CAT CCSS JUNY 2013 4.4

19. En un hort hi ha plantades 50 pomeres. Cada arbre produeix 800 pomes. Per cada

arbre addicional que hi plantem, la producció de cada arbre es redueix en 10 pomes.

Quants arbres més ens cal plantar per a obtenir la producció més alta possible? Quina és

aquesta producció? Solució PAU CAT CCSS JUNY 2013 4.3

20. Un equip científic ha estudiat l’evolució de la població d’una petita illa de la

Polinèsia. Com a conclusió, ha determinat que, per tal d’obtenir una bona estimació de

la població, cal fer servir l’expressió 2

3

618400)( tttP , on t indica els anys

transcorreguts des del principi de l’estudi.

a) Determineu la població de l’illa quan va començar l’estudi, i al cap d’un any. Quina

ha estat la taxa de creixement en aquest període?

b) Al cap de quants anys després del començament de l’experiment va deixar de créixer

la població de l’illa? Quin va ser el nombre màxim d’habitants?

Solució PAU CAT CCSS JUNY 2013 4.1

21. Sigui f una funció polinòmica de grau 3, amb un màxim a (0, 0) i un mínim a

(2, –4).

a) Feu una gràfica aproximada de f.

b) Determineu la fórmula de la funció.

Solució PAU CAT CCSS JUNY 2012 3.5

22. Un estudi de laboratori sobre la propagació d’una espècie de mosques mostra que,

passades t setmanes, el nombre d’individus és N(t) centenars de mosques, en què

9)2()( 2 ttN

a) Quantes mosques formen la població al cap d’una setmana? Quantes setmanes han de

transcórrer fins a la desaparició total de les mosques?

b) Quina és la població màxima d’individus? Quantes setmanes han hagut de passar per

a obtenir aquesta població màxima?

Solució PAU CAT CCSS SET 2011 2.1

23. Una empresa que fabrica bicicletes ven la totalitat de la producció. Anomenarem x

el nombre de bicicletes que fabrica mensualment. Els costos mensuals de producció, en

euros, segueixen la funció C(x)= 180x +12000. La venda de les bicicletes li reporta uns

ingressos que segueixen la funció 2

2

1500)( xxxI . Els beneficis de l’empresa són,

lògicament, la diferència entre ingressos i costos.

a) En quin interval cal situar la producció per a no perdre diners?

b) Quantes bicicletes ha de produir mensualment l’empresa per a obtenir el benefici

màxim? En aquest cas, quant guanya per cada bicicleta?

Solució PAU CAT CCSS JUNY 2011 1.5

24. Considereu la funció f(x)= x3 –ax

2 +9x +b.

a) Determineu a i b, sabent que la gràfica de f passa pel punt (2, 2) i té un extrem en

1x .

b) Per a a=6 i b= 0, determineu els possibles màxims i mínims de f i classifiqueu-los.

Solució PAU CAT CCSS SET 2010 2.2

25. Un fons d’inversions posa en marxa un producte financer que aporta un benefici de

R(x) euros en fer una inversió de x centenars d’euros, segons la funció

R(x)= –0,01x2

+4x +20.

a) Calculeu quina inversió produeix més beneficis.

b) Calculeu el tant per cent de benefici que s’obtindrà amb una inversió de 1000€, i el

que s’obtindrà amb una de 10000€. Solució PAU CAT CCSS SET 2010 2.3

26. En una explotació ramadera es declara una epidèmia, i els veterinaris preveuen que

la propagació d’aquesta seguirà la funció f(x) = –2x2 +48x +162, en què x representa el

nombre de setmanes que han transcorregut des del moment de la declaració de

l’epidèmia, i f(x) indica el nombre d’animals afectats.

a) Quants animals hi ha afectats en el moment de declarar-se l’epidèmia? Quantes

setmanes durarà l’epidèmia fins al moment en què ja no quedi cap animal afectat?

b) Indiqueu quin serà el nombre màxim d’animals afectats, i en quina setmana es

produirà. Solució PAU CAT CCSS JUNY 2010 4.6

27. El preu de cost d’una unitat d’un cert producte és de 120€. Si es ven a 150€ la

unitat, el compren 500 clients. Per cada 10€ d’augment en el preu, les vendes

disminueixen en 20 clients.

a) Trobeu una fórmula mitjançant la qual obtinguem els beneficis.

b) Calculeu a quin preu p per unitat hem de vendre el producte per a obtenir un benefici

màxim.

c) En el cas anterior, trobeu el nombre d’unitats que es venen i calculeu el benefici

màxim. Solució PAU CAT CCSS JUNY 2009 4.6

28. Una fàbrica de televisors ven cada aparell a 300€. Les despeses derivades de

fabricar x televisors són D(x) = 200x+x2, en què 0≤x≤80.

a) Suposant que es venen tots els televisors que es fabriquen, trobeu la funció dels

beneficis que s’obtenen després de fabricar i vendre x televisors.

b) Determineu el nombre d’aparells que convé fabricar per a obtenir el benefici màxim,

i també quin és aquest benefici màxim.

Solució PAU CAT CCSS JUNY 2008 5.4

29. La gràfica següent representa una funció polinòmica de segon grau (paràbola).

a) Trobeu el vèrtex de la paràbola i les interseccions amb els eixos.

b) Determineu l’equació de la paràbola.

Solució PAU CAT CCSS SET 2008 4.4

30. La funció f(x) = ax3

+bx2

+cx té un màxim en el punt (1, 4) i passa pel punt (3, 0).

Trobeu a, b i c.

Solució PAU CAT CCSS JUNY 2007 1.4

31. Si el preu de l’entrada d’un cinema és de 6€, hi van 320 persones. El propietari sap

per experiència que per cada augment de 0,25€ en el preu de l’entrada hi van 10

espectadors menys. Trobeu:

a) la funció que determina el nombre d’espectadors en funció del preu de l’entrada;

b) la funció que determina els ingressos del cinema en funció del preu de l’entrada;

c) el preu de l’entrada per tal que els ingressos del propietari siguin màxims;

d) el nombre d’espectadors que aniran al cinema quan el preu sigui el que correspon als

ingressos màxims i aquests ingressos màxims.

Solució PAU CAT CCSS JUNY 2006 1.5

32. El benefici B(x) (expressat en milers d’euros) que obté una empresa per la venda de

x unitats d’un determinat producte és representat per la funció:

B(x) = –x2+ 300x–16100 per a 50≤x≤250.

a) Si ha venut 110 unitats, quin benefici ha obtingut?

b) Quantes unitats pot haver venut si el benefici obtingut ha estat de 3.900 milers

d’euros?

c) Quantes unitats ha de vendre per tal que el benefici sigui màxim? Quin és aquest

benefici màxim?

d) Quina quantitat d’unitats ha de vendre per no tenir pèrdues?

Solució PAU CAT CCSS JUNY 2006 3.5

33. Si una joguina es ven a 130€, la compren 1000 persones. Per cada euro que

augmenta aquest preu, disminueix en 50, respectivament, el nombre de compradors.

a) Feu un gràfic del nombre de joguines que es venen en funció del preu de venda i

doneu la fórmula que l’expressa.

b) El preu de cost d’una joguina és de 80 €. Calculeu el preu p, que dóna un benefici

total màxim.

c) Trobeu el nombre de joguines que es venen si el preu és p i calculeu-ne el benefici

màxim.

Solució PAU CAT CCSS SET 2005 3.5

34. Trobeu els valors de b i c per tal que la funció f(x) = x2 + bx + c tingui un extrem

relatiu en el punt (–1, –4). Quin tipus d’extrem és?

Solució PAU CAT CCSS JUNY 2004 4.4

35. Com a resultat del test efectuat amb un nou model d’automòbil per determinar-ne el

consum de benzina, s’ha observat que, per a velocitats compreses entre 25 i 175 km/h,

el consum C(x) de gasolina, expressat en litres consumits en 100 km, fets a la velocitat

constant de x km/h, es pot aproximar per la funció

C(x) = 7,5 –0,05x +0,00025x2

a) Determineu el consum a les velocitats de 50 km/h i de 150 km/h.

b) A quina velocitat s’obté el mínim consum? Quin és aquest consum mínim?

c) Feu un estudi del creixement i decreixement de la funció C(x) a l’interval [25,175].

Determineu les velocitats que corresponen a consum màxim, així com aquest consum.

Solució PAU CAT CCSS JUNY 2003 2.5

36. La funció f (x) = –5ax

2 +700x +1440 té un extrem relatiu per x = 10. Calculeu el

valor de a. Solució PAU CAT CCSS JUNY 2003 5.2

37. El preu de cost d'una joguina és de 80 €. Venuda a 130 € la compren 1000 persones.

Per cada € que augmenta o disminueix aquest preu, el nombre de compradors

disminueix o augmenta, respectivament, en 60.

a) A quin preu s'ha de vendre la joguina per obtenir un benefici màxim?

b) Calculeu també el benefici màxim i el nombre de compradors corresponent.

Solució PAU CAT CCSS JUNY 2002 3.5

38. Calculeu a i b sabent que la funció f(x) = ax2 + bx – 7 té un extrem local en el punt

(2, 1). És un màxim o un mínim? Solució PAU CAT CCSS SET 2001 4.2

39. Una fàbrica de vídeos decideix introduir al mercat un nou model. El departament de

màrqueting de l'empresa estima que la relació entre la demanda x del producte,

mesurada en unitats, i el preu de venda p de cada unitat, mesurat en milers de ptes., ve

donada per l'expressió

12100

xp

Els costos de producció estimats responen a la fórmula

C(x) = 1250 + 10x

Determineu:

a) la demanda x en funció de p;

b) els costos C(p) en funció del preu;

c) els ingressos I(p) que rep el fabricant per la venda d'aparells, en funció del preu;

d) el benefici B(p) del fabricant per la venda dels aparells, en funció del preu;

e) el preu pel qual el benefici és màxim, el benefici màxim i el nombre d'unitats venudes

corresponent. Solució PAU CAT CCSS SET 2001 4.5

40. En una indústria es produeixen recanvis de peces d'automòbil. S'ha fet un estudi de

costos d'un dels recanvis fabricats i ha resultat que el cost diari de producció de x peces

(en ptes.) ve donat per la funció C(x) = 3200 + 20x + 2x2.

a) Quantes peces d'aquest recanvi s'han de produir diàriament perquè el cost unitari (el

cost de cada peça) sigui el mínim possible?

b) Quin és el cost diari de fabricar aquest nombre de peces?

c) Quin és, en aquest cas, el preu de cost de cada peça?

Solució PAU CAT CCSS JUNY 2001 2.5

41. Considereu la funció 32)1( xxy . Digueu quin és el seu domini de definició.

Calculeu els seus intervals de creixement i decreixement, així com els màxims i mínims

(si en té). Calculeu també els punts en què la gràfica talla els eixos. Feu després un

esbós d'aquesta gràfica.

PAU CAT CCSS SET 2000 2.6

42. Una entitat financera llança al mercat un pla d'inversió, la rendibilitat R(x) del qual,

en milers de pessetes, ve donada en funció de la quantitat x que s'inverteixi, per mitjà de

l'expressió següent:

R(x) = –0.001x2 + 0.5x + 2.5

a) Deduïu raonadament quina quantitat de diners li convé invertir a un client en aquest

pla per obtenir rendibilitat màxima.

b) Quina rendibilitat obtindria en aquest cas?

Solució PAU CAT CCSS JUNY 2000 3.1

43. Una empresa que fabrica un determinat producte ha observat experimentalment que

la venda de x milers d'unitats diàries d'aquest producte li produeix uns ingressos en

milions de pessetes donats per la funció següent:

5227

28

108

235

54

11)( 2 xxxxI

Així mateix el cost de fabricació de x milers d'unitats diàries del producte el dóna

(també en milions de pessetes) la funció

5214

3)( xxxC

Determineu quants milers d'unitats hauria de fabricar d'aquest producte per obtenir un

benefici màxim.

Solució PAU CAT CCSS JUNY 1999 6.5

44. Trobeu els nombres a i b de manera que la funció f(x)=ax

2 +bx tingui un màxim en

el punt (3, 9).

Solució PAU CAT CCSS JUNY 1998 3.3

45. En una fàbrica determinada, el cost de producció expressat en pessetes de x unitats

d'un producte s'ajusta aproximadament a la funció

0,16000)( 3 xaperxxC

a) Feu un esquema senzill de la gràfica d'aquesta funció (només per a x≥0).

b) Trobeu la funció que representa el cost per unitat fabricada. Trobeu el cost mínim per

unitat fabricada.

Solució PAU CAT CCSS JUNY 1998 6.5

6 Funcions racionals amb derivació.

6.1 Derivada del producte, potència i quocient de funcions.

Derivada del producte de funcions:

)(')()()(')(')()()( xhxgxhxgxfxhxgxf

Derivada d’una potència d’una funció:

)(')()(')()( 1 xgxgnxfxgxf nn

Derivada del quocient de funcions:

2)(

)(')()()(')('

)(

)()(

xh

xhxgxhxgxf

xh

xgxf

1. Exercici. Calcula )(' xf

a) 12

34)(

x

xxf

b)5

82)(

x

xxf

c) 17

1)(

xxf

d) 5

5)(

x

xxf

e) 2

23

)(x

xxxf

f) 33

1)(

x

xxf

g) 21

)(x

xxf

h) 4

)(2

3

x

xxf

i) 4

)(2

2

x

xxf

6.2 Estudi de funcions racionals (sense segona derivada).

Representa les següents funcions racionals (sense fer servir la segona derivada):

1.

14

)12()(

2

2

x

xxf

2.

x

xxf

3)(

4

3.

1)(

2

x

xxf

4.

22 1

4)(

x

xxf

5.

22 1

12)(

xx

xxf

6.

1

)2()(

2

x

xxf

7.

1

33)(

2

2

x

xxxf

8.

3

23

2

44)(

x

xxxxf

9.

)1()2()(

2

3

xx

xxf

6.3 Estudi de funcions racionals (amb segona derivada).

6.3.1 Quadre resum general.

a) Domini de definició.

Una funció racional està definida sempre que no s’anul·li el denominador.

0)(|)( xQxfDom

b) Indeterminacions 0/0.

Si per a ax tenim que 0)(0)( aQiaP ens trobem en un cas d’indeterminació

del tipus 0

0. Vegeu apartat 1.5.

c) Punt de tall amb l’eix Y.

Avaluem la funció per a )0(0 fx

d) Punts de tall amb l’eix X.

Una funció racional )(

)()(

xQ

xPxf té un zero (punt de tall amb l’eix X) en ax si

0)(0)( aQiaP , és a dir, s’anul·la el numerador però no el denominador en a.

La funció es perfectament contínua en aquest punt.

e) Asímptotes verticals.

Una funció racional )(

)()(

xQ

xPxf té una asímptota vertical en ax si

0)(0)( aQiaP , és a dir, s’anul·la el denominador però no el numerador.

També poden aparèixer asímptotes verticals a algunes de les indeterminacions 0/0.

En particular, la funció no està definida en x = a, i per tant té una discontinuïtat en

aquest punt. Vegeu apartat 1.5.

f) Comportament d’una funció racional en l’infinit.

Podem trobar infinits, asímptotes horitzontals i asímptotes obliqües. Vegeu apartat 1.3.

g) Monotonia: Creixement, decreixement, extrems relatius.

Hem de fer un estudi de la primera derivada.

h) Curvatura: Concavitat, convexitat i punts d’inflexió.

Hem de fer un estudi de la segona derivada.

g) Gràfica.

6.3.2 Asímptotes de les funcions racionals.

)(

)()(

xden

xnumxf

Asímptota

vertical

0)(0)( xnumixden

(després de factoritzar)

Asímptota

horitzontal

)()( xdengrauxnumgrau

Asímptota

obliqua

1)()( xdengrauxnumgrau

6.3.3 Exercici autocorrectiu de derivació de funcions racionals.

Calcula la primera i segona derivada de les següents funcions i resol les equacions

corresponents. Comprova que els resultats obtinguts són correctes.

)(

)(')()()(')('

)(

)()(

2 xq

xqxpxqxpxf

xq

xpxf

)('))(()('))(()( 1 xpxpnxfxpxf nn

)(xf 0)( xf 0)(' xf 0)('' xf

3

1)(

x

xxf

-1 -3/2 -2

1

2)(

2

x

xxf 0 0

3)(

2

x

xxf 0 0, 6

3

2 1)(

x

xxf

-1, 1 3,3 6,6

4)(

2

2

x

xxf 0 0

3

2,

3

2

1

1)(

2

2

x

xxf 1, -1 0

3

1,

3

1

4)(

2

x

xxf 0 0

1)(

2

x

xxf 0 1, -1 3,3,0

3

)3()(

2

x

xxf 3 -9, 3

2

3)2()(

x

xxf

2 2, -4 2

22 )1(

4)(

x

xxf

0 0

22

3

)1(

32)(

x

xxxf

2

3,

2

3,0

2

333,

2

333 3,3,0

6.3.4 Exercicis resolts d’estudi de funcions racionals.

1. Estudi de la funció

6)(

2

3

xx

xxf

a) Punt de tall amb l’eix Y.

b) (Possibles) punts de tall amb l’eix X.

c) (Possibles) asímptotes verticals.

d) Comportament de la funció en l’infinit.

e) Creixement, decreixement, màxims i mínims.

f) Representació gràfica de la funció, amb la informació dels apartats anteriors.

a) (0,0)

b) (0,0)

c) x =-2 i x = 3

d) Asímptota obliqua y = x + 1

e) (indicacions)

5.358x

-3.3589

0

0)6(

)182()('

22

22

x

x

xx

xxxxf

Estudiant el signe de la derivada comprovem que la funció té un mínim en x = 3.3589-

i un màxim en x =5.358 . El punt x = 0 no és un extrem relatiu.

4.38553f(-3.3589)

8.86553f(5.358)

f)

2. Estudi de la funció 4

)(2

x

xxf

a) Domini: }4{

b) Punt de tall amb l’eix Y: (0,0)

c) Punt de tall amb l’eix X: (0,0)

d) Asímptotes verticals : x = -4

e) La gràfica de la funció té una asímptota obliqua: 4 xy

f) Monotonia.

22

2

2

22

2

22

2

4

)8(

4

8

4

82

4

)4(2

4

14

2

x

xx

x

xx

x

xxx

x

xxx

x

x

x

xx

8

00

)4(

)8()('

2 x

x

x

xxxf

8, 4,8 0,4 ,0

creixent

055.0)10('

f

f

decreixent

03)6('

f

f

decreixent

03)2('

f

f

creixent

036.0)2('

f

f

Deduïm que la funció té un màxim relatiu a x = -8 , 16)8( fy

i un mínim relatiu a x = 0 , 0)0( fy

g) Curvatura.

333

22

4

22

4

22

2

2

2

2

4

32

4

162

4

81682

4

84)4(2

4

)4(284)4(2

4

8

)4(24

)4(2828

xxx

xxxx

x

xxxx

x

xxxxx

x

xx

xx

xxxx

solució téno 0)4(

32)('

3

xxf

4, ,4

baix a cap corbada

04)6(''

f

f

dalt a cap corbada

032)3('

f

f

La funció no presenta punts d’inflexió perquè canvia la seva curvatura a l’asímptota

vertical.

h) Gràfica.

3. Estudi de la funció 1

1)(

2

2

x

xxxf

a) Punts de tall amb l’eix X. No en té.

b) Punt de tall amb l’eix Y: 1

c) Asímptotes verticals: No en té

d) Comportament de la funció en l’infinit : La funció tendeix a una asímptota

horitzontal y=1

e) Monotonia:

22

2

)1(

1)('

x

xxf

1

10)('

x

xxf

1, decreixent 1,1 creixent ,1 decreixent

Per tant, la funció té un mínim en P =

2

1,1 i un màxim en P =

2

3,1

f) Curvatura:

32

2

1

)3(2)(''

x

xxxf

732.13

732.13

0

0)(''

x

x

x

xf

3, corbada cap a baix 0,3 corbada cap amunt

3,0 corbada cap a baix ,3 corbada cap amunt

g) Gràfica:

4. Estudi de la funció 1

)(2

3

x

xxf

a) Punt de tall amb l’eix Y: 010

0)0(

2

3

f

b) Punts de tall amb l’eix X: 0001

)( 3

2

3

xxx

xxf

c) Asímptotes verticals: 1,1012 xxx

d) Comportament a l’infinit: xyobliquaAsímptota2

3

DenGrau

NumGrau

e) Monotonia:

73.13,73.13,0

01

3

1

2)1(3

1

)2()1(3)('

22

22

22

222

22

322

xxx

x

xx

x

xxx

x

xxxxxf

Estudi per intervals:

09/4)2(' f f creixent

73.13 x màxim relatiu

008.1)5.1(' f f decreixent

1x asímptota vertical

022.1)5.0(' f f decreixent

0x No n’hi ha extrem relatiu

022.1)5.0(' f f decreixent

1x asímptota vertical

008.1)5.1(' f f decreixent

73.13 x mínim relatiu

09/4)2(' f f creixent

La funció té un màxim relatiu a:

60.22/33)3(73.13 fyx

i un mínim relatiu a:

60.22/33)3(73.13 fyx

f) Gràfica de la funció:

6.3.5 Estudi de funcions racionals.

Estudi complet de les següents funcions:

a) Punts de tall amb l’eix X.

b) Punt de tall amb l’eix Y.

c) Asímptotes verticals.

d) Comportament de la funció en l’infinit.

e) Monotonia: Creixement, decreixement, màxims i mínims.

f) Curvatura: Concavitat, convexitat i punts d’inflexió.

g) Representació gràfica de la funció.

1. 1

)2()(

2

x

xxf

2. 3

107)(

2

x

xxxf

3. 6

1)(

2

xxxf

4. 32

2)(

2

xx

xxf

Indicació per a la curvatura: l’equació 0836 23 xxx només té una

solució real 634.6x

5. 2

4)(

34

x

xxxf

Indicació per a la curvatura: l’equació 0164 23 xx només té una solució

real 679.1x

6. 2

3

)1()(

x

xxf

7. 3

)(2

2

x

xxf

8. 30

1055)(

2

2

xx

xxxf

6.4 Problemes PAU TEC amb funcions racionals i derivació.

1. La funció 12522

40)(

2

xxxf mostra aproximadament la venda diària, en milers

d’unitats, d’un perfum de moda en funció de x, en què x és el dia del mes de febrer.

a) Quantes unitats de perfum es van vendre, aproximadament, el dia 5 de febrer? Quin

és l’increment de vendes entre el dia 7 i el dia 9 de febrer?

b) Quin dia del mes de febrer es van vendre més perfums i quantes unitats se’n van

vendre?

PAU CAT CCSS JUNY 2019 1.4 (Solució: "Compendium CCSS", Pàg. 427)

2. Es preveu un canvi important en la població d’una determinada zona per qüestions

mediambientals. El nombre d’habitants de la zona, en milions, vindrà donat per la

funció 2

2

)2(

28)(

t

ttP , en què t mesura el temps en anys des del moment actual (t = 0).

a) Digueu quin és el nombre d’habitants de la zona actualment i quin serà aquest

nombre a molt llarg termini.

b) En quin moment s’arribarà al nombre mínim d’habitants? Quants habitants hi

haurà en aquell moment? Quin és el nombre màxim d’habitants que s’assoleix en

aquesta zona?

PAU CAT CCSS SET 2019 5.3 (Solució: "Compendium CCSS", Pàg. 435)

3. Considereu la funció ax

xxf

2

)( , en què a és un paràmetre real.

a) Trobeu per a quins valors del paràmetre a la recta tangent a la funció f en x = 1 és

paral·lela a la recta y + 3x + 5 = 0.

b) Per al valor del paràmetre a = 1, trobeu els intervals de creixement i decreixement i

els punts on s’assoleixen els màxims i mínims relatius de la funció f.

Solució PAU CAT TEC SET 2018 1.2

4. Sigui la funció kx

xf

2

1)( , en què k és un paràmetre real diferent de 0. Per als

diferents valors del paràmetre k:

a) Calculeu el domini i les asímptotes de la funció.

b) Calculeu els punts amb un màxim o un mínim relatiu. Solució PAU CAT TEC JUNY 2017 1.3

5. Una fàbrica produeix diàriament x tones d’un producte A i (40 – 5x)/(10 – x) tones

d’un producte B. La quantitat màxima de producte A que es pot produir és 8 tones. El

preu de venda del producte A és 100€ per tona i el del producte B és 250€ per tona.

a) Construïu la funció de la variable x que ens proporciona els ingressos diaris, suposant

que es ven tota la producció.

b) Calculeu quantes tones de cada producte s’han de produir diàriament per a obtenir el

màxim d’ingressos, i comproveu que és realment un màxim relatiu.

Solució PAU CAT TEC SET 2012 4.4

6. Sigui bx

axxf

2

)( , en què a≠0.

a) Determineu si té alguna asímptota vertical, en funció del paràmetre b.

b) Indiqueu el valor dels paràmetres a i b perquè la funció f (x) tingui la recta

y = 2x − 4 com a asímptota obliqua a +∞. Solució PAU CAT TEC SET 2012 4.2

7. Trobeu les asímptotes de la funció

54

253)(

2

3

xx

xxxf

Solució PAU CAT TEC SET 2010 2.1

8. Determineu el valor dels paràmetres a, b i c perquè la gràfica de la funció

cbxx

axf

2)( sigui la següent:

Solució PAU CAT TEC JUNY 2010 5.3

9. Considereu la funció de variable real 1

2)(

2

3

x

xxf

a) Trobeu-ne el domini.

b) Calculeu l’equació de les seves asímptotes, si en té.

c) Estudieu-ne els intervals de creixement i decreixement, Així com les abscisses dels

seus extrems relatius, si en té, i classifiqueu-los. Solució PAU CAT TEC SET 2009 1.5

10. Trobeu el domini i les asímptotes de la funció definida per 1

14)(

2

x

xxxf

Solució PAU CAT TEC JUNY 2006 1.4

11. Considereu la funció 2

61)(

xx

axf on a és un paràmetre.

a) Calculeu el valor del paràmetre a sabent que f(x) té un extrem relatiu en el punt

d’abscissa x = 3.

b) Aquest extrem relatiu, es tracta d’un màxim o d’un mínim? Raoneu la resposta. Solució PAU CAT TEC JUNY 2004 3.3

12. a) Determineu el valor del paràmetre a que fa que la funció

3)(

x

axxf

Presenti un extrem relatiu en el punt d’abscissa x = 3.

b) Per a aquest valor del paràmetre a, calculeu els intervals de creixement i

decreixement, i les asímptotes de la funció.

c) A partir de les dades que heu obtingut, feu una gràfica aproximada d’aquesta funció.

Solució PAU CAT TEC JUNY 2003 5.5

13. Se sap que la derivada d'una funció f(x) és:

1

6)(

2

x

xxxf

Calculeu les abscisses dels punts on la funció f(x) té els seus extrems relatius,

especificant per a cada un dels valors que obtingueu si es tracta d'un màxim o d'un

mínim relatiu. Solució PAU CAT TEC SET 2002 1.2

14. Determineu el valor que ha de tenir k perquè la funció

2

532)(

2

x

xkxxf

tingui límit quan x tendeix a 2 (és a dir, existeixi )(lim2

xfx

) i calculeu el valor que tindrà

aquest límit. Solució PAU CAT TEC JUNY 2002 3.2

15. Sigui 1

2)(

x

xmxf , on m és un paràmetre.

a) Determineu per a cada valor del paràmetre m el valor del límit )(lim1

xfx

(si existeix).

b) Per a quins valors de m la derivada f'(x) de la funció f(x) és positiva per a tot x?

Solució PAU CAT TEC JUNY 2002 2.3

16. Considereu la funció 12

2)(

2

2

x

xxxf

a) Determineu les seves asímptotes.

b) Calculeu els intervals on creix i on decreix, i els extrems relatius.

c) D'acord amb els resultats que heu obtingut, dibuixeu aproximadament la seva gràfica.

d) Fixant-vos en la gràfica anterior, expliqueu quina seria la gràfica de la funció

3)()( xfxg (feu-ne un esquema). En quins punts té màxims la funció g(x)?

Solució PAU CAT TEC 2001 2.6

17. Considereu la funció ax

xxf

2

)( , on a és un paràmetre.

a) Calculeu a sabent que la recta y = x+2 és una asímptota obliqua d'aquesta funció.

b) Prenent el valor de a obtingut en l'altre apartat, calculeu el domini, les interseccions

de la gràfica amb els eixos, els intervals de creixement I decreixement i els extrems

relatius de la funció f. Feu una gràfica aproximada d'aquesta funció a partir de les dades

que heu obtingut.

PAU CAT TEC JUNY 2000 3.5 (Problema)

18. Considereu la funció 28

1)(

xxxf

a) Trobeu el domini de f(x) i les asímptotes.

b) Determineu el signe de la funció en el seu domini (determinar el signe de f(x) vol dir

establir per a quins valors de x es compleix f(x) ≥0 i per a quins f(x) ≤0).

c) Trobeu-ne els intervals de creixement i decreixement i els extrems relatius.

d) Feu un esquema de la gràfica de la funció.

Solució PAU CAT TEC JUNY 1999 2.5

19. Considereu la funció 1

)(2

x

xxxfy

a) Feu un estudi de les seves asímptotes.

b) Calculeu els punts en què aquesta funció té extrem relatiu i digueu per a quins

intervals del domini la funció és creixent.

c) Feu un esbós de la gràfica de la funció a partir de les dades obtingudes en els apartats

anteriors.

Solució PAU CAT TEC JUNY 1999 6.5

6.5 Problemes PAU CCSS amb funcions racionals i derivació.

1. La despesa mensual en tabac d’un fumador ve determinada pel seu salari mitjançant

la funció 4

400)(

2

x

xxf , en què x representa el salari en milers d’euros i f(x) la despesa

mensual en tabac en euros.

a) Determineu el salari per al qual la despesa en tabac és màxima. A quant ascendeix

aquesta despesa?

b) Per a quins salaris la despesa mensual és inferior a 60 €?

Solució PAU CAT CCSS SET 2018 3.4

2. El nombre d’individus, en milions, d’una població ve determinat per la funció

2

2

)1(

5)(

t

ttP , en què t mesura el nombre d’anys transcorreguts.

a) Quina és la població inicial i la població després de 9 anys? A partir de quin moment

la població serà inferior a un milió d’individus?

b) Amb el pas dels anys, cap a quin valor tendirà el nombre d’individus de la població?

Solució PAU CAT TEC SET 2018 1.6

3. Sigui la funció 4

3)(

2

2

x

xxxf .

a) Indiqueu-ne justificadament el domini i determineu els punts en què la gràfica de f

talla l’eix de les abscisses.

b) Estudieu-ne el creixement i feu un esbós aproximat de la gràfica de la funció.

Solució PAU CAT CCSS JUNY 2018 5.1

4. Considereu la funció 1

)(2

x

xxf .

a) Determineu-ne, si en té, les asímptotes horitzontals i verticals.

b) Justifiqueu que és decreixent en tot el domini de f.

Solució PAU CAT CCSS SET 2014 5.6

5. La demanda d’energia elèctrica d’una ciutat, comptada a partir de la mitjanit i fins a

les vuit del matí, és donada per la funció 6

126)(

2

tttf , on t s’expressa en hores (h)

i f(t), en milions de kilowatts hora (kW h).

a) A quina hora el consum coincideix amb el de la mitjanit, i quin és aquest consum?

b) A quina hora es donarà el mínim consum? Justifiqueu que, efectivament, es tracta

d’un mínim.

Solució PAU CAT CCSS SET 2013 1.6

6. Sobre la funció cbxx

axf

2)( disposem de les dades següents:

- les seves asímptotes verticals són x=-3 i x=1;

- la seva gràfica passa pel punt (0, -4).

a) Determineu la fórmula de la funció i feu un dibuix aproximat de la gràfica

corresponent.

b) En el cas a=1, b=-2 i c=-1, determineu i classifiqueu, si existeixen, els extrems

relatius de la funció.

Solució PAU CAT CCSS JUNY 2012 3.1

7. Considereu la funció següent 1

2)(

2

ax

xxf :

a) Determineu el valor de a que fa que la funció f tingui un extrem en el punt x=1, i

indiqueu si es tracta d’un màxim o d’un mínim.

b) Per a a=3, indiqueu les asímptotes horitzontals i verticals de la funció f.

Solució PAU CAT CCSS JUNY 2011 1.3

8. Considereu la funció següent:

2

13)(

x

xxf

a) Determineu-ne les asímptotes horitzontals i verticals, si n’hi ha.

b) Si f′(x) > 0 en tot el domini de la funció f, calculeu els límits laterals quan x tendeix a

–2 i feu un esbós de la gràfica de la funció f.

Solució PAU CAT CCSS JUNY 2010 1.1

9. En una empresa artesana que pot produir fins a 25 cadires setmanals, la funció de

costos en relació amb el nombre q de cadires produïdes és

204100

)(3

qq

qC

Si q és el nombre de cadires produïdes, el cost mitjà de cada cadira s’expressa

mitjançant la funció q

qCqQ

)()(

a) Calculeu el cost mitjà de cada cadira, si l’empresa produeix 5 cadires. I si en produeix

20?

b) Determineu quantes cadires cal produir perquè el cost mitjà sigui mínim, justifiqueu

que es tracta efectivament d’un mínim i calculeu aquest cost mitjà.

Solució PAU CAT CCSS JUNY 2010 1.4

10. La taxa d’inflació interanual d’un país determinat durant l’any 2008 expressada en

punts percentuals, i (t), es pot aproximar mitjançant la funció

121,340

910)(

2

ttt

ti

en què t és el temps en mesos des del començament de l’any i t=1 és el mes de gener.

a) Trobeu en quins mesos la taxa d’inflació interanual és de 3 punts percentuals.

b) Trobeu en quins mesos la taxa d’inflació és decreixent i en quins mesos és creixent.

c) Trobeu en quin mes la taxa assoleix el valor mínim i calculeu aquest valor.

d) Feu un esbós de la gràfica d’aquesta funció.

e) Trobeu en quin mes la taxa assoleix el valor màxim i calculeu aquest valor.

Solució PAU CAT CCSS SET 2009 1.6

11. Segons un estudi sobre l’evolució de la població d’una espècie protegida

determinada, podem establir el nombre d’individus d’aquesta espècie durant els propers

anys mitjançant la funció

1

50050)(

t

ttf

en què t és el nombre d’anys transcorreguts.

a) Calculeu la població actual i la prevista per d’aquí a nou anys.

b) Determineu els períodes en què la població augmentarà i els períodes en què

disminuirà.

c) Esbrineu si, segons aquesta previsió, la població tendirà a estabilitzar-se en algun

valor i, si escau, determineu-lo.

Solució PAU CAT CCSS JUNY 2009 4.3

12. L’evolució de la població d’un estat, en milions d’habitants, es pot aproximar

mitjançant la funció

404

20)(

2

t

ttP

en què t és el temps en anys.

a) Calculeu la població actual (per a t = 0).

b) Determineu el límit de P(t) quan t tendeix a infinit.

c) Determineu al cap de quants anys la població serà màxima i el nombre d’habitants

que la funció prediu per a aquest màxim.

Solució PAU CAT CCSS JUNY 2009 3.3

13. Considereu la funció real de variable real 4

5)(

2

x

xxxf .

a) Determineu-ne els intervals de creixement i decreixement.

b) Trobeu-ne els extrems relatius.

Solució PAU CAT CCSS SET 2008 4.1

14. Els beneficis mensuals d’un artesà expressats en euros, quan fabrica i ven x

objectes, s’ajusten a la funció B(x) = –0,5x2+50x– 800, en què 6020 x .

a) Trobeu el benefici que obté en fabricar i vendre 20 objectes i en fabricar i vendre 60

objectes.

b) Trobeu el nombre d’objectes que ha de fabricar i vendre per a obtenir el benefici

màxim, així com aquest benefici màxim.

c) Feu un esbós del gràfic de la funció B(x).

d) El benefici mitjà per x objectes és x

xBxM

)()( . Digueu quants objectes ha de

fabricar i vendre perquè el benefici mitjà sigui màxim, i quin és aquest benefici.

Solució PAU CAT CCSS JUNY 2007 2.5

15. En els sis primers mesos, des que va obrir, una llibreria ha anat anotant el nombre de

compradors de cada mes. Aquest nombre N(x) es pot ajustar per la funció

x

xxN

6001000)(

,

essent x el número del mes comptat des que van obrir.

a) Quants compradors van tenir el segon mes? En quin mes, comptat a partir de

l’obertura, van tenir 900 compradors?

b) Suposem que aquesta fórmula serveix per predir el nombre de compradors en el

futur. Podem assegurar que aquest nombre sempre anirà en augment?

Solució PAU CAT CCSS SET 2006 4.1

16. La funció 5

10090)(

x

xxf indica el nombre de minuts que s’aconsella de caminar

diàriament en funció del nombre x de setmanes que han passat des que es va començar

un programa de manteniment.

a) Segons aquest programa de manteniment, a partir de quina setmana s’ha de caminar

més d’una hora?

b) Feu un gràfic aproximat de la funció i expliqueu el seu creixement. Quant de temps

aproximadament hauria de dedicar diàriament a caminar una persona que fa molt de

temps que segueix el programa?

Solució PAU CAT CCSS JUNY 2005 4.4

17. Sigui la funció 1

1)(

2

2

x

xxf .

a) Trobeu les equacions de les asímptotes de f(x).

b) Estudieu el signe de la funció.

c) Estudieu el creixement i decreixement de la funció i indiqueu quins són els seus

màxims i mínims.

d) Feu un esbós de la gràfica de f(x).

Solució PAU CAT CCSS SET 2004 5.5

18. Considereu la funció x

xxf

1

3)(

a) Determineu les seves asímptotes verticals i horitzontals (si en té) i els intervals de

creixement i decreixement. Feu després un esquema senzill de la seva gràfica.

b) Determineu els punts de la gràfica de la funció on la tangent és paral·lela a la recta

d'equació y = x.

Solució PAU CAT CCSS SET 2000 6.5

19. Representeu gràficament la funció

45

1)(

2

xxxf

de manera raonada (domini de definició, asímptotes, intervals de creixement i

decreixement, màxims i mínims...).

PAU CAT CCSS SET 1999 5.5

20. Considereu la funció

1

44)(

2

2

x

xxxf

Trobeu el domini de definició, els punts de tall amb els eixos, les possibles asímptotes,

els intervals de creixement i decreixement, així com els possibles màxims i mínims. Feu

després un esquema senzill de la gràfica d'aquesta funció.

Solució PAU CAT CCSS JUNY 1999 1.6

21. Digueu raonadament quin és l'interval de creixement de la funció

12

x

xy

Solució PAU CAT CCSS JUNY 1999 6.2

22. Considereu la funció

18)(

2

2

x

xxxf

Busqueu-ne el domini de definició, els límits quan x i quan x , les

asímptotes, els punts de tall amb els eixos, els intervals de creixement i decreixement, i

els màxims i mínims locals. Feu després un dibuix aproximat de la seva gràfica.

PAU CAT CCSS SET 1998 5.5

7 Tècniques de derivació.

7.1 Taula de derivades de les funcions fonamentals.

1. cxf )( 0)(' xf

2. xxf )( 1)(' xf

3. nxxf )( 1)(' nxnxf

4. xxf )( x

xf2

1)('

5. 0,)( aaxf x )ln()(' aaxf x

6. xexf )( xexf )('

7. )(log)( xxf a xa

ex

xf a

1

)ln(

1log

1)('

8. )ln()( xxf x

xf1

)('

9. )sin()( xxf )cos()(' xxf

10. )cos()( xxf )sin()(' xxf

11. )tan()( xxf )(tan1)(' 2 xxf

12. )arcsin()( xxf 21

1)('

xxf

13. )arccos()( xxf 21

1)('

xxf

14. )arctan()( xxf 21

1)('

xxf

7.2 Àlgebra de derivades.

Si )(xf i )(xg són derivables en x, llavors també ho és també la suma, resta, producte i

divisió (si el denominador no s’anul·la) de ambdues funcions, i es verifica:

a) )(')('))(( xgxfxgf

b) )(')('))(( xgxfxgf

c) )(')()()('))(( xgxfxgxfxgf

d) 0)()(

)(')()()(')(

2

xgsi

xg

xgxfxgxfx

g

f

De l'apartat (c) deduïm que )('))(( xfkxfk . Efectivament:

)(')(')(0)(')(')()'( xfkxfkxfxfkxfkxfk

7.3 Regla de la cadena.

Sigui g una funció derivable en ax i sigui f una funció derivable en )(agbx .

Llavors la funció composta ))(())(( xgfxgf és derivable en ax i es compleix

)('))((')(' agagfagf

En general direm que la derivada de la funció composta és )('))((')(' xgxgfxgf

7.4 Derivada de la funció inversa.

Siguin A i B són dos intervals oberts de IR, i sigui BAf : una funció bijectiva amb

inversa contínua.

Si es compleix 0)(' xf per a tot Ax , llavors 1f és derivable per a tot By .

A més a més, anomenant )(xfy , es verifica:

)('

1)()'( 1

xfyf

7.4 Pràctica amb derivades.

Derivació d’una funció amb divisió i regla de la cadena.

Determina la derivada de la funció 2

2

1

12)(

xx

xxf

22

2

12

12

11

x

xx

xx

Derivem el denominador:

2

2

2

2

2

2

2

2

2

22

2

22

2

22

1

21

11

1

11

11

11

1111

x

x

x

x

x

x

x

x

x

xx

x

xx

x

xxxxx

I ara derivem la funció completa:

*

1

1

211214

1

122

2

2

222

2

2

xx

x

xxxxx

xx

x

2

2

2

422

2

2222

2

22

2

222

2

2222

1

14

1

)14()1(4

1

)21)(12()1(4

1

2112

1

114

1

211214

x

x

x

xxx

x

xxxx

x

xx

x

xxx

x

xxxx

23

22

2

222

2

22

2

2

)1(

14

1)1(

14

)1(

1

14

*xx

x

xxx

x

xx

x

x

7.4.1 Exercicis de derivació.

1. 63)( 24 xxxf

2. 236)( xxxf

3.

5

1)(

23

xxxf

4. xxxxf 246)( 2

5

2

7

5.

xxxxf

13)( 3

6. 23

3)1()(

x

xxf

7. 52)(3 2 xxxf

8. 23 2141)( xxxf

9. )23)(12()( xxxxf

10. 3612)( 2 xxxxf

11. 2

3

1)(

t

ttf

12.

3

)4()(

2

s

ssf

13.

2

1)(

2

3

xx

xxf

14. 22 32)( xxf

15.

x

xxf

1

1)(

16. 2

2

1

12)(

xx

xxf

17. 3 2 1)( xxxf

18. 331)( xxf

19. xxf 2sin)(

20. xxxf 3cossin2)(

21.

x

xxf

cos1

sin)(

22. xxxf 3cos2sin)(

23. ttttf cossin)(

24. tttf cossin)( 3

25. xxf cosln)(

26. xxf tanln)(

27. xxf 2sinln)(

28.

x

xxf

sec

1tan)(

29.

x

xxf

sin1

sin1ln)(

30. )sin(ln)( xxf

31. )sin(cos)( xxf

32.

x

xxf

1

1ln)(

33. xxxf sinlog)( 2

3

34. 2

2

1

1ln)(

x

xxf

35. )ln()( 2 xxxf

36. )52ln()( 3 xxxf

37. xxxf ln)(

38. xxf 3ln)(

39. 21ln)( xxxf

40. )ln(ln)( xxf

41. 54)( xexf

42. 2

)( xaxf

43. xxxf 22

7)(

44. )1()( 2xexf x

45.

1

1)(

x

x

e

exf

46. xexf sin)(

47. xexf x sin)( cos

48. )ln(sin)( xexf x

49. xxxf )(

50. xxexf )(

Font: http://www.uv.es/~montes/biologia/matcero.pdf

7.5 Derivació de polinomis i funcions racionals amb la Regla de la

Cadena.

Davant d’un polinomi, la temptació de desfactoritzar-lo és forta, però no és el

bon camí. Ans al contrari, hem d’intentar sempre factoritzar els polinomis,

principalment traient factor comú.

54 )3()1()( xxxf

Apliquem la fórmula de la derivada del producte i la regla de la cadena:

4453

445

334

)3(5)1()3()1(4)(')3(5)1()3(5)3(

)1(4)1()1(4)1(

xxxxxf

xxx

xxx

i traiem factor comú:

79)3()1(55124)3()1(

)1(5)3(4)3()1()('

2323

43

xxxxxxx

xxxxxf

Només després de treure factor comú podem resoldre l’equació 0)(' xf :

9

7079

30)3(

10)1(

079)3()1(0)(' 2

3

23

xx

xx

xx

xxxxf

Les solucions són 1x , 3x i 9

7x .

1

1)(

2

2

x

xxf

Apliquem la fórmula de la derivada de la divisió de funcions:

22

22

2

2

)1(

2)1()1(2)('

21

21

x

xxxxxf

xx

xx

Ara no caiem en la temptació de desfactoritzar el numerador, i en comptes de fer les

multiplicacions traiem factor comú:

222222

22

22

22

)1(

4

)1(

22

)1(

112

)1(

2)1()1(2)('

x

x

x

x

x

xxx

x

xxxxxf

0040)1(

40)('

22

xx

x

xxf

Per calcular la segona derivada, en comptes de desfactoritzar el denominador apliquem

la Regla de la Cadena:

42

222

2222 )1(

)1(4)4()1(4)(''

142121

44

x

xxxxxf

xxxxx

x

I ara, en comptes de treure parèntesis al numerador, en traiem factor comú:

)13)(1(441)1(4

)4()1()1(4)1(4)4()1(4

22222

222222

xxxxx

xxxxxxx

Perquè d’aquesta manera podem simplificar numerador i denominador tatxant el factor

repetit:

32

2

42

22

)1(

)13(4

)1(

)13)(1(4)(''

x

x

x

xxxf

I, ara sí, podem resoldre còmodament l’equació 0)('' xf :

0130)13(40)1(

)13(40)('' 22

32

2

xx

x

xxf , que no té solució.

7.6 Derivació de funcions radicals.

La fórmula de la derivada de l’arrel quadrada es pot deduir escrivint l’arrel quadrada en

forma potencial:

xxxxxxf

xxfxxf

2

1)2/1()2/1()2/1()2/1()('

)()(

2/1

2/112/1

2/1

En general, per derivar una arrel d’índex superior a 2: n xxf )(

escriurem la funció en forma potencial i aplicarem la fórmula de la derivada de la

potència: 1/1/1 )/1()(')( nnn xnxfxxxf

Exemples:

1.

3 23 23/2

3/213/1

3/13

3

1)3/1()3/1()3/1()3/1()('

)()(

xxxxxxf

xxfxxf

2.

333/1

3/113/2

3/23 2

3

2)3/2()3/2()3/2()3/2()('

)()(

xxxxxxf

xxfxxf

Què has de saber:

Les regles d’escriptura de les potències:

m nmn

n

n

xx

xx

/

1

i la fórmula de la derivada de la potència: 1)(')( nn xnxfxxf

Exercici: Determineu els màxims i mínims relatius de la funció 33 1)( xxxf

7.7 Exercicis de continuïtat i derivació de funcions definides a trossos i regla de la

cadena.

1.Estudieu la continuïtat i derivabilitat de la funció

1

1)2()(

2

3

xsix

xsixxf

2. a) Estudieu la continuïtat i derivabilitat de la funció

2)2(

22)(

3

xsixx

xsixxf

b) Determineu l’equació de la recta tangent a la gràfica de f al punt )1,3(

3. Determineu el valor de k per al què sigui contínua la següent funció. És derivable?

3104

33)(

2 xsixx

xsikxxf

4. Determineu el valor de k per al què sigui contínua la següent funció. És derivable?

01

0)(

xsixe

xsikxf

x

8 Recta tangent.

8.1 Concepte de recta tangent.

8.1.1 Funcions tangents en un punt.

Direm que les gràfiques de dues funcions )(xf i )(xg són tangent en un punt px si

a) )()( pgpf

b) )(')(' pgpf

és a dir, si passen pel mateix punt i a més a més tenen el mateix pendent en aquest punt.

8.1.2 Recta tangent a la gràfica d’una funció.

És un cas particular de l’anterior, quan )(xg és una recta : baxxg )(

En aquest cas la funció queda totalment determinada per les condicions a) i b)

Per la condició b): )(')(')(' pfpgaaxg

Per la condició a): papfbpfbpapg )()()(

Per tant, per cada punt de la gràfica x = p on f és derivable existeix una única recta

tangent bxay on

(1) )(' pfa (Propietat fonamental de la recta tangent)

(2) papfb )(

8.1.3 Pendent d’una recta.

Donada qualsevol recta en el pla de la forma baxy , el valor a s’anomena pendent

de la recta.

Per trigonometria sabem que el pendent d’una recta és la tangent de l’angle que forma

aquesta recta amb l’eix d’abscisses.

)tan(a

El valor b es pot deduir si sabem que la recta passa per un punt ),( 0 oyx , ja que en

aquest cas

0axyb o

8.1.4 Segona definició. Recta tangent a la funció en un punt.

Definim la recta tangent a la funció f en el punt d’abscissa x com aquella recta que

passa per ))(,( xfxP i té com a pendent el valor de la derivada de la funció en aquest

punt:

)(' xfa

el valor b es pot determinar fàcilment amb la fórmula axxfb )(

8.2 Determinació de la recta tangent.

8.2.1 Exemples.

Troba la recta tangent a la funció 3

)(2x

xf en el punt x = 1

Tenim (fent p = 1)

3

1)1(,

3)(

2

fx

xf

3

2)1(',

3

2)(' fa

xxf

3

1

3

21

3

1)1('1)1(

ffb

La recta tangent és doncs 3

1

3

2 xy

Determina la recta tangent a la funció xxf ln)( en el punt x = 2

xxx

xfxxf2

1

2

11)('ln)(

La recta tangent és de la forma y = ax + b, on

25.04

1)2(' fa

153426.0)2('2)2( ffb

La recta tangent és doncs 153426.025.0 xy

8.2.2 Exercicis.

1. Donada la funció ,13)( 2 xxxf troba l’equació de la recta tangent en els punts

d’abscissa x = -1, x = -3, i x = 1

2. Calcula l’equació de la recta tangent a la gràfica de 1

)(2

2

x

xxf en el punt

d’abscissa x = 1 i en el punt d’abscissa x = -0.5

3. Esbrina l’equació de la recta tangent a la corba d’equació xxxf ln)( en el punt

d’abscissa x=1, x=0.5 i x=0.25

4. Determineu el polinomi cxbxaxxP 23)( de forma que es verifiquin les

condicions següents:

a) El polinomi té extrems relatius als punts 3

1x , 1x .

b) La recta tangent a la gràfica de )(xP al punt )0(,0 P és 3 xy

8.2.3 Problemes PAU de recta tangent amb funcions polinòmiques.

1. Sigui f(x) = x3 + ax

2 + bx + c. Sabem que la gràfica d’aquesta funció és tangent a la

recta r: y = x + 3 en el punt d’abscissa x = –1, i que en el punt d’abscissa x = 1 la recta

tangent és paral·lela a la recta r.

Calculeu el valor dels paràmetres a, b i c. Solució PAU CAT TEC JUNY 2013 3.6

2. Donades la recta y = ax + 1 i la paràbola y = 3x – x

2,

a) Calculeu els valors del paràmetre a perquè siguin tangents.

b) Calculeu els punts de tangència. Solució PAU CAT TEC SET 2012 4.6

3. Considereu la funció bxaxxf 2)( ba, . Trobeu els valors de a i b que fan

que la recta 12 xy sigui tangent a la gràfica de f quan x = 1.

Solució PAU CAT TEC SET 2008 4.1

4. Calculeu els valors del paràmetre a, 0a , que fan que les tangents a la corba

d’equació 15122 34 axaxaxy en els punts d’inflexió siguin perpendiculars.

Solució PAU CAT TEC JUNY 2007 2.3

5. Considereu la funció 7)( 234 cxbxaxxxf

a) Calculeu c sabent que la seva recta tangent en el punt d’abscissa x = 0 és horitzontal.

b) Per al valor de c trobat a l’apartat anterior, calculeu a i b sabent que aquesta funció té

un extrem relatiu en el punt d’abscissa x = -2 i que talla l’eix OX quan x = 1.

c) Per als valors obtinguts als altres apartats, calculeu els intervals on la funció creix i

decreix, els seus extrems relatius i feu una representació gràfica aproximada.

Solució PAU CAT TEC 2006 1.6

6. Sigui la paràbola y= 2x2 +x +1 i sigui A el punt de la paràbola d’abscissa 0.

a) Trobeu l’equació de la recta tangent a la paràbola en el punt A.

b) En quin punt de la paràbola la recta tangent és perpendicular a la recta que heu trobat

en l’apartat anterior?

Solució PAU CAT TEC JUNY 2005 4.4

7. Considereu la funció f(x) = x3 –3x

2 +2x +2.

a) Calculeu l’equació de la recta tangent a la gràfica de f(x) en el punt d’abscissa x=3.

b) Existeix alguna altra recta tangent a la gràfica de f(x) que sigui paral·lela a la que heu

trobat? Raoneu la resposta i, en cas afirmatiu, trobeu-ne l’equació.

Solució PAU CAT TEC JUNY 2004 3.1

8. Calculeu les equacions de les dues rectes del pla que passen pel punt P = (1, –1) i que

són tangents a la corba d’equació y = (x– 1)2.

Solució PAU CAT TEC JUNY 2003 2.1

9. Calculeu els valors de a tals que les tangents a la gràfica de la funció

3 + 2x+ ax = f(x) 23 en els punts d'abscisses x = 1 i x = –1 siguin perpendiculars entre si.

Solució PAU CAT TEC JUNY 2000 1.1

10. Sigui bxaxxxf 53)( 23 . Trobeu els valors de a i b de manera que la gràfica

de f(x) tingui la tangent horitzontal per a x=1 i, a més, la corba passi pel punt (-1,-8) .

Solució PAU CAT TEC JUNY 1998 6.2

8.2.4 Problemes PAU TEC.

1. Sabem que una funció f(x) està definida per a tots els nombres reals i que és derivable

dues vegades. Sabem també que té un punt d’inflexió en el punt d’abscissa x = 2, que

l’equació de la recta tangent a la gràfica de la funció f(x) en aquest punt és

249+–124x=y i que -4f(-3) .

a) Calculeu f ''(2), f '(2) i f (2).

b) Calculeu 2

3)(' dxxf .

Solució PAU CAT TEC JUNY 2018 5.4

2. Sigui la funció 1)( xexxf .

a) Calculeu l’equació de la recta tangent a la gràfica de la funció f en el punt d’abscissa

x = 1.

b) Determineu en quins intervals la funció f és creixent i en quins intervals és

decreixent. Solució PAU CAT TEC JUNY 2016 3.3

3. Siguin les funcions 4

)(be

xfax

i 43)( xxg .

a) Determineu el domini i el recorregut de la funció g.

Calculeu per a quins valors de a i de b les gràfiques de les dues funcions són tangents

(és a dir, tenen la mateixa recta tangent) en el punt d’abscissa x = 0.

Solució PAU CAT TEC SET 2014 5.2

4. La funció f(x) és derivable i passa per l’origen de coordenades. La gràfica de la

funció derivada és la que veieu aquí dibuixada, essent f’(x) creixent als intervals

(–∞, –3] i [2, + ∞).

a) Trobeu l’equació de la recta tangent a la gràfica de la funció f(x) en el punt d’abscissa

x = 0.

b) Indiqueu les abscisses dels extrems relatius de la funció f(x) i classifiqueu aquests

extrems.

Solució PAU CAT TEC JUNY 2013 4.6

5. Considereu la funció definida per 1

1)(

2

x

xxf . Calculeu quant val el pendent de la

recta tangent a la seva gràfica pel punt d’abscissa x = 0. Trobeu si hi ha altres punts en

els quals el pendent de la tangent sigui igual al que s’ha obtingut.

Solució PAU CAT TEC JUNY 2006 3.1

6. Donada la funció xe

xxf

1)(

, determineu l'equació de la recta tangent a la seva

gràfica en el punt on s'anul·la la segona derivada.

Solució PAU CAT TEC SET 2000 6.2

8.2.5 Problemes PAU CCSS.

1. La gràfica de la funció x

baxxf8

)( passa pel punt (–2, –6) i la recta tangent en

aquest punt és paral·lela a l’eix de les abscisses.

a) Calculeu el valor de a.

b) Calculeu el valor de b.

PAU CAT CCSS JUNY 2019 1.3 (Solució: "Compendium CCSS", Pàg. 427)

2. Considereu la funció 21

1)(

xxf

.

a) Calculeu l’equació de la recta tangent a la gràfica en aquells punts en què la recta

tangent és horitzontal.

b) Calculeu les coordenades del punt de la gràfica de la funció f(x) en què el pendent de

la recta tangent és màxim.

PAU CAT TEC SET 2019 5.4 (Solució: "Compendium Tec", Pàg. 546)

3. A continuació es mostra la gràfica d’una funció f que presenta un mínim relatiu en el

punt d’abscissa x = −1 i un màxim relatiu en el punt d’abscissa x = 1.

a) Sabent que f '(0) = 1, determineu l’equació de la recta tangent a f que passa per

l’origen de coordenades.

b) Feu un esbós de la gràfica de la funció f' amb les dades de què disposeu.

Solució PAU CAT CCSS JUNY 2018 5.6

4. Considerem una funció f(x) tal que la seva primera derivada és f ′(x) = x2 + bx – 3, en

què b és un paràmetre real.

a) Determineu el valor de b perquè f(x) tingui un extrem relatiu en x = –3 i raoneu si es

tracta d’un màxim o d’un mínim.

b) Per a b = –8, trobeu l’equació de la recta tangent a f(x) en el punt (0, 2).

Solució PAU CAT CCSS SET 2017 2.3

5. Considereu la funció 21

1)(

xxf

.

a) Estudieu-ne el creixement i, si en té, determineu-ne i classifiqueu-ne els extrems

relatius.

b) Calculeu l’equació de la recta tangent a la gràfica de f en el punt d’abscissa x = 1.

Solució PAU CAT CCSS JUNY 2016 3.2

6. La gràfica de la derivada f′ de la funció f és una paràbola que talla l’eix d’abscisses

en els punts (5,0) i (1,0), i té el vèrtex en el punt (3, –4).

a) Expliqueu raonadament en quins intervals la funció f és creixent i en quins intervals

és decreixent. Indiqueu-ne els extrems relatius i classifiqueu-los.

b) Sabem que f(3) = 2. Determineu l’equació de la recta tangent a la funció f en el punt

(3,2). PAU CAT CCSS JUNY 2014 3.2

7. Determineu els valors dels paràmetres a, b i c que fan que les corbes d’equació

baxxxf 3)( i 2)( 23 cxxxg tinguin la mateixa recta tangent en el punt

(1, 1).

Solució PAU CAT CCSS JUNY 2013 4.6

8. Considerem les funcions 3)()( axxf i cbxxxg 2)( .

a) Determineu els valors dels paràmetres que fan que les dues corbes tinguin la mateixa

tangent en el punt (2, 1).

b) En el cas a= 1, feu una gràfica aproximada de la funció f.

Solució PAU CAT CCSS SET 2012 4.1

9. Considereu la funció –xex=f(x) .

a) Indiqueu-ne els extrems relatius, si n’hi ha, i classifiqueu-los.

b) Escriviu l’equació de la recta tangent a la corba en el punt d’abscissa 0.

Solució PAU CAT CCSS JUNY 2010 4.3

10. Calculeu els paràmetres a, b i c de la funció f(x) =ax

2 +bx +c, sabent que la recta

0=2–y–5x és tangent a la corba f(x) en el punt d’abscissa x= 0 i que el valor mínim

absolut que pren la funció és –49/12. Solució PAU CAT CCSS SET 2009 1.3

11. Considereu la funció 12

)(2

x

xxf .

a) Trobeu l’equació de la recta tangent a la corba y=f(x) en el punt d’abscissa x=2.

b) Determineu els intervals de creixement i decreixement, així com els extrems, si n’hi

ha. Solució PAU CAT CCSS SET 2007 3.1

12. La corba y=f(x) de la figura té per domini el conjunt de tots els nombres reals.

a) Determineu els punts on la funció val 0. Determineu els valors de x pels quals la

funció és positiva.

b) Digueu en quins punts s’anul·la la derivada i en quins punts f’(x)<0.

c) Trobeu l’equació de la recta tangent en el punt d’abscissa x=2.

d) Determineu la recta tangent en el punt d’abscissa x= –1.

e) Determineu a sabent que f(x)=a(x+1)(x–2)2.

Solució PAU CAT CCSS SET 2007 3.5

13. Considereu la funció real de variable real x

mxxf

2)( , on m és un paràmetre

real.

a) Calculeu el valor que ha de tenir m perquè la tangent a la gràfica de f(x) en el punt

d’abscissa x= –3 sigui paral·lela a la recta x –3y +1 = 0. Calculeu també l’equació

d’aquesta tangent.

Ara fixeu el valor de m=1.

b) Determineu el domini de la funció i els intervals on és creixent o decreixent.

c) Determineu-ne les asímptotes.

d) Dibuixeu un esbós de la gràfica resultant. Solució PAU CAT CCSS JUNY 2007 1.5

14. Considereu la funció 21

1)(

xxf

.

a) Calculeu l’equació de la recta tangent a la corba que representa f(x), en el punt

d’abscissa x = 2.

b) Quina és la funció que dóna el pendent de la recta tangent en cadascun dels punts de

la corba?

c) Calculeu el punt de la corba que representa f(x) en el qual el pendent de la recta

tangent és màxim. Trobeu el valor d’aquest pendent màxim.

Solució PAU CAT CCSS JUNY 2005 1.5

15. Considereu la funció 25001530

)( 23

xx

xf .

a) Calculeu l’equació de la recta tangent en el punt d’abscissa x = 0.

b) En quin punt de la corba és mínim el pendent de la recta tangent? Quin és el valor del

pendent mínim? Solució PAU CAT CCSS JUNY 2004 1.5

16. Esbrineu si les gràfiques de la funció f (x) = x

2 –2x +2 i de la recta y = 2x –2 són

tangents en algun punt. En cas que ho siguin, determineu aquest punt. Hi ha algun altre

punt d’intersecció entre la recta i la gràfica de la funció? Solució PAU CAT CCSS SET 2003 3.3

17. Trobeu l'equació de la recta tangent a la gràfica de la funció f(x) = 3x

2 +5x +1 en el

punt d'abscissa x = 2. Solució PAU CAT CCSS JUNY 2002 2.1

18. Considereu la funció y = ln x (on ln indica el logaritme en base e).

a) Determineu el seu domini de definició. Poseu en evidència que aquesta funció és

creixent en tot el seu domini.

b) Feu un esquema senzill de la seva gràfica tot indicant els límits de la funció quan

x i quan 0x .

c) Escriviu l'equació de la recta tangent a la gràfica d'aquesta funció en el punt

d'abscissa x = 1.

d) Escriviu l'equació de la recta tangent a la gràfica de la funció en el punt d'abscissa

a x i determineu a per tal que aquesta recta sigui paral·lela a y = 2x.

PAU CAT CCSS SET 1999 2.6

19. Escriviu l'equació de la recta tangent a la gràfica de la funció y = ex en el punt en

què aquesta gràfica talla l'eix de les y.

PAU CAT CCSS SET 1999 5.2

20. Escriviu l'equació de la recta tangent a la gràfica de la funció f(x)=e

3x en el punt

(0, 1).

PAU CAT CCSS SET 1998 5.3

8.3 Determinació del punt de tangència.

De vegades ens donen la recta tangent i necessitem obtenir el punt de tangència. En

aquests casos la propietat fonamental de la recta tangent:

)(' xfa

esdevé una equació que hem de resoldre.

Donada la paràbola 22)( xxf

a) Determineu la recta tangent a la seva gràfica al punt 2x .

b) Determineu la recta tangent a la seva gràfica que sigui paral·lela a la recta d’equació

42 yx .

a)

62)4(22)2(

4)2('2)('2)( 2

afb

faxxfxxf

La recta buscada és 64 xy

b)

31)2(11)1(

2)1('1222)('

afb

faxxxfa

La recta buscada és 32 xy

a) Determineu els punts de la gràfica de la funció 1234)( 3 xxf

en els què la recta tangent és paral·lela a la recta xy .

b) Determineu la recta tangent per a 2x .

a)

3 2

3/213/1

3/13

)23(

43)23(

3

4)23(

3

14)('

1)23(41234)(

xxxxf

xxxf

3/10

22364)23(4)23(41

)23(

4 233 2

3 2 x

xxxx

x

b) 1)2(' fa

912418412234)2( 33 f

62192)2( afb , la recta tangent és 6 xy

8.3.1 Problemes.

1. Trobeu els punts de la funció xxxxf 23)( en què la recta tangent és paral·lela a

la recta 52 xy .

2. Trobeu l’equació de les rectes tangents a la funció 23 3)( xxxf que són

paral·leles a la recta 039 yx .

3. Trobeu l’equació de la recta tangent a la funció 34)( 2 xxxf en els punts en

què la paràbola talla l’eix horitzontal.

4. Trobeu els valors d’x on les rectes tangents a les funcions 3)( 2 xxxf i 23)( xxxg són paral·leles. Doneu les seves equacions.

8.3.2 Problemes PAU TEC.

1. Considereu la funció polinòmica f(x) = x3 – ax

2 + bx + c.

a) Calculeu els valors dels paràmetres a, b i c, sabent que la funció té un extrem relatiu

en el punt d’abscissa x = 1 i que la recta tangent a la gràfica de la funció en el punt

d’abscissa x = 0 és la recta y = x + 3.

b) Per als valors a = 2, b = 1 i c = 3, calculeu les abscisses dels extrems relatius de la

funció i classifiqueu-los.

Solució PAU CAT TEC SET 2018 3.1

2. Sigui la funció f(x) = x

3 – x

2.

a) Trobeu l’equació de la recta tangent a la gràfica i que és paraŀlela a la recta d’equació

x + 3y = 0.

b) Calculeu, si n’hi ha, els punts de la gràfica en què la funció presenta un màxim o

mínim relatiu o un punt d’inflexió.

Solució PAU CAT TEC JUNY 2018 1.3

3. Donades la recta y=3x + b i la paràbola y = x

2,

a) Calculeu l’abscissa del punt on la recta tangent a la paràbola és paral·lela a la

recta donada.

b) Calculeu el valor del paràmetre b perquè la recta sigui tangent a la paràbola.

Solució PAU CAT TEC JUNY 2012 3.2

4. Sigui 132)( 23 xxxxf . Donades les rectes 2:1 xyr i 27:2 xyr :

a) Expliqueu, raonadament, si alguna de les dues rectes pot ser tangent a la corba

)(xfy en algun punt.

b) En cas que alguna d’elles ho sigui, trobeu el punt de tangència.

Solució PAU CAT TEC SET 2009 1.3

5. Digueu per a quin valor de x la recta tangent a la corba )1ln( 2 xy és paral·lela a la

recta xy . Escriviu l’equació d’aquesta tangent.

Solució PAU CAT TEC JUNY 2008 5.3

6. En quin punt la recta tangent a la funció xexxf )( és paral·lela a l’eix d’abscisses?

Escriviu l’equació de la recta tangent en aquest punt.

Solució PAU CAT TEC JUNY 2007 1.1

7. La corba d’equació 13 2 xy i la recta y = 4x + b són tangents.

a) Determineu el punt de tangència.

b) Determineu b. Solució (Curs 2004-2005. Setembre)

8. Calculeu el punt de la corba y = 2 +x –x2 en què la tangent és paral·lela a la recta

y=x.

Solució PAU CAT TEC SET 2003 3.2

9. Determineu els punts de la gràfica de f(x) = x4 + 5x on la recta tangent és paral·lela a

la bisectriu del primer quadrant. Calculeu l'equació d'aquestes rectes tangents.

Solució : (-1,-4), y = x-3. PAU CAT TEC JUNY 2000 3.2

10. a) Calculeu els punts del gràfic de la corba 12 23 xxxy on la recta tangent

té pendent 3

1 .

b) Determineu la recta tangent en aquests punts. Solució PAU CAT TEC JUNY 2005

11. Esbrineu si les gràfiques de la funció 22)( 2 xxxf i de la recta y = 2x – 2 són

tangents en algun punt. En cas que ho siguin, determineu aquest punt. Hi ha

algun altre punt d’intersecció entre la recta i la gràfica de la funció?

PAU CAT TEC SET 2003

12. En quin punt de la corba xxf ln)( la recta tangent és paral·lela a la corda AB

determinada pels punts A=(1,0) i B=(e,1) ? Solució PAU CAT TEC JUNY 1998 3.4

8.3.3 Problemes PAU CCSS.

1. Considereu la funció f(x) = –x2 + bx + c, amb b i c nombres reals.

a) Trobeu b i c de manera que la gràfica de la funció passi pel punt (–1, 0) i tingui un

extrem local en el punt d’abscissa x = 3. Raoneu de quin tipus d’extrem relatiu es tracta.

b) Per al cas b = 3 i c = 2, trobeu l’equació de la recta tangent a la gràfica que és

paraŀlela a la recta y = 5x – 2. Solució PAU CAT CCSS JUNY 2017 1.6

2. Considereu la funció 3

1)(

2

x

xxf .

a) Determineu els punts en què la funció f talla cadascun dels eixos. Determineu també

els intervals on la funció f és positiva.

b) Determineu els punts en què la recta tangent a la gràfica de f és horitzontal.

Solució PAU CAT CCSS SET 2016 1.5

3. Considereu la funció 2

22)(

2

xx

xxf .

a) Escriviu l’equació de la recta tangent a la gràfica de f en el punt de tall amb l’eix de

les ordenades.

b) Determineu els punts de la corba en què la recta tangent és horitzontal. Solució PAU CAT CCSS JUNY 2016 2.3

4. Considerem la funció x

xf12

)( .

a) Indiqueu-ne el domini i estudieu-ne el creixement.

b) Calculeu les equacions de les rectes tangents a la gràfica de f que són paral·leles a la

recta y+3x=2. Solució PAU CAT CCSS SET 2012 4.2

5. Sabem que la funció 3

13)( 23 bxxaxxf passa pel punt (1, 0), i que la recta

tangent a la gràfica de la funció en aquest punt és paral·lela a la recta 12x –2y=3.

a) Determineu els valors dels paràmetres a i b.

b) Per a a=1 i b=9, determineu, si n’hi ha, les abscisses dels extrems possibles (màxims

o mínims) de la funció, i classifiqueu-los. Solució PAU CAT CCSS SET 2011 2.6

6. Donada la funció següent: 1

4)(

2

x

xxf

a) Determineu-ne les asímptotes horitzontals i verticals, si n’hi ha.

b) Trobeu els punts de la corba en què la recta tangent és paral·lela a la recta

43 xy

Solució PAU CAT CCSS JUNY 2010 4.2

7. Considereu la funció següent:

2

2 23)(

x

xxxf

a) En quin punt de la corba la recta tangent a la gràfica de f és paral·lela a la recta

x+y=5?

b) Calculeu les asímptotes horitzontals i verticals de la funció, si n’hi ha, i feu un esbós

de la gràfica de la funció f.

Solució PAU CAT CCSS JUNY 2010 5.4

8. Considereu la funció x

xxf

23)(

.

a) Trobeu els punts de la gràfica en els quals la recta tangent és paral·lela a la recta

3x+ 4y+ 5 = 0.

b) Calculeu les equacions d’aquestes rectes tangents. Solució PAU CAT CCSS JUNY 2006 1.1

9. La corba d’equació y = 3x

2 – 1 i la recta y = 4x + b són tangents.

a) Determineu el punt de tangència.

b) Determineu b. Solució PAU CAT CCSS SET 2005 3.3

10. a) Calculeu els punts del gràfic de la corba y = x

3 – 2x

2 + x + 1 on la recta tangent té

pendent 3

1 .

b) Determineu la recta tangent en aquests punts. Solució PAU CAT CCSS JUNY 2005 4.3

11. Calculeu en quin punt (si és que n'hi ha algun) la recta tangent a la gràfica de la

funció f(x) = e2x

forma un angle de 45° amb l'eix de les x.

Solució PAU CAT CCSS JUNY 2001 2.2

11. Calculeu l'abscissa del punt en què la tangent a la gràfica de la funció f(x) = 2 ln x és

paral·lela a la recta 16x – 2y = 7.

PAU CAT CCSS SET 2000 2.4

8.3.4 Exercicis en anglès.

Solve Tangent Lines Problems in Calculus.

a) Find all points on the graph of y = x 3 - 3x where the tangent line is parallel to the x

axis (or horizontal tangent line).

b) Find a and b so that the graph of y = ax3 + bx is tangent to the line y = -3x + 4 at x =

1.

c) Find conditions on a and b so that the graph of y = ae x + bx has NO tangent line

parallel to the x axis (horizontal tangent).

d) Find all points on the graph of y = x 3 - 3x where the tangent line is parallel to the

line whose equation is given by y = 9x + 4.

e) Find a and b so that the graph of y = ax2 + bx is tangent to the line y = -2 at x = 1.

f) Find conditions on a, b and c so that the graph of y = ax 3

+ bx 2

+ cx has ONE tangent

line parallel to the x axis (horizontal tangent).

Font: http://www.analyzemath.com/calculus/Problems/tangent_lines.html

8.4 Recta normal a la gràfica d’una funció.

De forma similar a com vam definir la recta tangent, també podem definir la recta

normal:

Donada una funció f(x) i un punt P=(p, f(p)) de la seva gràfica, existeix una única recta

y=ax+b que compleix alhora les dues propietats següents:

a) És perpendicular a la gràfica de la funció f en x = p.

b) Passa per P.

Determinació de la recta normal:

La recta és perpendicular a la gràfica de la funció en ))(,( pfpP , per tant,

)('

1

pfa

La recta passa pel punt P, per tant bappf )( , i d’aquí aïllem b.

Trobar la recta tangent a la funció 2)( xxf en el punt x=1

Tenim (fent p=1)

1)1()( 2 fxxf

2

1

)1('

12)1('2)('

fafxxf

2

3

2

11

)1('

1)1(

ffb

La recta tangent és doncs 2

3

2

1

xy

Exercicis:

1. Determina les rectes normals a la funció xxf )( en els punts A d’abscissa x=1, B

d’abscissa x=2 i C d’abscissa x=0.5

2. Determina les rectes normals a la funció xexf )( en els punts A d’abscissa x=-0.5,

B d’abscissa x=0 i C d’abscissa x=-1.5

3. Determina les rectes normals a la funció x

xf1

)( en els punts A d’abscissa x=-0.5, B

d’abscissa x=1 i C d’abscissa x=1.5

9 Funcions exponencials i logarítmiques.

9.1 Extrems relatius de funcions exponencials i logaritmes.

Donada la funció

xe

xxf

2

)(

a) Determineu el seu domini.

b) Estudi de la monotonia (creixement, decreixement, màxims i mínims relatius).

c) Estudi de la curvatura (convexitat, concavitat i punts d’inflexió).

d) Estudieu el comportament d’aquesta funció en l’infinit.

e) Representeu gràficament aquesta funció.

a) Domini: IR . (el denominador no s’anul·la mai).

b)

xx

xx

x

x

xx

x

ee

xx

e

xx

e

xxe

e

exxexf

e

xxf

2

222)(')(

2

22

22

0,20)(' xxxf

)0,( )2,0( ),2(

)(' xf 03)1(' ef 0/1)1(' ef 0/3)3(' 3 ef

)(xf decreixent creixent Decreixent

La funció té un màxim relatiu al punt )0,0( i un mínim relatiu al punt 54.0,24

,22

e

c)

xx

x

x

x

x

x

xx

ee

xxxxxx

e

xxe

e

xxxe

e

exxexxf

)1(2222)2(

)42)2()1(2)2()1(2)(''

2

2

2

22

414.322

586.0220420

420)('' 2

2

2

xxxe

xxexf

x

x

)22,( )22,22( ),22(

)('' xf 02)0('' f 0/1)1('' ef 0/2)4('' 4 ef

)(xf convexa còncava convexa

La funció té punts d’inflexió a )191.0,586.0( i )384.0,414.3(

d) La funció és una exponencial contra una potencial, guanya l’exponencial, per tant

xxxxx

xxx

ex

e

xxf

e

xxf

1limlim)(lim

0lim)(lim

22

2

e)

1. Determina els extrems relatius de les següents funcions:

a) 2

)(x

exf

x

b) 3

)(x

exf

x

c) xe

xxf

3

)(

d) x

xxf

)ln()(

e) 2

)ln()(

x

xxf

f) 3

)ln()(

x

xxf

2. Determina les coordenades dels extrems relatius (màxims M i mínims m) i dels

possibles punts d’inflexió I de les següents funcions:

a) xe

xxf )(

b) xe

xxf

2

)(

c) x

xxf

ln)(

d) x

xxf

ln)(

2

e) x

exf

x

)(

9.2 Exercicis de derivació amb exponencials i logaritmes.

1. Estudieu la monotonia i la curvatura de la funció )1ln()( 2 xxf .

2. Determineu el punt de la gràfica de la funció

1ln)(

2

x

xxf en el què la recta

tangent és paral·lela a l’eix OX.

3. Estudieu la monotonia de la funció 2)1(

)(x

x

e

exf

.

4. Estudieu la monotonia de la funció 221

4)(

x

xxf

.

5. Estudieu la monotonia i la curvatura de la funció 2ln)( xxxf .

6. Estudieu la monotonia i la curvatura de la funció xe

xxf

1)(

2 .

7. Estudieu la monotonia de la funció x

exexf

x)( .

(Retalls de problemes de la selectivitat de Madrid)

9.2 Problemes PAU TEC amb funcions exponencials i logarítmiques.

1. Sigui la funció bxxeaxf 2

)( , amb 0a 0 i 0b .

a) Calculeu els valors de a i de b que fan que la funció tingui un extrem relatiu en el

punt (1, e).

b) Per al cas a = 3 i b = 5, calculeu l’asímptota horitzontal de la funció f quan x tendeix

a +∞. Solució PAU CAT TEC JUNY 2018 5.3

2. Sigui la funció f(x) = ex – x – 2.

a) Demostreu que la funció f té una arrel (un zero) en l’interval [0, 2].

b) Comproveu que la funció és monòtona en l’interval [0, 2] i calculeu les coordenades

dels punts mínim absolut i màxim absolut de la funció en aquest interval.

Solució PAU CAT TEC SET 2015 5.3

3. Sigui xaexxf 2)( quan 0a .

a) Calculeu el valor de a perquè aquesta funció tingui un extrem relatiu en el punt

d’abscissa 2x .

b) Quan 2a , classifiqueu-ne els extrems relatius. Solució PAU CAT TEC JUNY 2011 1.6

4. Donades les funcions 2

)(xx ee

xf

i 2

)(xx ee

xg

:

a) Comproveu que 1)()(22 xfxg

b) Comproveu també que )()(' xgxf i )()(' xfxg

c) Comproveu que )()()()()( xgyfxgxfyxf

d) Calculeu )(

)(lim

xg

xf

x dividint per xe el numerador i el denominador; amb un

procediment similar (però no igual), trobeu )(

)(lim

xg

xf

x

Solució PAU CAT TEC SET 2008 4.5

5. Donada la funció xxexf 22

)( .

a) Trobeu el seu domini i les possibles interseccions amb els eixos.

b) Trobeu els intervals on creix i decreix i els extrems relatius.

c) Trobeu les possibles asímptotes.

d) Feu la representació gràfica aproximada de la funció. Solució PAU CAT TEC JUNY 2006 3.5

6. Sigui :)(xf la funció definida per )()( baxexf x , on a i b són nombres

reals.

a) Calculeu els valors de a i b per tal que la funció tingui un extrem relatiu en el punt

3,3 e .

b) Per als valors de a i b obtinguts, digueu quin tipus d’extrem té la funció en el punt

esmentat. Solució PAU CAT TEC SET 2006 4.1

7. El consum d’un cotxe depèn de la seva velocitat v (expressada en km/h) segons la

funció v

evf

v012,03)( (en litres/km). Quina és la velocitat més econòmica?

Solució PAU CAT TEC JUNY 2004 4.3

9.3 Problemes PAU CCSS amb funcions exponencials i logarítmiques.

1. Durant la darrera epidèmia d’Ebola es va considerar que, sense cap intervenció, el

virus es propagava augmentant en un 3 % diari el nombre d’afectats. Suposeu que en

una població, avui, hi ha 25 persones infectades.

a) Escriviu la fórmula de la funció que dóna el nombre de persones infectades en passar

els dies. Quantes persones estaran infectades al cap de 20 dies?

b) A partir d’una data determinada, en aquesta població s’apliquen unes mesures

sanitàries que permeten que el nombre de persones infectades disminueixi segons la

funció g(x) = 1.000 · (0,95)x. Si considerem controlada l’epidèmia quan el nombre

d’afectats és igual o inferior a 10 persones, quants dies hauran de passar després

d’aplicar les mesures sanitàries per a poder declarar controlada l’epidèmia?

Solució PAU CAT CCSS JUNY 2016 3.4

2. La funció derivada d’una funció f és f′(x) = e–2x

· (x – x2).

a) Estudieu el creixement i el decreixement de la funció f.

b) Si la funció f té extrems relatius, indiqueu-ne les abscisses i classifiqueu-los.

Solució PAU CAT CCSS JUNY 2016 2.5

3. La funció derivada d’una funció f és )()(' 2xxexf x .

a) Estudieu el creixement i el decreixement de la funció f.

b) Si la funció f té extrems relatius, indiqueu-ne les abscisses i classifiqueu-los.

Solució PAU CAT CCSS SET 2014 5.3

4. S’han corregit unes quantes proves de selectivitat i s’han puntuat amb notes entre 0 i

10. El nombre de persones que han obtingut una determinada qualificació x és definit

per la funció .)92(250)( 2 xxN

a) Quantes persones han tret un 10 en aquesta prova? Quantes persones han tret un 6?

b) Quina és la nota que han tret més persones? Quantes persones han tret aquesta nota?

Solució PAU CAT CCSS SET 2014 5.2

5. Sigui la funció xexxf )( .

a) Si la funció f té extrems relatius, determineu-los i classifiqueu-los.

b) Calculeu la recta tangent a la gràfica de f en el punt d’abscissa x=0.

Solució PAU CAT CCSS JUNY 2014 3.6

6. Donada una funció f, sabem que xxexf x 32)(' 2 .

a)Estudieu el creixement i el decreixement de la funció f.

b) Si la funció f té extrems relatius, indiqueu-ne les abscisses i classifiqueu-los.

Solució PAU CAT CCSS SET 2013 1.1

7. Segons uns estudis de laboratori, l’evolució de la població en un cultiu de bacteris al

llarg del temps segueix la funció 10)1(30)( tetf , on t són els dies que han

transcorregut des de l’inici de l’experiment, i f(t) és la població, en milions de bacteris.

a) Quina població hi ha en el moment de començar l’experiment? Justifiqueu si en algun

moment hi arribarà a haver 40 milions de bacteris.

b) Hi haurà algun moment en què la població sigui màxima? Justifiqueu la resposta.

Solució PAU CAT CCSS JUNY 2013 3.5

8. Considereu la funció xexxf 3)( .

a) Indiqueu-ne el domini, i demostreu que f és estrictament creixent en tot el domini.

b) Calculeu l’equació de la recta tangent a la gràfica de f en el punt d’abscissa x=0.

Solució PAU CAT CCSS JUNY 2011 1.6

9. Considereu la funció )ln()( xxxf .

a) Indiqueu-ne el domini. Determineu l’asímptota vertical de la funció f.

b) Determineu els intervals en què la funció f és creixent i els intervals en què és

decreixent, i classifiqueu-ne els extrems possibles.

Solució PAU CAT CCSS JUNY 2011 4.1

10. Un bosc té una massa forestal de 40000m3 de fusta. Es calcula que la pluja àcida i

els incendis provoquen una disminució del 6% anual de l’esmentada massa forestal, que

es pot expressar en termes de la funció txF 94,040000)( , en què F(t) és la massa

forestal que queda passats t anys.

a) Justifiqueu que la funció F és estrictament decreixent.

b) D’aquí a quants anys la massa forestal s’haurà reduït a la meitat?

Solució PAU CAT CCSS JUNY 2011 4.6

11. Donada la funció f(x) =x2·e

x:

a) Justifiqueu si hi ha cap valor de x que compleixi f(x )<0. Hi ha cap valor de x que

compleixi f(x)=0?

b) Indiqueu si la funció f és creixent o decreixent en el punt x= –1. Estudieu el

creixement de la funció f per als valors que compleixen x> 0.

Solució PAU CAT CCSS JUNY 2010 5.5

12. Determineu els intervals de creixement i decreixement, així com els màxims i

mínims, de la funció f(x) =x2e

–x.

Solució PAU CAT CCSS JUNY 2008 2.2

13. Calculeu a i b de manera que f(x) = a ln(x) + bx2 + x tingui extrems relatius en els

punts d’abscisses x = 1 i x = 2, i digueu, en cada cas, si es tracta d’un màxim o d’un

mínim.

Solució PAU CAT CCSS JUNY 2005 1.3

14. Fa quatre anys es va repoblar un llac amb una nova espècie de peixos. Llavors es

van introduir 100 exemplars d'aquesta nova espècie. Actualment s'estima que hi ha

25.000 exemplars. S'estima que el nombre N de peixos ve donat en funció del temps t

per la funció N = AeBt

, on A i B són dues constants. El temps t es considera expressat

en anys des del moment de la repoblació. Quant temps haurem d'esperar perquè hi hagi

200.000 exemplars?

Solució PAU CAT CCSS JUNY 2000 1.6

10 Les funcions trigonomètriques.

10.1 Pràctica derivant funcions trigonomètriques.

Dues coses molt importants que has de tenir clares al treballar amb funcions

trigonomètriques:

Recorda que les fórmules de les derivades de les funcions trigonomètriques:

)cos()(')sin()( xxfxxf

)sin()(')cos()( xxfxxf

)(tan1)(')tan()( 2 xxfxxf

Només funcionen si la unitat de mesura angular són els radians. Per tant,

comprova sempre que la calculadora està en “Mode Rad” R quan treballis amb

aquestes derivades.

Recorda que les funcions trigonomètriques són periòdiques, i per tant les equacions de

la forma

ax )sin( ax )cos( ax )tan(

tenen, en general, infinites solucions.

La calculadora científica només et donarà una d’elles, la que està més pròxima al 0:

)arcsin()sin( axax

)arccos()cos( axax

)arctan()tan( axax

Per exemple, la calculadora científica dóna com a solució de l’equació

0)sin( x

el valor

0)0arcsin( x

però també són solucions tots els valors

,...4,3,2,,...,4,3,2,.0

10.2 Exercicis.

1. Calcula la derivada de x

xxf

sin1

cos)(

2. Donada la funció xxf 3cos)( , resol l’equació 0)(' xf

10.3 Problemes PAU CCSS.

1. Considereu la funció f(x) = 2002x2 +ax +b +sin x, amb a, b reals. Calculeu els valors

dels paràmetres a, b perquè f passi pel punt (0, 3) i tingui un extrem relatiu en aquest

punt. Expliqueu raonadament quin tipus d'extrem té f en aquest punt.

Solució PAU CAT CCSS SET 2002 1.5

10.4 Recta tangent amb funcions trigonomètriques.

Determinació de la recta tangent amb funcions trigonomètriques.

1. Calcula la recta tangent y=ax+b a la funció f en el punt x donat.

a) f(x) = sin(x) x = 0.9

b) f(x) = sin(x) x = 2.8

c) f(x) = cos(x) x = -0.7

d) f(x) = tan(x) x = 0.8

e) f(x) = tan(x) x = -0.5

2. Calcula el punt de tangència P=(x,y) coneixent la recta tangent g(x)=ax+b a la funció

f en aquest punt

a) f(x) = sin(x) y = 0.6216x + 0.2239

b) f(x) = cos(x) y = -0.8415x + 1.3818

10.5 Exercicis.

1. Calculeu la primera i segona derivada de la funció x

xxf

cos2

sin)(

2. Determineu l’equació de la recta tangent a la gràfica de la funció 2)(sin1

1)(

xxf

al punt

4,

4

f .

11 Problemes d’optimització.

11.1 Fases de la resolució.

Un problema de la física, economia,

geometria... que hem d’optimitzar, és

a dir, trobar el punt mínim o màxim.

Una equació que relaciona les

variables x i y entre elles.

De vegades hem d’aplicar

coneixements de geometria de

cursos anteriors.

Una funció ),( yxf per a la què

hem de trobar el punt màxim o

mínim.

Però (ai!) és multivariable, i no

sabem treballar amb funcions

multivariables.

Una funció )(xf per a la

què hem de trobar el màxim

o mínin, amb una única

variable x.

Hem de resoldre l’equació

0)(' xf

I després comprovar que realment

es tracta d’un extrem adequat.

Hem de comprovar que la solució

(x,y) proposada té sentit dintre del

context inicial del problema.

En primer lloc hem de llegir detingudament

l’enunciat, fer un dibuix esquemàtic de la

situació i posar-hi lletres: x,y.

Sense lletres no podem fer matemàtiques!

11.2 Exemples resolts.

1. Determineu, entre tots els rectangles de diagonal 4 m., aquell que té superfície

màxima.

Funció: yxyxÀrea ),( (màxim)

Equació: 2222 10010 xyyx (Teorema de Pitàgores)

22

2

2

22

2

22

100)2(

1002

1100

1

100100

1001001)('100)(

x

xx

xx

x

x

xx

x

xxxxAxxxA

071.7501002

1002100100

100100

1000100

100)('

22222

22

2

2

22

2

22

xxxxxx

xxx

xx

x

xxxA

L’única solució acceptable és la positiva. Comprovem que es tracta d’un màxim:

071.7x

02

7)6(' f

0

3

14)8('

f

f creixent Màxim

relatiu f decreixent

2 4 6 8 10

10

20

30

40

50

50501002

y , la figura solució és un quadrat.

2. Trobeu el punt de la recta 532 yx més pròxim a l’origen (0,0).

La funció ),( yxf que hem d’optimitzar (en aquest cas trobar el mínim) és la distància

22),( yxyxf

L’equació que lliga ambdues variables és l’equació de la recta:

3

25253532

xyxyyx

Per tant la funció a optimitzar és

2

2

3

25)(

xxxf

Derivem aquesta funció aplicant la regla de la cadena:

9

)25(42

3

252

1)('

2

2

xx

xx

xf

9

)25(42

9

)2)(25(22)('

9

25

3

25)(

2

2

2

2 xx

xxxg

xx

xxxg

13

100

9

)25(420

9

)25(42

3

252

10)('

2

2

xx

xx

xx

x

xf

Podem garantir que es tracta efectivament d’un mínim avaluant la derivada a l’esquerra

i a la dreta del punt 77.013

10x

decreixentf 0174.0)6.0('

creixentf 0032.0)8.0('

La solució és, doncs, 13/10x , 13

15

3

)13/10(25

y , és a dir, el punt

)15.1,77.0(P

3. Una cartolina rectangular de perímetre 36 cm i dimensions x , y gira al voltant d’un

costat fixe de longitud y desenvolupant un cilindre C. Per a quins valors de x i y

obtenim un cilindre de volum màxim?

Perímetre : xyyxyx 18183622

Volum : )18(22 xxyrhAb

xxxxxxxxxx

xxxxxxxx

x

xx

xxxV

123336236)18(2

).18(2)1().18(2)18(

118

2

)18()(

222

2

2

12012

0030)12(3)('

xx

xxxxxV

La solució x = 0 no té sentit.

Per a x = 12 observem que és un màxim relatiu pel criteri de la segona derivada:

)6(6)212(3)12(3

3)12(3)1(3)12(3)12(3

112

33

xxxx

xxxxxx

x

x

)6(6)('' xxV

036)126(6)12('' V

També podríem haver garantit el màxim en x = 12 avaluant la primera deriva de la

funció just abans i després d’aquest punt:

02.11)9.11('

04.11)1.12('

V

V

Les dimensions de la cartolina són x = 12 , y = 18 – 12 = 6

Font: http://catedu.es/matematicas_mundo/PAU/Analisis_CNS.pdf

4. Considereu un prisma recte de base rectangular, amb dos dels costats d’aquest

rectangle de longitud doble que els altres dos, tal i com s’indicava a la figura. Trobeu les

dimensions que ha de tenir aquest prisma per tal que la seva àrea total sigui 12 metres

quadrats i que amb aquestes condicions tingui un volum màxim.

aceptable no 1

10)('

1413

1266

3

2)('

263

226

3

2

3

2622

3

26263632622

12)22(2

222

322

2

2222

2

xxV

xxxxV

xxxxx

xxhxxV

x

xhxxhxhxxhxhx

xhxhxS

màximun és 8)1(''

824)(''

V

xxxV

Per tant la solució és x = 1 m , mh3

4

13

126 2

5. Determineu el punt Q de la gràfica 2xy més a prop del punt )3,6(P

Sigui ),( yxQ el punt que volem trobar.

Volem trobar el mínim de la funció distància entre dos punts, que és el mòdul del vector

)3,6( yxPQPQ

22 )3()6(),( yxPQPQd

El punt pertany a la paràbola 2xy , i per tant 222 )3()6()( xxxd

Derivem:

222

3

222

3

222

3

2

222

)3()6(

652

)3()6(2

12104

)3()6(2

124122

2)3(2)6(2)3()6(2

1)('

xx

xx

xx

xx

xx

xxx

xxxxx

xd

06520)3()6(

6520)(' 3

222

3

xx

xx

xxxd

Resolem per Ruffini per obtenir l’única solució real possible 2x .

A l’esquerra de 2x la derivada és negativa, i a la dreta de 2x la derivada és

positiva, per tant es tracta efectivament d’un mínim.

La solució és, doncs, )4,2(42,2 2 Qyx

6. Entre tots els cilindres d’un metre de diagonal, determineu l’alçària h i el radi r del

que té volum màxim.

Dintre del cilindre podem observar un triangle rectangle de hipotenusa 1, base 2r i altura

h, en el qual podem aplicar Pitàgores:

22

2222222

4

14141)2(1 r

hrhhrhr

322

2

41

44

1hhhhh

hhrhAV base

hhhhhV 3

1

3

131031031

4' 2222

3

3

33

3

3

1

3

1

3

1h

Podem garantir que el valor h és un màxim tenint veient que 'V és una paràbola amb les

branques cap a a vall o aplicant el criteri de la segona derivada.

6

6

66

6

6

1

6

1

6

1

6

1

4

3/2

4

3/11

4

1 22

r

hr

Solució: 6

6r ,

3

3h

7. Determina l’angle d’un sector circular que hem d’eliminar d’un cercle de radi 1

pera construir un con de volum màxim.

Volum del con:

hrhAV b 2

3

1

3

1

On r és el radi del cercle de la base del con i h és l’altura del con.

Al con tenim un triangle rectangle que relaciona el radi r de la base, l’altura h i la

generatriu lateral que és 1.

22222 11 hrhr

A més a més, sabem la relació entre el radi r i la longitud 1 del cercle de la base del con:

rl 2

Però, per construcció, la longitud 1 del cercle de la base del con és la longitud del sector

circular, que és proporcional a l’angle del sector (és un cercle de radi 1):

2360

360 l

Així doncs, ja tenim les equacions que relacionen V amb l’angle .

Escrivim el volum V en funció de l’altura h:

3

1

031

0313

10)('

313

1)('

3

11

3

1)(

3

1

2

2

2

322

h

h

hhV

hhV

hhhhhVhrV

L’única solució vàlida és la positiva: 3

1h

Per tant:

3

2

3

2

3

11

3

111

2

22

r

hr

Prenem només la solució positiva: 3

2r

3

222 rl

I per últim:

66.0612º 3

21360

3

2360360

3603

2360

360

360

3

2

2360

360

3

22

50 100 150 200 250 300 350

0.1

0.2

0.3

0.4

Gràfica del Volum V en funció de l’angle

El volum és màxim per a l’angle 66.0612º 3

21360

8. Un mirall pla que tenia forma rectangular de costats 300 x 165 cm s’ha trencat per

una cantonada, amb un tros petit en forma de triangle rectangle de catets 90 cm i 50 cm.

Trobeu les dimensions del mirall rectangular d’àrea màxima que es pot retallar del tros

que queda, de forma que els costats del nou mirall siguin paral·lels als costats del mirall

inicial.

Definim un sistema d’eixos de coordenades tal i com s’indica al següent esquema, on el

punt P=(x,y) és la cantonada inferior esquerra del rectangle gris:

Per tant, l’àrea que volem maximalitzar és )165)(300( yxA

La relació entre les dues variables x, y la trobem tenint en compte que el punt P=(x,y)

ha d’estar a la recta que passa pels punts A=(90,0) y B=(0,50). Determinem la recta

baxy que passa pels punts A=(90,0) y B=(0,50):

9

5

90

505090900

50050

aaba

bba

La recta és 509

5

xy

I per tant es queda la funció:

345003

155

9

5

)9/5115)(300(

)509/5165)(300(

))509/5(165)(300()(

2

xx

xx

xx

xxxA

5.462

930

3

155

9

10)('

xxxA

A(x) és una paràbola amb les branques cap avall, i per tant garantim que el punt anterior

és un màxim.

24.1667 6

14550

2

93

9

5

y

Per tant, les dimensions del nou mirall són:

5.2532/93300300 x

140.83 6/145165165 y

9. Un cocodril vol capturar una zebra que està 20m més endavant però a l’altra banda

d’un riu de 6 metres d’ample. El cocodril viatja a diferent velocitat a l’aigua i a terra:

Per l’aigua a 1/5 metres per segon i per terra a 1/4 de metres per segon. Calcula el temps

mínim que necessitarà per atrapar la zebra.

La part del trajecte que recorre per l’aigua es dedueix per Pitàgores: 226 x . Si

avança 1/5 metres per segon, cada metre necessita 5 segons, per tant el temps total

nedant serà 2265 x .

La part restant la recorre per terra i és 20x . Si avança a una velocitat de 1/4 metres

per segon, cada metre necessita 4 segons, i per tant el temps serà )20(4 x

La funció temps que hem de minimitzar és la suma dels dos temps parcials:

)20(465)( 22 xxxT

8

649

576576957616251657625

)36(162536453645

436

504

36

5042

362

15)('

222222

222

222

222

x

xxxxxx

xxxxxx

x

x

x

xx

xxT

Hem de comprovar que es tracta d’un mínim:

0-0.0956)5.7('T decreixent

00.08484)5.8('T creixent.

La funció té un mínim a 8x

2 4 6 8 10 12 14

100

102

104

106

11.3 Problemes PAU TEC d’optimització.

1. Les pàgines d’un llibre han de tenir cada una 600 cm2 de superfície, amb uns marges

al voltant del text de 2 cm a la part inferior, 3 cm a la part superior i 2 cm a cada costat.

Calculeu les dimensions de la pàgina que permeten la superfície impresa més gran

possible.

PAU CAT TEC JUNY 2019 1.1 (Solució: "Compendium Tec", Pàg. 528)

2. Considereu un con de 120 cm3 de volum que té una altura h, un radi de la base x i una

aresta a, com el de la figura següent:

a) Comproveu que 22 1360h

ha

.

b) Calculeu l’altura del con que té l’aresta de longitud mínima.

Nota: Recordeu que el volum del con és un terç del volum del cilindre recte que té la

mateixa base i la mateixa altura que el con.

Solució PAU CAT TEC JUNY 2017 1.6

3. Volem fer un envàs de gelat amb forma de prisma regular de base quadrada i amb una

capacitat de 80 cm3. Per a elaborar-ne la tapa i la superfície lateral, farem servir un

material determinat que costa 1 €/cm2, però per a la base haurem d’utilitzar un material

que és un 50 % més car.

a) Si x és la mesura, en cm, del costat de la base, comproveu que la funció que

determina el preu de l’envàs és x

xxp320

5,2)( 2 .

b) Calculeu les mides que ha de tenir l’envàs perquè el preu sigui el mínim possible. Solució PAU CAT TEC SET 2016 1.3

4. Considereu el tetràedre que té per vèrtexs els punts A = (x, 0, 1), B = (0, x, 1), C = (3,

0, 0) i D = (0, x, 0), amb 0 < x < 3.

a) Comproveu que el volum del tetràedre és donat per l’expressió xxxV 36

1)( 2 .

b) Determineu el valor de x que fa que el volum sigui màxim i calculeu aquest volum

màxim.

Nota: Podeu calcular el volum del tetràedre de vèrtexs A, B, C i D amb l’expressió.

ADACAB ,,det6

1

Solució PAU CAT TEC JUNY 2016 3.5

5. Siguin x i y les mesures dels costats d’un rectangle inscrit en una circumferència de

diàmetre 2.

a) Comproveu que la superfície del rectangle, en funció de x, és donada per l’expressió

424)( xxxS .

b) Calculeu els valors de les mesures x i y per als quals la superfície del rectangle és

màxima i calculeu el valor d’aquesta superfície màxima. Solució PAU CAT TEC SET 2015 5.5

6. La portalada d’una catedral està formada, en la part superior, per un arc de mitja

circumferència que recolza sobre dues columnes, com iŀlustra la figura adjunta, en què x

és el diàmetre de la circumferència, és a dir, la distància entre columnes, i y és l’alçària

de cada columna.

a) Comproveu que la funció xyx

yxf 8

),(2

determina l’àrea d’aquesta portalada.

b) Si el perímetre de la portalada fa 20 m, determineu les mides x i y de la portalada que

en maximitzen l’àrea. Solució PAU CAT TEC JUNY 2015 2.6

7. Un nedador és al mar en un punt N, situat a 3 km d’una platja recta, i just al davant

d’un punt S, situat a la platja arran de l’aigua; i vol anar a un punt A, situat també arran

de l’aigua i a 6 km del punt S, de manera que el triangle NSA és rectangle en el vèrtex

S. El nedador neda a una velocitat constant de 3 km/h i camina a una velocitat constant

de 5 km/h.

a) Si P és un punt entre el punt S i el punt A que està a una distància x de S, demostreu

que el temps, en hores, que necessita el nedador per a nedar del punt N al punt P i

caminar des del punt P fins al punt A és determinat per l’expressió

5

6

3

9)(

2 xxxt

.

b) Calculeu el valor de x que determina el temps mínim que cal per a anar del punt N al

punt A, passant per P. Quin és el valor d’aquest temps mínim?

Solució PAU CAT TEC JUNY 2014 3.3

8. Volem construir una tenda en forma de piràmide regular de base quadrada. Disposem

de 300 m2 de tela per a la fabricació de les quatre cares de la tenda (se suposa que en

l’elaboració de les cares no es perd gens de tela). Designem x la longitud d’un costat de

la base de la tenda.

a) Sabent que el volum d’una piràmide és igual a un terç del producte de l’àrea de la

base per l’altura, comproveu que, en aquest cas,

6

)109()(

44 xxxV

b) Determineu el valor de x perquè el volum sigui el més gran possible (no cal que

comproveu que el valor obtingut correspon realment a un màxim).

Solució PAU CAT TEC SET 2013 1.6

9. Un triangle rectangle situat en el primer quadrant té el vèrtex A en l’origen de

coordenades, el vèrtex B = (x, 0) en el semieix positiu d’abscisses i el vèrtex C pertany

a la recta x + 2y = 8. L’angle recte és el que correspon al vèrtex B.

a) Comproveu que l’àrea del triangle es pot expressar de la manera següent:

42)(

2xxxA .

b) Trobeu els vèrtexs B i C perquè l’àrea del triangle sigui màxima i comproveu que es

tracta realment d’un màxim.

Solució PAU CAT TEC JUNY 2013 5.6

10. Es vol construir un canal que tingui com a secció un trapezi isòsceles de manera que

l’amplària superior del canal sigui el doble de l’amplària inferior i que els costats no

paraŀlels siguin de 8 metres.

A la dreta teniu un esquema de la secció del canal.

a) Trobeu el valor del segment L de la gràfica en funció de la variable x (amplària

inferior del canal).

b) Sabem que l’àrea d’un trapezi és igual a l’altura multiplicada per la semisuma de les

bases. Comproveu que, en aquest cas, l’àrea de la secció és donada per

4

2563)(

2xxxA

c) Calculeu el valor de x perquè l’àrea de la secció del canal sigui màxima (no cal que

comproveu que és realment un màxim).

Solució PAU CAT TEC JUNY 2013 4.4

11. En una semiesfera de radi R inscrivim un con situant el vèrtex al centre de la

semiesfera, tal com es veu en el dibuix.

a) Sabent que el volum d’un con és igual a l’àrea de la base multiplicada per l’altura i

dividida per 3, comproveu que, en aquest cas, podem expressar el volum com

22

3hR

hV

b) Trobeu les dimensions d’aquest con (el radi de la base i l’altura) perquè el seu volum

sigui màxim i comproveu que es tracta realment d’un màxim.

Solució PAU CAT TEC JUNY 2013 3.5

12. Un triangle equilàter de vèrtexs A, B i C té els costats de 8 cm. Situem un punt P

sobre una de les altures del triangle, a una distància x de la base corresponent.

a) Calculeu l’altura del triangle de vèrtexs A, B i C.

b) Indiqueu la distància del punt P a cadascun dels vèrtexs (en funció de x).

c) Determineu el valor de x perquè la suma dels quadrats de les distàncies del punt P a

cadascun dels tres vèrtexs sigui mínima. Solució PAU CAT TEC JUNY 2012 3.5

13. Dins d’un triangle rectangle, de catets 3 i 4 cm, hi ha un rectangle. Dos costats del

rectangle estan situats en els catets del triangle i un dels vèrtexs del rectangle és a la

hipotenusa del triangle.

a) Feu un esbós de la situació descrita.

b) Si x és la longitud del costat del rectangle que està situat en el catet petit i y és l’altre

costat del rectangle, comproveu que es compleix que 1234 yx .

c) Determineu les dimensions del rectangle perquè l’àrea sigui màxima.

Solució PAU CAT TEC JUNY 2011 4.6

14. Un segment de longitud fixada m recolza sobre els eixos de coordenades. Calculeu

el valor de l’angle que forma el segment amb l’eix OX perquè el triangle

rectangle determinat pel segment amb els eixos i del qual m és la hipotenusa

tingui àrea màxima. Comproveu que es tracta realment d’un màxim.

Solució PAU CAT TEC JUNY 2010 1.3

15. Considereu tots els prismes rectes de base quadrada amb un volum V fixat.

Anomeneu x el costat de la base del prisma i y la seva altura.

a) Trobeu l’expressió del volum i de l’àrea total del prisma en funció de les

variables x i y.

b) Comproveu que el que té àrea total mínima és en realitat un cub.

Solució PAU CAT TEC SET 2010 2.3

16. De tots els triangles rectangles d’hipotenusa 10 cm, trobeu la longitud dels catets

del triangle que té el perímetre màxim. Comproveu que la solució trobada

correspongui realment al perímetre màxim. Solució PAU CAT TEC JUNY 2008 5.6

17. Un magatzem té forma de prisma recte de base quadrada i un volum de 768 m

3. Se

sap que la pèrdua de calor a través de les parets laterals val 100 unitats per m2, mentre

que a través del sostre és de 300 unitats per m2. La pèrdua pel sòl és molt petita i es pot

considerar nul·la. Calculeu les dimensions del magatzem perquè la pèrdua de calor total

sigui mínima. Solució PAU CAT TEC JUNY 2007 2.5

18. Considereu la recta d’equació 2

1

2

2:

zyxr .

a) Expresseu el quadrat de la distància d’un punt qualsevol (x,y,z) de la recta al punt

P= (1, 2, 5) com una funció de la coordenada x.

b) Trobeu quin valor de x fa mínima aquesta funció, deduïu quin punt Q de la recta és el

més proper a P i calculeu la distància del punt a la recta.

c) Escriviu l’equació de la recta que passa per P i Q i comproveu que és perpendicular a

r.

Solució PAU CAT TEC JUNY 2007 1.5

19. Considereu la funció 23 xy i un punt de la seva gràfica, M, situat en el primer

quadrant (x ≥ 0, y ≥ 0). Si pel punt M tracem paral·leles als eixos de coordenades, la

seva intersecció amb OX i OY determina dos punts, A i B, respectivament.

a) Feu un gràfic dels elements del problema.

b) Trobeu les coordenades del punt M que fa que el rectangle OAMB tingui l’àrea

màxima.

Solució PAU CAT TEC SET 2005 3.5

20. La recta tangent a la paràbola y = 3 – x

2 en un punt M situat dins del primer

quadrant (x > 0, y > 0), talla l’eix OX en el punt A i l’eix OY en el punt B.

a) Feu un gràfic dels elements del problema.

b) Trobeu les coordenades del punt M que fan que el triangle OAB tingui l’àrea

mínima.

Solució PAU CAT TEC JUNY 2005 1.5

21. Donats la funció xxf )( i el punt A(2, 0) situat sobre l’eix de les abscisses:

a) Trobeu la funció que expressa la distància del punt A a un punt qualsevol de la

gràfica de la funció.

b) Trobeu les coordenades del punt de la gràfica de f(x) més proper a A.

Solució PAU CAT TEC JUNY 2004 1.6

22. Volem unir el punt M situat en un costat d’un carrer de 3 m d’amplada amb el punt

N situat a l’altre costat i 9 m més avall mitjançant dos cables rectes, un des de M fins a

un punt P situat a l’altre costat del carrer i un altre des de P fins a N seguint el mateix

costat del carrer, segons l’esquema següent:

El cost de la instal·lació del cable MP és de 12 € per metre i del cable PN de 6 € per

metre.

Quin punt P haurem d’escollir de manera que la connexió de M amb N sigui tan

econòmica com sigui possible? Quin serà aquest cost mínim?

Solució PAU CAT TEC JUNY 2003 2.5

23. Determineu quin és el punt de la gràfica de xy (és a dir, de la forma xx, ),

que és més a prop del punt P = (4, 0).

Solució PAU CAT TEC JUNY 2003 5.1

24. Un camp té forma de trapezi rectangle, de bases 240 m i 400 m, i el costat

perpendicular a les bases també de 400 m. Es vol partir tal com indica la figura per fer

dos camps rectangulars C1 i C2. Anomenem x i y els catets d’un dels triangles

rectangles que es formen.

a) Comproveu que xy2

5 .

b) Utilitzant la igualtat anterior, escriviu la suma de les àrees dels dos camps en funció

de x.

c) El camp C1 es vol sembrar amb blat de moro i el camp C2 amb blat. Amb el blat de

moro s’obté un benefici de 0,12 € per m2 i amb el blat un benefici de 0,10 € per m

2.

Determineu les mides de cada un dels camps per obtenir el benefici màxim.

Solució PAU CAT TEC SET 2003 3.5

25. S’ha de construir un gran dipòsit cilíndric de 81 m

3 de volum. La superfície lateral

ha de ser construïda amb un material que costa 30€ el m2 i les dues bases amb un

material que costa 45€ el m2.

a) Determineu la relació que hi haurà entre el radi r de les bases circulars i l'altura h del

cilindre, i doneu el cost C(r) del material necessari per construir aquest dipòsit en funció

de r.

b) Quines dimensions (radi i altura) ha de tenir el dipòsit perquè el cost del material

necessari per construir-lo sigui el mínim possible?

c) Quin serà, en aquest cas, el cost del material? Solució PAU CAT TEC JUNY 2002 3.5

26. Suposem que el Sol es troba a l'origen d'un sistema de coordenades i que un cometa

segueix una trajectòria donada per la paràbola 21 xy , tal com es veu a la figura

següent:

a) Quin és el punt en què el cometa es troba més proper al Sol?

b) Quant val en aquest cas la distància del Sol al cometa?

c) Hi ha algun punt en què el cometa es trobi a la màxima distància del Sol?

d) Hi ha algun punt en què la distància entre el Sol i el cometa sigui un màxim local o

relatiu?

Nota: Teniu present que la distància entre dos punts és màxima o mínima quan el

quadrat de la distància és màxim o mínim. Solució PAU CAT TEC JUNY 2002 1.5

27. La riba d'un tram de riu descriu la corba 2

4

1xy , per a x entre –3 i 3, i en el

punt A = (0, 4) hi ha un poble, tal com es pot veure en l'esquema següent:

a) Expresseu la distància des d'un punt qualsevol d'aquesta vora del riu fins al

poble, en funció de l'abscissa x.

b) Quin és el punt de la vora d'aquest tram de riu que és més lluny del poble?

c) Hi ha algun punt de la vora del riu a una distància del poble inferior a 2? Solució PAU CAT TEC SET 2001 4.5

28. El costat desigual d'un triangle isòsceles mesura 12 m, i l'altura sobre aquest costat

és de 5 m.

a) Donat un punt arbitrari sobre aquesta altura, obtingueu una expressió de la suma de

les distàncies d'aquest punt a cada un dels vèrtexs del triangle.

b) Determineu els punts sobre l'altura que compleixen que la suma de les distàncies als

tres vèrtexs del triangle sigui màxima i els punts per als quals sigui mínima.

Solució PAU CAT TEC SET 2000 2.5

29. Un terreny té forma de triangle rectangle, els catets mesuren AB = 60 m i AC = 45

m. En aquest terreny es pot construir una casa de planta rectangular com indica la part

ombrejada de la figura següent:

Voleu vendre aquest terreny i us paguen 5.000 pessetes per cada metre quadrat no

edificable i 25.000 pessetes per cada metre quadrat edificable.

a) Determineu la relació que hi ha entre l'amplada x i la profunditat y del rectangle que

determina la part edificable.

b) Determineu l'expressió que dóna el valor del terreny en funció de l'amplada x del

rectangle edificable.

c) Quines són les dimensions de la part edificable que ens permeten obtenir un valor

màxim per a aquest terreny?

d) Quin és aquest valor màxim?

Solució PAU CAT TEC JUNY 2000 1.5

30. Volem penjar un llum a una certa distància del sostre d'una habitació. Per fer-ho,

agafem una corda, hi lliguem el llum i la clavem pels extrems en dos punts del sostre

separats per una distància de 140 centímetres, de manera que els angles entre la corda i

el sostre són de 40º i 60º a cada un dels extrems.

a) Quina serà la longitud total de la corda?

b) A quina distància del sostre quedarà el llum?

PAU CAT TEC SET 1999 5.5

31. Considereu els rectangles del pla, els vèrtexs A, B, C i D dels quals compleixen les

condicions següents: a) A és l’origen de coordenades; b) B és a sobre del semieix de les

y>0; c) C és a sobre del semieix de les x>0; d) D és a sobre de la recta d’equació

2x+y=1, tal com es veu en la figura següent:

D’entre tots aquests rectangles, trobeu l’àrea del que la té màxima.

PAU CAT TEC SET 1998 5.2

32. Una via de tren passa a 2 km del poble A i a 3 km del poble B, de manera que el

tram de via comprès entre ambdós pobles és de 5 km, tal com s’indica en la figura.

Volem construir una nova estació ferroviària i una carretera formada per dos trams

rectes que uneix A amb B passant per l’estació. En quin punt del tram de via hem de

col·locar l’estació si volem que el recorregut de A a B passant per la nova carretera

sigui mínim? Quina serà la longitud total de la nova carretera?

Solució PAU CAT TEC JUNY 1998 3.6

33. Trobeu els costats d’un rectangle d’àrea màxima inscrit a l’el·lipse d’equació

1416

22

yx

,

tal com s’indica en la figura següent:

PAU CAT TEC JUNY 1998 6.1

11.4 Problemes PAU CCSS d’optimització.

1. Disposem de 48 cm

2 de material per a fabricar una capsa de base quadrada, sense

tapa. Calculeu les dimensions de la capsa de volum més gran que podem construir en

aquestes condicions. Quin serà el volum de la capsa?

Solució PAU CAT CCSS SET 2012 4.5

2. Un triangle té els vèrtexs O(0, 0), A(6, 0) i B(0, 3).

a) Dibuixeu-lo i escriviu l’equació de la recta que conté el segment AB.

b) Considerem un punt P situat sobre el segment AB , i dibuixem el rectangle que té per

diagonal OP i dos costats sobre els eixos de coordenades. Determineu les coordenades

de P que fan màxima l’àrea del rectangle.

Solució PAU CAT CCSS JUNY 2012 3.4

3. Determineu dos nombres enters positius que sumin 25, de manera que el doble del

quadrat del primer sumat amb el triple del quadrat del segon doni el mínim valor

possible.

Solució PAU CAT CCSS SET 2011 2.4

4. Volem construir el marc d’una finestra rectangular de 100 dm2 de superfície. El cost

de cada decímetre de marc horitzontal és de 6€, mentre que el de cada decímetre de

marc vertical és de 24€. Calculeu les dimensions de la finestra perquè el marc ens surti

tan barat com sigui possible.

Solució PAU CAT CCSS JUNY 2010 5.2

5. Es vol construir una piscina que tingui per base un rectangle i dos semicercles adjunts

tal com s’indica a la figura següent:

Sabent que el perímetre de la piscina ha de ser de 30 m, calculeu les seves dimensions

per tal que la superfície sigui màxima.

Solució PAU CAT CCSS JUNY 2004 1.6

6. Disposem de material per poder impermeabilitzar 200 m2 de superfície. Volem fer

una bassa de base rectangular en què la llargada mesuri el triple que l’amplada i amb la

profunditat adequada per gastar tot el material. Interessa que el volum d’aigua que

càpiga a la bassa sigui màxim.

a) Escriviu la relació que hi ha entre l’altura i el costat petit de la base de la bassa.

b) Escriviu la funció que dóna la capacitat de la bassa en funció del costat petit de la

base.

c) Calculeu les dimensions de la bassa perquè la capacitat sigui màxima. (Els resultats

s’han de precisar fins als centímetres.)

d) Determineu-ne el volum.

Solució PAU CAT CCSS SET 2003 3.5

7. Amb un llistó de fusta de 300 cm de llarg volem fabricar el marc d’un quadre.

a) Determineu la relació que hi ha entre la base i l’alçada del marc.

b) Determineu la funció que expressa la superfície del quadre en termes de la base del

marc.

c) Feu un gràfic d’aquesta funció on es posin de manifest els seus intervals de

creixement i decreixement i els extrems relatius.

d) Trobeu les dimensions del marc que fan màxima la superfície del quadre. Trobeu el

valor de la superfície.

Solució PAU CAT CCSS JUNY 2003 5.6

12 Regles de L’Hôpital.

Sota el nom de regles de L’Hôpital s’engloben una sèrie de resultats que, mitjançant

derivació, permeten el tractament d’una classe àmplia de límits. Donarem l’enunciat

bàsic de L’Hôpital i comentarem les extensions sobre exemples.

12.1 Regla de L’Hôpital fonamental.

12.1.1 Teorema. Teorema de L’Hôpital.

Siguin )(xf i )(xg dues funcions }{,:, ahahagf tals que:

a) )(xf i )(xg són derivables

b) )(' xg no s’anul·la

c) 0)(lim

xfax

, 0)(lim

xgax

Llavors si Lxg

xfL

xg

xf

axax

)(

)(lim

)('

)('lim

Demostració.

Demostrarem que, si )(xf i )(xg són contínues en ba, i derivables en ),( ba . Si

0)()( agaf , baxxg ,0)(' i Lxg

xf

ax

)('

)('lim , llavors L

xg

xf

ax

)(

)(lim .

Considerem la funció

)()()()()( xgbfxfbgxh

que compleix les hipòtesis del teorema de Rolle i, per tant, existeix ),( bac tal que

0)(')()(')()(' cgbfcfbgch

Ho podem posar

)('

)('

)(

)(

cg

cf

bg

bf

perquè 0)(' cg per hipòtesi, i 0)( bg perquè pel teorema de Lagrange

0))((')()()( abdgagbgbg per a algun ),( bad .

Quan ab , ac perquè bca , i

Lcg

cf

bg

bf

acab

)('

)('lim

)(

)(lim

Observacions:

a) Adoneu-vos que és un resultat aplicable, en les hipòtesis adequades, al cas

d’indeterminació 0

0, però la podem utilitzar en qualsevol tipus d’indeterminació: fem

transformacions en el límit fins que aconseguim una indeterminació 0

0.

b) El teorema també es pot aplicar al cas d’indeterminació

.

c) El teorema de L’Hôpital és vàlid si a és un nombre real, o .

d) Noteu que l’enunciat diu “ " i no "" . És a dir pot ser que )(

)(lim

xg

xf

ax existeixi i en

canvi no el podem trobar per aquest procediment perquè no existeix )('

)('lim

xg

xf

ax.

Calcula els següents límits aplicant Hopital: a) x

x

x

sinlim

0 b)

x

x

x ln

1lim

2

1

a) x

x

x

sinlim

0

1)(')(

cos)('sin)(

xgxxg

xxfxxf

i clarament aquestes funcions compleixen les condicions del teorema de L’Hôpital.

Per tant 10coscoslim1

coslim

sinlim

000

x

x

x

x

xxx

b) x

x

x ln

1lim

2

1

xxgxxg

xxfxxf

1)('ln)(

2)('1)( 2

2122lim/1

2lim

ln

1lim 22

11

2

1

x

x

x

x

x

xxx

12.2 Altres indeterminacions que es poden resoldre amb L’Hôpital.

Totes les altres indeterminacions es poden reduir a les del tipus 0

0 i

, a les quals es

pot aplicar el teorema de L’Hôpital. Ho veurem amb exemples.

12.2.1 Indeterminació

1

1

ln

1lim

1 xxx

És un cas d’indeterminació .

Reduïm a denominador comú:

xx

xx

xx

x

xx

x

xx ln)1(

ln1

ln)1(

ln

ln)1(

1

1

1

ln

1

que queda una indeterminació 0

0 i podem aplicar el teorema de L’Hôpital:

xxxxgxxxg

xxfxxxf

1)1(ln)('ln)1()(

11)('ln1)(

1ln

1

1ln

11

)('

)('

xxx

x

x

xx

x

xg

xf

Continua la indeterminació 0

0, i tornem a aplicar el teorema de L’Hôpital:

2ln11

ln1)('1ln)(

1)('1)(

xx

xxxgxxxxg

xfxxf

2

1

21ln

1

2ln

1

)('

)(' 1

x

xxg

xf

12.2.2 Indeterminació 0

22lim xe x

x

És un cas d’indeterminació 0 .

xx

x

x e

xxe

2

222 limlim

que és una indeterminació

. Apliquem L’Hôpital dues vegades:

02

1limlim

2

2limlim

2222

2

xxxxxxxx ee

x

e

x

e

x

12.3 Quatre exercicis de Hôpital resolts.

Calcula els següents límits:

1. 2

sin

0lim

x

ee xx

x

PAU ANDALUCÍA JUNY 2010

2.

1

2

ln

1lim

21 xxx

PAU ANDALUCÍA JUNY 2009

3. 2

2

1 )1(

)(lnlim

x

xx

x

PAU ANDALUCÍA JUNY 2009

4. x

xex /1

0lim

PAU ANDALUCÍA JUNY 2008

1.

0

0lim

2

sin

0

x

ee xx

x és indeterminació. Apliquem la Regla de L’Hôpital:

xeexfeexf xxxx cos)(')( sinsin

xxgxxg 2)(')( 2

0

0

2

coslim

sin

0

x

xee xx

x. Apliquem de nou la Regla De L’Hôpital:

0)01(1sincos)('cos)(0

sin2sinsin x

xxxxx xexeexfxeexf

22)('2)(0

x

xgxxg

Per tant el límit és 02

0

2.

1

2

ln

1lim

21 xxx indeterminació.

Fem la resta passant a comú denominador per obtenir la fracció equivalent:

xx

xx

xx ln)1(

ln2)1(

1

2

ln

12

2

2

02

2)('ln2)1()(1

2

x

xxxfxxxf

0ln21

)('ln)1()(12

2

x

xxx

xxgxxxg

Continuem tenint una indeterminació 0/0, tornem a aplicar la Regla de Hôpital:

4222

2)('2

2)(1

2

x

xxf

xxxf

42022ln21

)('ln21

)(1

2

22

x

xx

xxgxx

x

xxg

Per tant, el límit és 14

4

Observació: També podríem haver simplificat abans:

xxx

x

x

xxx

x

x

xxx

x

xx

ln21

22

ln21

22

ln21

22

22

2

22

2

2

3.

0

0

)1(

)(lnlim

2

2

1

x

xx

x és una indeterminació, i apliquem la Regla de Hôpital:

0ln2)(ln1)(')(ln)(1

22

x

xxxfxxxf

0)1(2)(')1()(1

2

x

xxgxxg

De nou ens trobem davant d’una indeterminació 0/0, i tornem a aplicar Hôpital :

21

22ln22ln2)('ln2)(ln1)(

12

x

x

x

xx

xxfxxxf

22)(')1(2)(1

x

xgxxg

Per tant el límit és 12

2

4.

El comportament de la funció és diferent per l’esquerra i per la dreta, els hem d’estudiar

per separat:

00000lim 0/1/1

0

eeex x

x , no és una indeterminació.

x

eex

x

x

x

x /1limlim

/1

0

/1

0 és una indeterminació i apliquem Hôpital:

0/1

2

/1

2

2

/1

2

/1

1

1

)('

)('

1)('

1)(

1)(')(

xx

xxx

e

x

ex

xg

xf

xxg

xxg

ex

xfexf

El límit és, doncs, .

13 Integral indefinida, definida i àrea.

13.1 Integral indefinida.

13.1.1 Primitiva d’una funció. Integral indefinida. Antiderivació.

La integració d’una funció és el pas invers de la derivació.

Anomenem funció primitiva de )(xf una altra funció )(xF que compleix que

)()(' xfxF .

Observació important: La primitiva d’una funció no és única:

Si )(xF és una funció primitiva de )(xf , qualsevol funció de la forma CxF )( , amb

C també és una funció primitiva de )(xf .

Per expressar la integració d’una funció s’utilitza un símbol ∫, anteposat a la funció, i el

símbol dx (denominat diferencial de x) després de la funció, és a dir, la integral

indefinida d’una funció s’expressa

CxFdxxf )()(

essent F(x) una primitiva de f(x) i c una constant, és a dir, un nombre qualsevol.

La integral indefinida d’una funció )(xf és el conjunt de totes les seves primitives, i es

representa per dxxf )( . Es llegeix “la integral indefinida de f(x) diferencial de x”.

)()(')()( xfxgdxxfxg

Per tant, si )(xF és una primitiva de )(xf , llavors

CxFdxxf )()(

13.1.2 Integració de monomis.

Cxkdxk

)1(1

1

nsiCn

xdxx

nn

13.1.3 Integral d’una suma.

La integral d’una suma és la suma d’integrals

dxxgdxxfdxxgxf )()()()(

13.1.4 Integral d’un producte per una constant.

La integral del producte d’una funció per una constant és la constant per la integral de la

funció.

dxxfkdxxfk )()(

13.1.5 Integració de polinomis.

De les propietats anteriors podem deduir la fórmula de la integral d’un polinomi:

Cxaxa

n

xa

n

xadxxfaxaxaxaxf

n

n

n

nn

n

n

n

01

1

101

1

12

...1

)(...)(

13.1.6 Integració de radicals.

La fórmula

)1(1

1

nsiCn

xdxx

nn és vàlida per a exponents fraccionaris, i és

molt útil per a integrar arrels.

Per exemple:

Cx

Cx

Cx

Cx

dxxdxx

3

2

3

2

2/312/1

32/32/312/12/1

Cx

Cx

Cx

Cx

dxxdxx

4

3

4

3

3/413/1

3 43/43/413/13/13

13.1.3 Problemes PAU TEC d’antiderivació amb funcions polinòmiques.

1. De les funcions f (x), f ′(x), g (x) i g′(x), en coneixem els valors següents:

a) De la funció f(x) sabem també que el pendent de la recta tangent a un punt d’abscissa

x és 4x3 – 9x

2 – 2x + 1. Trobeu f (x).

b) Calculeu (g º f )′(1). Solució PAU CAT TEC SET 2017 2.4

2. Sabem que una funció f té per derivada la funció f '(x) = (3x – 2)2(x – 2).

a) Calculeu els valors de x en què la funció f té un màxim relatiu, un mínim relatiu o un

punt d’inflexió, i indiqueu en cada cas de què es tracta.

b) Determineu la funció f sabent que s’anuŀla en el punt d’abscissa x = 2.

Solució PAU CAT TEC SET 5.4

3. La funció derivada F’(x) d’una funció contínua IRIRF : que passa per l’origen és

una funció a trossos formada per les semirectes del dibuix.

Escriviu l’expressió de la funció F(x) com una funció a trossos.

Solució PAU CAT TEC SET 2007 3.4

13.2 Integral definida.

13.2.1 Definició. Integral definida.

La integral definida neix de la necessitat de calcular l’àrea tancada per una funció i l’eix

X en cert interval. Aquesta àrea es pot aproximar sumant certs rectangles, la base dels

quals sigui constant, i l’altura dels quals sigui el valor de la funció en certs punts

escollits convenientment. El límit d’aquest càlcul quan la base d’aquests rectangles

tendeix a 0 és igual a la integral definida d’aquesta funció en aquest interval, és a dir,

l’àrea que s’estava buscant.

La integral definida d’una funció f(x) entre els extrems a i b es defineix com el límit de

la suma dels rectangles de la imatge, quan la seva base tendeix a 0:

n

i

iii

b

a xxxxxfdxxf

ii 0

10

))((lim)(1

Nota: La definició de integral definida com a suma d’àrees de rectangles de base més i

més petita justifica l’origen del símbol integral és una S allargada, indicant que es tracta

d’un sumatori, mentre que l’origen del símbol diferencial, dx, prové del fet que es tracta

de diferències de x (prenent la inicial de "diferència" juntament amb la x, resulta

precisament dx).

Molt important: Aquesta definició d’integral com a límit d’un sumatori no es farà

servir mai en el nostre context. Nosaltres utilitzarem sempre la fórmula de Barrow, que

lliga integral definida i antiderivada.

13.2.2 Teorema fonamental del càlcul. Regla de Barrow.

Teorema. Teorema del valor mitjà per a la integral.

Si )(xf és integrable en ba, , existeix un Mm, tal que

b

aabdxxf )()(

sent m i M l’ínfim i el suprem de la funció en l’interval ba, .

Demostració.

Clarament baxMxfm ,)( per tant

b

a

b

a

b

adxMdxxfdxm )(

és a dir,

)()()( abMdxxfabmb

a

dividint per )( ab

Mdxxfab

mb

a

)(

1

Prenent

b

adxxf

ab)(

1 tenim que Mm , és a dir, Mm, , i clarament es

comprova que b

aabdxxf )()( .

Aquest teorema ens diu que l’àrea determinada per la funció és entre les àrees dels

rectangles de base ab i altura m i M.

Corol·lari.

Si )(xf és contínua en ba, , existeix un bac , tal que

b

aabcfdxxf )()()(

Demostració.

La imatge per f de l’interval ba, és Mm, i, per tant, existeix un bac , tal que

)(cf perquè Mm, .

És a dir,

b

acfdxxf

ab)()(

1 amb bac ,

Corol·lari.

Si )(xf és integrable en ba, , la funció x

adttfxF )()( és contínua en ba, .

Demostració.

Hem de veure que

0)()(lim0

xFhxFh

hx

hh

x

a

hx

ahhdttfdttfdttfxFhxF )(lim)()(lim)()(lim

000

Aplicant el Teorema del valor mitjà

0lim)(lim)(lim000

hx

h hhhhxhxdttf

Teorema fonamental del càlcul.

Si f és contínua en ba, , la funció dttfxFx

a )()( és una primitiva de f en ba, ,

és a dir, baxxfxF ,)()('

Demostració.

Per la demostració de derivada tenim que:

h

dttfdttf

h

xFhxFxF

x

a

hx

a

hh

)()(lim

)()(lim)('

00

Observem que hem hagut de canviar a la lletra t dintre de les integrals perquè la lletra x

ja l’estem utilitzant.

Per la propietat ---- de la integral definida:

dttfdttfdttfdttfdttfdttfhx

x

x

a

hx

x

x

a

x

a

hx

a

)()()()()()(

Per tant, h

dttfxF

hx

x

h

)(lim)(

0

Aplicant ara el teorema del valor mitjà, resulta que existeix un hxxc , tal que

hcfxhxcfdttfhx

x)())(()(

I per tant

)(lim)(

lim)(00

cfh

hcfxF

hh amb hxxc ,

Si 0h i hxxc , llavors )()( xfcf (recordem que per hipòtesi f és contínua)

d’on deduïm que )()( xfxF .

Corol·lari. Regla de Barrow.

Si f és contínua en ba, i G és una primitiva de f en ba, , llavors

)()()( aGbGdxxfb

a

Demostració.

Sabem que dues primitives d’una mateixa funció difereixen en una constant.

Acabem de veure que dttfxFx

a )()( és una primitiva de f, i per tant

kxGxF )()( per a certa constant k.

En particular, per a ax , 0)()()( dttfaFkaGa

a i per tant )(aGk , és a

dir, )()()( aGxGxF .

Prenem ara bx , )()()( aGbGbF , és a dir, )()()( aGbGdttfb

a , tal i com

volíem veure.

Exemple:

8

5

338

5

32

3

5

3

8

3

xdxx 129, ja que una primitiva de x

2 és

3

3x.

Problemes PAU amb integrals definides.

1. De la funció polinòmica P(x) = x3 + ax

2 + bx + 2 sabem que

- té un extrem relatiu en el punt d’abscissa x = –3;

- la integral definida en l’interval [0, 1] val 4

5 .

Calculeu el valor dels paràmetres a i b.

Solució PAU CAT TEC SET 2013 1.2

2. Sigui a

dxxaaf/1

0

22 )()( per a>0,

a) Comproveu que aa

af 33

1)( .

b) Calculeu el valor del paràmetre a perquè la funció f (a) tingui un mínim relatiu.

Solució PAU CAT TEC SET 2011 2.6

3. Considereu la funció y=f(x) definida per a x[0,5] que apareix dibuixada a la figura

adjunta.

a) Quina és l’expressió de la seva funció derivada quan existeix?

b) Calculeu 3

0)( dxxf .

Solució PAU CAT TEC JUNY 2006 3.2

4. El polinomi p(x) = x2 + ax + b s'anul·la per a x=2 i compleix

2

04)( dxxp . Calculeu

raonadament a i b.

Solució PAU CAT TEC JUNY 2000 1.5

13.3 Determinació d’àrees amb funcions polinòmiques.

13.3.1 Àrea entre dues funcions (Indicant els extrems d’integració)

Des d’un punt purament teòric, tota la relació entre integració i àrea es redueix al

següent teorema:

Si )(xf i )(xg són dues funcions integrables en un interval ba, , llavors l’àrea

determinada entre les dues funcions és igual a la integral del valor absolut de la seva

diferència:

Àrea entre )(xf i )(xg = b

adxxgxf )()(

Però aquesta fórmula no ens serveix perquè no disposem d’una fórmula que ens doni

directament la integral d’una funció amb valor absolut. Per tant, a la pràctica, hem de

considerar per separat els següents cinc casos:

CAS 1. Àrea determinada per la gràfica d’una funció positiva i l’eix d’abscisses.

Si una funció )(xf és sempre positiva en un interval ba, , llavors l’àrea que

determina la seva gràfica i l’eix d’abscisses és igual al valor de la integral definida:

b

adxxfÀrea )(

Calcular l’àrea de la funció 345)( 23 xxxxf a l’interval 1,3

Aquesta funció és sempre positiva en aquest interval, per tant l’àrea coincidirà amb la

integral definida:

Cxxxx

Cxxxx

dxxxxxF 323

5

43

24

3

5

4345)( 2

3423423

67.223

68

4

63

12

83)3()1()(

1

3

FFdxxfÀrea

12

831312

3

15

4

1)1( 2

34

F

4

63)3(3)3(2

3

)3(5

4

)3()3( 2

34

F

CAS 2. Àrea determinada per la gràfica d’una funció negativa i l’eix d’abscisses.

La integral d’una funció negativa és un nombre negatiu, per tant si una funció )(xf és

sempre negativa en un interval ba, , llavors la integral serà negativa, per tant, li

traiem el signe, o dit d’una altra manera, determinem el seu valor absolut:

Si 0)(xf b

adxxfÀrea )(

Calcular l’àrea de la funció 13)( 2 xxxf a l’interval 2,1

Observem que aquesta funció es estrictament negativa en aquest interval, per tant

calculem la seva integral definida, que ja sabem que donarà un valor negatiu, i prenem

el seu valor absolut (és a dir, eliminem el signe):

Cxxx

dxxxxF 2

3

313)(

232

3

4)2(

F ,

6

1)1(

F

6

7

6

7

6

1

3

4)1()2()(

2

1

FFdxxfÀrea

CAS 3. Àrea determinada per la gràfica d’una funció que canvia de signe i l’eix

d’abscisses.

Si una funció )(xf és negativa, llavors la seva integral també serà negativa, per tant si

una funció canvia de signe dintre de l’interval ba, haurem de calcular les integrals

per separat i sumar sempre els valors positius, és a dir, en valor absolut.

Calcular l’àrea determinada per la gràfica de la funció 652)( 23 xxxxf i l’eix

d’abscisses a l’interval 2,1

La gràfica d’aquesta funció és positiva entre –1 i 1 i és negativa entre 1 i 2, per tant hem

de calcular les integrals per separat.

*)1()2()1()1()()(2

1

1

1 FFFFdxxfdxxfÀrea

Cxxxx

dxxxxxF 62

53

2

4652)(

23423

12

91)1(

F

12

37)1( F

3

2)2( F

42.1812

221

12

29

2

32

12

37

3

2

12

91

12

37*

CAS 4. Àrea entre dues gràfiques que no es creuen.

Si )()( xgxf en tot l’interval ba , , llavors l’àrea determinada entre ambdues

gràfiques és igual a la integral de la diferència de les funcions (la superior menys la

inferior):

b

adxxgxfÀrea )()(

Calcular l’àrea entre les funcions 34)( 2 xxxf i 22)( 3 xxxg a l’interval

1,1

Observem que en aquest interval )()( xgxf , i per tant

*)()(1

1 dxxgxfÀrea

52)22(34)()( 2332 xxxxxxxxgxf

Cxxxx

Cxxxx

dxxxxxF

534

52

2

3452)( 2

3423423

33.93

28

12

47

12

65)1()1(*

FF

CAS 5. Àrea entre dues gràfiques que es creuen.

Si )(xf i )(xg es creuen dintre de l’interval ba , , és a dir, )()( xgxf canvia de

signe, hem d’integrar per separat i sumar les integrals eliminant el signe, és a dir,

calculem el valor absolut dels resultats.

Calcular l’àrea determinada entre les gràfiques de les funcions

10112)( 23 xxxxf i xxxg 2)( a l’interval 2,2

Observem que les dues gràfiques es creuen al punt x=-1, per tant hem d’integrar per

separat:

*)1()2()2()1()()()()(2

1

1

2

FFFFdxxgxfdxxgxfÀrea

1011)()( 23 xxxxgxf

Cxxxx

dxxgxfxF 10534

)()()( 234

3

4)2( F ,

12

59)1( F ,

3

100)2(

F

83.416

251

4

153

12

43

4

153

12

43

12

59

3

100

3

4

12

59*

83.416

251

4

153

12

43)()()()(

2

1

1

2

dxxgxfdxxgxfÀrea

Determinació de l’àrea amb una funció polinomial (Exemple resolt)

a) Determina la integral indefinida de 13)( 2 xxxf

b) Determina l’àrea entre la funció anterior, l’eix X i les rectes x = 0 i x = 2.

Solució:

a)

Cxxx

dxdxxdxxdxxxxF 2

3

31313)(

2322

b) Determinem els punts de tall de la funció amb l’eix X:

x

xxxxf 013)( 2

La funció, tal i com es pot veure a la gràfica superior, a l’interval (0,2) talla l’eix X al

punt 2x , i observem que 44.0)2.0( f i 1)2( f , és a dir, passa de positiu a

negatiu, per tant

1.696731.51503-0.181695

)382.0()2()0()382.0()()(2

382.0

382.0

0

FFFFdxxfdxxfA

13.3.2 Àrea entre dues funcions (sense indicar els extrems d’integració)

Si no ens donen els extrems d’integració a i b, hem de suposar que les dues gràfiques

delimiten una superfície tancada entre totes dues. Els extrems d’integració seran, doncs,

dos dels seus punts d’intersecció, que haurem de calcular.

Determineu l’àrea entre les gràfiques de xxxxf 103)( 23 i xxxg 2)( 2

En primer lloc determinem els seus punts d’intersecció :

2,2,00)4(30123

0210321032

103

223

223223

2

23

xxxxx

xxxxxxxxxxxx

xxx

En segon lloc, representem aproximadament les dues gràfiques :

Veiem que les dues gràfiques es creuen, i per tant hem de determinar l’àrea amb dues

integrals:

xxxxxxxxgxf 123)2(103)()( 3223

*)0()2()2()0()()()()(2

0

0

2 FFFFdxxgxfdxxgxf

2424

3 64

3

2

12

4

3123)()()( x

xxxdxxxdxxgxfxF

0064

03)0( 2

4

F

12264

23)2( 2

4

F

12)2(64

)2(3)2( 2

4

F

2412121212012)12(0*

Problemes PAU amb àrea entre dues funcions.

1. Considereu les funcions f(x) = x2 i

xxg

1)( , i la recta x = e.

a) Feu un esbós de la regió delimitada per les seves gràfiques i l’eix de les abscisses.

Calculeu les coordenades del punt de tall de y = f(x) amb y = g(x).

b) Calculeu l’àrea de la regió descrita en l’apartat anterior.

PAU CAT TEC JUNY 2019 1.6 (Solució: "Compendium Tec", Pàg. 536)

2. Considereu les rectes y = x i y = 2x, i la paràbola y = x2.

a) Calculeu els punts d’intersecció entre les gràfiques de les diferents funcions i feu un

esbós de la regió delimitada per les gràfiques.

b) Calculeu l’àrea de la regió de l’apartat anterior.

PAU CAT TEC SET 2019 5.1 (Solució: "Compendium Tec", Pàg. 541)

3. Siguin les funcions f(x) = x2 – 1 i g(x) = 3 – x

2.

a) Feu un esbós de les gràfiques de les paràboles y = f(x) i y = g(x) en un mateix sistema

d’eixos cartesians i trobeu els punts de tall amb l’eix de les abscisses, els vèrtexs i els

punts de tall entre les dues gràfiques.

b) Calculeu l’àrea de la regió del semiplà y ≥ 0 compresa entre les gràfiques de f(x) i

g(x).

Solució PAU CAT TEC SET 2018 3.6

4. Responeu a les qüestions següents:

a) Determineu l’equació de la recta tangent a la corba y=x3 en el punt d’abscissa x=2.

b) Calculeu l’àrea de la regió plana finita limitada per la corba y=x3 i la recta y=3x – 2.

Solució PAU CAT TEC JUNY 2015 2.3

5. Calculeu l’àrea de la regió del pla limitada en el primer quadrant per les gràfiques de

les funcions y = x2, y = 4x

2 i y = 9.

Solució PAU CAT TEC JUNY 2014 3.4

6. Calculeu l’àrea del recinte limitat per les corbes d’equació f(x) = x2 −x +2 i

g(x) = 5 −3x. Solució PAU CAT TEC JUNY 2011 4.1

7. Considereu les corbes y = 4x–x2 i y = x

2–6.

a) Trobeu-ne els punts d’intersecció.

b) Representeu les dues corbes en una mateixa gràfica, on es vegi clarament el recinte

que limiten entre elles.

c) Trobeu l’àrea d’aquest recinte limitat per les dues corbes.

Solució PAU CAT TEC JUNY 2009 3.2

8. Busqueu els extrems relatius i els punts de tall amb els eixos, i feu una representació

aproximada de la corba d’equació y=x4–x

2. A continuació, calculeu l’àrea del recinte

tancat per aquesta corba i l’eix d’abscisses.

Solució PAU CAT TEC 2007 1.3

9. Considereu la paràbola d’equació y = x2 +2x –3.

a) Calculeu les equacions de les rectes tangents a la paràbola en els punts d’abscissa x=

–1 i x= 1.

b) Calculant el mínim de la funció y = x2 +2x –3, trobeu el vèrtex de la paràbola.

c) Trobeu les interseccions de la paràbola amb els eixos i feu una representació gràfica

de la paràbola i de les tangents obtingudes al primer apartat.

d) Calculeu l’àrea compresa entre la paràbola i les rectes tangents.

Solució PAU CAT TEC SET 2006 4.5

10. Considereu la funció f(x) = 4x – x2.

a) Calculeu l’equació de les rectes tangents a la gràfica de f en els punts d’abscisses x =

0 i x = 4.

b) Feu un gràfic dels elements del problema.

c) Calculeu l’àrea compresa entre la gràfica de f i les rectes tangents que heu trobat a

l’apartat a).

Solució PAU CAT TEC JUNY 2005 1.6

11. Determineu l’àrea finita de la regió del pla compresa entre les dues paràboles

y = –x2 +4x +1 i y = x

2 –6x +9.

Solució PAU CAT CCSS JUNY 2003 2.3

12. Considereu la corba d'equació f(x) = x3 – x.

a) Calculeu els punts en què la gràfica de f(x) talla l'eix d'abscisses i expliqueu

raonadament on f(x) és positiva i on és negativa.

b) Trobeu l'àrea del recinte limitat per la part positiva de la gràfica de f(x) i el semieix

negatiu d'abscisses. Solució PAU CAT CCSS SET 2002 1.1

13. Sabeu que la gràfica de la funció f(x) passa pel punt (1, –4) i que la seva funció

derivada és f'(x) = 2x – 2.

a) Determineu l'expressió de f(x).

b) Calculeu l'àrea de la regió limitada per la gràfica de f(x) i l'eix d'abscisses

OX.

Solució PAU CAT TEC SET 2001 4.4

14. Calculeu per integració la superfície del recinte delimitat per les corbes y = x2 i la

recta d'equació y – x – 6 = 0 representat en el dibuix següent:

Solució PAU CAT TEC SET 2000 2.2

15. Calculeu l'àrea que té l'únic recinte tancat limitat per les gràfiques de les funcions

72 xy i y =6/x representat en el dibuix següent:

Solució PAU CAT TEC JUNY 2000 1.2

16. Calculeu l'àrea de l'únic recinte tancat limitat per les gràfiques de les funcions

y = 8x i y = x4.

Solució PAU CAT CCSS JUNY 2000 3.2

17. Calculeu l'àrea de la regió limitada per la gràfica de la funció y = –x4 +x i l'eix de les

x que està representada en el dibuix següent:

PAU CAT CCSS SET 1999 2.3

18. Calculeu l'àrea determinada per les corbes d'equacions y = x4 – 2x

2 i y = 2x

2

representada en el dibuix següent:

Solució PAU CAT TEC JUNY 1999 1.2

19. a) Dibuixeu la gràfica de la funció y = –x2 +5x –6.

b) Calculeu l'àrea del recinte limitat per la gràfica de la funció anterior i l'eix de les x.

Solució PAU CAT CCSS JUNY 1999 1.4

20. a) Escriviu l'equació de la recta tangent a la paràbola y=x2 en el punt d'abscissa

x=-1.

b) Calculeu l'àrea de la regió que limiten la tangent i la corba anteriors per a x entre 0 i

1.

PAU CAT CCSS SET 1998 2.6

21. Considereu la funció xxxxf 86)( 23 la gràfica de la qual és aproximadament

la del dibuix següent:

Calculeu l’àrea de la regió ombrejada.

Solució PAU CAT TEC JUNY 1998 3.3

13.3.3 Exercicis de càlcul d’àrees.

1. Determineu l’àrea del recinte tancat per les corbes 833 xxy , xy 3 , i les

rectes verticals 3x , 0x .

2. Determineu l’àrea del recinte tancat per les corbes xy , 2xy , 4

2xy .

3. Trobeu l’àrea de la regió limitada per les corbes 32 23 xxy i xxy 2 entre

2x i 2x

4. Calcula l’àrea ombrejada de la figura següent. Les funcions representades són:

2

2)(

2

x

xf i xxxg 2)( 2

5. Determina l’àrea de la regió tancada per les corbes 3xy i xxy 2

13.3.4 Problemes PAU amb àrees de funcions polinòmiques parametritzades.

1. Siguin les paràboles 22)( kxxf i 22 9)( kxxg .

a) Calculeu les abscisses, en funció de k, dels punts d’intersecció entre les dues

paràboles.

b) Calculeu el valor del paràmetre k perquè l’àrea compresa entre les paràboles sigui de

576 unitats quadrades. Solució PAU CAT TEC JUNY 2016 3.6

2. La corba y = x2 i la recta y = k, amb k > 0, determinen una regió plana.

a) Calculeu el valor de l’àrea d’aquesta regió en funció del paràmetre k.

b) Trobeu el valor de k perquè l’àrea limitada sigui 26 u .

Solució PAU CAT TEC JUNY 2013 4.2

3. Definim les funcions )1()( 2xaxf i a

xxg

1)(

2 , en què a>0.

a) Comproveu que l’àrea del recinte limitat per les gràfiques de les funcions és:

a

a

3

14 2

b) Calculeu el valor del paràmetre a perquè aquesta àrea sigui mínima.

Solució PAU CAT TEC JUNY 2011 1.3

4. En la figura es mostra la corba y = x(4 – x) i una recta r que passa per l’origen i talla

la corba en un punt P d’abscissa k, amb 0 < k < 4.

a) Trobeu l’àrea ombrejada, delimitada per la corba i la recta, en funció de k.

b) Trobeu per a quin valor de k l’àrea de la regió ombrejada és la meitat de l’àrea

del recinte limitat per la corba i l’eix OX. Solució PAU CAT TEC JUNY 2010 5.6

5. Considereu la funció 3

)()(

a

xaxxf

, amb a>0.

a) Trobeu els punts de tall de la funció f(x) amb l’eix OX.

b) Comproveu que l’àrea del recinte limitat per la gràfica de la funció f(x) i l’eix

d’abscisses no depèn del valor del paràmetre a. Solució PAU CAT TEC JUNY 2009 4.3

6. Donades les funcions 4)( 2 axxxf i bx

xg 2

)(2

:

a) Calculeu a i b de manera que les gràfiques de f(x) i de g(x) siguin tangents en el punt

d’abscissa x= 3, és a dir, que tinguin la mateixa recta tangent en aquest punt.

b) Trobeu l’equació de la recta tangent esmentada en l’apartat anterior.

c) Pel valor de a obtingut en el primer apartat, calculeu el valor de l’àrea de la regió

limitada per l’eix d’abscisses OX i la funció f(x). Solució PAU CAT TEC SET 2007 3.6

7. Considereu la funció f(x) = x3 + mx

2 + 1, m ≥ 0.

a) Calculeu el valor de m per tal que l’àrea del recinte limitat per la gràfica de la funció,

l’eix OX i les rectes x = 0 i x = 2 sigui de 10 unitats quadrades.

b) Per a m = 1, indiqueu el punt o els punts on la recta tangent a la gràfica de la funció

forma un angle de 45° amb el semieix positiu de OX.

Solució PAU CAT TEC JUNY 2004 1.5

8. Considerem la regió S del pla limitada per la paràbola y = 3x2 i la recta y = 3

representada en l'esquema següent:

Siguin A i B els punts d'intersecció de la recta i la paràbola, i T el triangle que té per

vèrtexs A, B i l'origen de coordenades (0, 0). Calculeu l'àrea de la regió que resulta quan

es treu el triangle T a la regió S.

Solució PAU CAT TEC JUNY 2003 5.3

9. Calculeu el valor positiu de a que fa que l'àrea compresa entre la recta d'equació

y=ax+2a i la paràbola y=ax2 valgui 18.

Solució PAU CAT TEC JUNY 2002 1.1

10. Considereu la paràbola y = x2 i la recta y = mx, amb m real positiu.

a) Calculeu l'àrea de la regió tancada delimitada per les gràfiques de la paràbola i la

recta en funció de m .

b) Quin valor té m si l'àrea de l'apartat a) és 288?

Solució PAU CAT CCSS JUNY 2002 3.1

11. Donada la funció f(x) = x2 + a, amb a > 0, calculeu el valor de a que faci que l'àrea

determinada per la gràfica de la funció, l'eix d'abscisses i les rectes x = 0 i x = 3 valgui

27.

Solució PAU CAT CCSS JUNY 2000 1.4

12. Trobeu el valor del coeficient k de manera que l'àrea limitada per la funció

f(x) = –x2 + k i l'eix d'abscisses sigui igual a 36 u

2.

PAU CAT TEC SET 1999 5.3

14 Tècniques d’integració.

14.1 Integrals immediates.

La pròpia definició d’integral com a inversa de la derivada ens permet construir una

primera taula de integrals:

Cxkdxk

)1(1

1

pCp

xdxx

pp

Cxdxx

ln1

Cxdxx cossin

Cxdxx sincos

Cxdxx tan)tan1( 2

Cxdxx

tancos

12

Cxdxx

cotsin

12

Cxdxx

arctan1

12

Cxdxx

arcsin1

1

2

Cxdx

xarccos

1

1

2

Cedxe xx

Ca

adxa

xx

ln

Exercici Resolt. Determinació d’intervals d’integració.

Determina el valor de k de forma que l’àrea entre la funció xxf )( , la recta vertical

kx i els eixos de coordenades sigui igual a 75.2

Veiem que la funció està sempre per sobre de l’eix X, per tant l’àrea coincideix amb el

valor de la integral definida:

3

2

3

02

3

2

3

2

2/3)(

)0()()(

2/32/32/3

2/32/32/1

00

kk

Cx

Cx

dxxdxxxF

FkFdxxdxxfÀreakk

Hem de resoldre l’equació

23.34578.1128.1122

2.753

2.753

2

3/22/32/3

2/3

kkk

k

Determina el valor de p positiu per al qual la gràfica de 2)( xxf , la seva recta

tangent pel punt d’abscissa px i l’eix d’abscisses determinen un recinte d’àrea igual

a 12/27 .

En primer lloc trobem la recta tangent a la funció pel punt px :

ppfa 2)(' , 22 2)( pppppapfb , 22 ppxbaxy

Aquesta recta tangent talla l’eix X al punt 22

022

2 p

p

pxppx , és a dir, al punt

mig del segment.

L’àrea que volem calcular la podem interpretar com la diferència entre l’àrea gran de la

gràfica (Àrea A) i l’àrea del triangle de la dreta que determina la recta tangent (Àrea B).

3)(

33

0

3)0()(

32

333

0

2

1

xdxxxF

ppFpFdxxA

p

L’àrea B és l’àrea d’un triangle que té base 2/p i altura 2)( ppf , per tant no cal fer

cap integral, apliquem la fórmula de l’àrea del triangle:

42

2/ 32 pppB

Finalment, 32712

27

1212

34

43

333333

ppppppp

BAÀrea

(suposem )0p

Calcula l’àrea determinada per les gràfiques de xxf sin)( i 2)( xxg .

(Observació: L’equació 2sin xx només té com a solució 0x i 0.8767x )

8767.0

0

2 )0()8767.0(sin FFdxxxA on dxxxxF 2sin)(

Cx

xdxxdxxdxxxxF 3cossinsin)(

322

13

0)0cos()0(

0.8643- 3

8767.0)8767.0cos()8767.0(

3

3

F

F

1357.0(-1)-0.8643- )0()8767.0(sin8767.0

0

2 FFdxxxA

14.2 Integració per substitució.

14.2.1 Fórmula del canvi de variable.

Pretenem calcular una integral de la forma

dxxgxgf )('))((

Fem el canvi de variable )(xgu ens queda

duufdxuufdxxgxgf )(')()('))((

1. dxxxcossin2 2. dxx

xln 3. dxxtan 4. dxxx 12

1. dxxxcossin2

Fem xuxu cos'sin , i per tant

Cx

Cu

duudxxx 3

sin

3cossin

3322

2. dxx

xln

xuxu

1'ln

Cx

Cu

duudxuudxx

xdxx

x 2

ln

2'

1ln

ln 22

3. dxxtan

xxx

xx sin

cos

1

cos

sintan

xuxu sin'cos

CxCuduu

dxuu

dxx coslnln1

'1

tan

4. dxxx 12

xuxu 2'12

C

xC

uduudxxxdxxx

3

1

3

2

2

1

2

121

2

11

32322

14.2.3 Exercicis d’integració immediata i quasiimmediata.

1. dxx5

2. dxxx

3.

dx

xx

x 4

3

4. dxx

x

2

5.

dx

xxx2

412

6. dxx4

1

7. dxe x5

8. dxx5cos

9. dxaxsin

10. dxx

xln

11. dxx3sin

12

12. dxx7cos

12

13. dx

x 73

1

14. dxx 1

1

15. dxx 25

1

16. dxx2tan

17. dxxxcossin 2

18. dxxxsincos3

19. dxxx 12

20.

dxx

x

32 2

Calculeu l’àrea entre la corba 41

)(x

xxf

i les rectes 0x ,

2

1x , i l’eix OX.

Comprovem que l’equació 01

)(4

x

xxf només té com a solució 0x , i per tant

podem calcula aquest àrea com a una única integral:

)0()2/1(1

2/1

0 4FFdx

x

xA

dx

x

xxF

41)(

Fem un canvi de variable xuxu 2'2

CxCuduu

dxxx

dxx

xxF )arctan(

2

1)arctan(

2

1

1

1

2

12

1

1

2

1

1)( 2

244

00arctan2

1)0(

0.122489)2/1(arctan2

1)2/1(

2

2

F

F

Per tant

122489001224890)0()2/1(1

2/1

0 4.-.FFdx

x

xA

14.2.4 Problemes PAU.

1. Considereu la funció x

xxf

)ln()( .

a) Calculeu el domini de la funció f, els punts de tall de la gràfica de f amb els eixos de

coordenades, i els intervals de creixement i decreixement de f.

b) Calculeu l’àrea de la regió del pla determinada per la gràfica de la funció f, les rectes

x = 1 i x = e, i l’eix de les abscisses.

PAU CAT TEC SET 2019 5.6 (Solució: "Compendium Tec", Pàg. 550)

2. Sigui la funció x

xxf

2cos

sin)( .

a) Calculeu una primitiva de la funció f (x).

b) Calculeu l’àrea limitada per la funció f (x) i l’eix de les abscisses entre les abscisses

0x i 4

x .

Solució PAU CAT TEC SET 2017 2.6

3. Sigui la funció f(x) = sin (x).

a) Calculeu l’equació de les rectes tangents a la funció f en els punts d’abscissa x = 0 i x

= π, respectivament. Trobeu les coordenades del punt en què es tallen les dues rectes.

b) Calculeu l’àrea de la regió limitada per la gràfica de la funció f i les rectes tangents

de l’apartat anterior (en cas de no haver resolt l’apartat anterior, suposeu que les rectes

són y = x i y = –x + π, respectivament).

Solució PAU CAT TEC SET 2016 1.4

4. Donada la funció 1)( xxf i la recta horitzontal y = k, amb k > 0,

a) Feu un esbós del recinte limitat per les gràfiques de la funció i la recta, i els eixos de

coordenades.

b) Trobeu el valor de k sabent que l’àrea d’aquest recinte és igual a 14/3.

Solució PAU CAT TEC JUNY 2013 3.3

5. Per a x ≥ 1, considereu la funció 1)( xxf .

a) Trobeu l’equació de la recta tangent a la gràfica de f(x) en el punt d’abscissa igual a

10.

b) Calculeu l’àrea del recinte limitat per la gràfica de f(x), la recta d’equació x = 5 i

l’eix OX. Solució PAU CAT TEC JUNY 2013 5.4

6. Se sap que certa funció derivable F(x) verifica les condicions següents:

4

1)('

xxF i F(1) = 3

a) Trobeu F(x).

b) Calculeu l’àrea compresa entre F(x) i l’eix OX des de x=0 fins a x=1.

Solució PAU CAT TEC JUNY 2008 2.1

7. Donada la funció 45

)(2

x

xxf

a) Calculeu la integral dxxf )( .

b) Trobeu la primitiva F de f que compleixi F(1) = 1. Solució PAU CAT TEC JUNY 2005 4.2

8. Calculeu el valor de la integral següent:

dxx

xx

3

0 1

11

Solució PAU CAT TEC SET 2004 5.1

9. Donada la funció f(x) = cos x – cos3x:

a) Trobeu la seva integral indefinida.

b) Quina és la primitiva de f(x) que passa pel punt

0,

2

?

Indicació: recordeu que sin2 x + cos

2 x = 1.

Solució PAU CAT TEC JUNY 2004 3.2

10. Calculeu l’àrea del recinte tancat que delimiten la gràfica de la funció xy 2 i la

recta y = x.

Solució PAU CAT TEC JUNY 2004 4.2

11. Donada xxexxf 2

)12()( , determineu la funció g(x) tal que g’(x) =f(x) (és a dir,

una primitiva de f(x)) i que el seu gràfic passa pel punt (0, 2).

Solució PAU CAT TEC SET 2003 3.1

12. Calculeu

e

dxx

x

1

3 )(ln2

Solució PAU CAT TEC JUNY 2003 2.2

13. Calculeu la primitiva de la funció 1)( 2 xxxf que s'anul·la en el punt

d'abscissa x = 2.

Solució PAU CAT TEC JUNY 2002 3.1

14. Calculeu l'àrea compresa entre les gràfiques de les corbes y = e2x

i y = e–2x

i la recta

y = 5 representada en l'esquema següent:

Solució PAU CAT TEC JUNY 2002 2.1

15. Trobeu l'expressió general de totes les primitives de 3 25)( xxf . Quina és la que

passa pel punt (8,0)?

Solució PAU CAT CCSS JUNY 2002 2.2

16. a) Quin és l'angle x en radians (0 < x < π/2) tal que sin(x) = cos(x)?

b) Considereu les funcions f(x) = sin(x) i g(x) = cos(x). Calculeu la superfície del

recinte delimitat superiorment per les gràfiques d'aquestes funcions, inferiorment per

l'eix d'abscisses i lateralment per les rectes verticals x = 0 i x = π/3 representat en

l'esquema següent:

Solució PAU CAT TEC JUNY 2001 2.1

17. Calculeu l'àrea de la regió limitada per la gràfica de la funció f(x) = xex per a x ≥ 0,

l'eix d'abscisses i la recta vertical x = 1. Solució PAU CAT TEC JUNY 2001 5.4

18. Determineu la funció f(x) tal que f '(x) = x

2 + sin x i f (0) = 2

Solució PAU CAT CCSS JUNY 2001 5.2

19. Trobeu la primitiva de la funció f(x) = e

–x/2 que compleix la condició que la seva

gràfica passa pel punt (0, 3).

Solució PAU CAT CCSS SET 2000 6.3

20. Sigui 3 21)( xxf . Calculeu l'àrea de la regió que limita la gràfica de f(x) i l'eix

d'abscisses i que està representada en el dibuix següent:

Solució PAU CAT TEC JUNY 1999 6.2

14.3 Integració per parts.

14.3.1 Fórmula de la integració per parts.

Ampliem a continuació el nostre repertori usant la regla de derivació del producte:

Si )(xf i )(xg són funcions de x, tenim

)(')()()('')()( xgxfxgxfxgxf

i integrant:

dxxgxfxgxfdxxgxf )(')()()('')()(

La integració és l’operació oposada a la derivació, i la integral d’una suma és suma

d’integrals, per tant arribem a la següent igualtat:

dxxgxfdxxgxfxgxf )(')()()(')()(

d’on aïllant l’últim membre obtenim la fórmula de la integració per parts:

dxxgxfxgxfdxxgxf )()(')()()(')(

Aquest mètode servirà sempre que la integral del segon membre sigui més senzilla que

la del primer membre que volem calcular.

Exercicis.

1. dxxx sin

2. dxex x

3. dxex x

2

4. dxxx ln3

5. dxx ln

Calcular la integral dxx)sin(ln

Apliquem el mètode d’integració per parts:

xxgxg

xxxfxxf

dxxxxdxxx

xxxdxx

)(1)('

1)cos(ln)(')sin(ln)(

)cos(ln)sin(ln1

)cos(ln)sin(ln)sin(ln

Ara hem d’integrar dxx)cos(ln , amb el mètode d’integració per parts:

xxgxg

xxxfxxf

dxxxxdxxx

xxxdxx

)(1)('

1)sin(ln)(')cos(ln)(

)sin(ln)cos(ln1

)sin(ln)cos(ln)cos(ln

Per tant ens queda la següent igualtat:

dxxxxxx

dxxxxxxdxx

)sin(ln)cos(ln)sin(ln

)sin(ln)cos(ln)sin(ln)sin(ln

És a dir,

Cxxxx

dxx

xxxxdxx

xxxxdxxdxx

2

)cos(ln)sin(ln)sin(ln

)cos(ln)sin(ln)sin(ln2

)cos(ln)sin(ln)sin(ln)sin(ln

Problema PAU.

1. La gràfica de la funció f (x) = x · sin(x) és la següent:

a) Trobeu-ne una primitiva.

b) Aplicant el resultat de l’apartat anterior, calculeu l’àrea del recinte limitat per la

gràfica de la funció f (x) i l’eix d’abscisses des de x = 0 fins a x = π.

Solució PAU CAT TEC JUNY 2010 1.5

1. Representa gràficament la funció xexxf )2()( en l’interval 3,2 . Calcula l’àrea

sota aquesta corba en aquest interval.

L’equació 0)2()( xexxf només té com a solució 2x , per tant podem

determinar aquest àrea amb una única integral:

3

2)2()3()2( FFdxexA x on dxexxF x)2()(

Apliquem el mètode de la integració per parts:

xx exgexg

xfxxf

)()('

1)('2)(

7.38906- )22()2(

0)23()3(

)2(1)2()2()(

222

3333

eeeF

eeeeF

eexdxeexdxexxF xxxxx

38906.7)2()3()2( 23

2 eFFdxexA x

2. Calcula l’àrea que determina la gràfica de 2ln)( xxxf en l’interval e,1 .

Resolem l’equació

10ln0ln

00ln)(

02

2

exxx

xxxxf

Veiem que la funció no canvia de signe dintre de l’interval e,1 i per tant l’àrea queda

determinada per una única integral:

e

FeFdxxxA1

2)1()(ln , on dxxxxF

2ln)(

Aquesta integral indefinida es resol amb el mètode d’integració per parts:

2)()('

1ln2)(')(ln)(

(*)ln2

)(ln2

1ln2

2)(lnln

2

2

22

2222

xxgxxg

xxxfxxf

dxxxx

xdxx

xx

xxdxxx

La integral dxxx ln l’hem de resoldre novament per parts:

2)()('

1)('ln)(

42

ln

22

ln

2

1

2lnln

2

22222

xxgxxg

xxfxxf

Cxxx

dxxxx

dxx

x

xxdxxx

Agrupem ara els elements:

Cxxx

Cxxxx

x

2

1ln)(ln

242

ln

2)(ln(*) 2

22222

4

1

2

1

2

1

2

11ln)1(ln

2

1)1(

42

111

22

1ln)(ln

2)(

22

2

222

2

F

eeee

eeF

i per últim

5973.14

1

4

1

4)1()(ln

22

1

2

eeFeFdxxxA

e

3. Determina l’àrea que determina la funció xexf x sin)( entre 0 i .

Resolem l’equació kxxxexf x 0sin0sin)( . La funció no creua l’eix

X entre 0 i , per tant només necessitem calcular una única integral:

)0()(sin0

FFdxxeA x

on dxxexF x sin)( .

Aquesta integral la resolem amb el mètode d’integració per parts:

xxgxxg

exfexf

dxxexedxxexedxxe

xx

xxxxx

cos)(sin)('

)(')(

(*)coscos)cos()cos(sin

Novament per integració per parts:

xxgxxg

exfexf

dxxexedxxexedxxe

xx

xxxxx

sin)(cos)('

)(')(

sinsinsinsincos

Per tant queda:

dxxexexedxxexexedxxe xxxxxxx sinsincossinsin)cos(sin

Passem la segona integral a l’esquerra:

xexedxxedxxe xxxx sincossinsin

xxexexedxxe xxxx sincossincossin2

2

sincossincossin

xxexexedxxe

xxxx

22

01

2

sincos)(

eee

F

2

1

2

1

2

0sin0cos)0(

00

eeF

5216.02

1

2

1

22

1

2)0()(sin

0

eee

FFdxxeA x

14.4 Integració de funcions racionals.

Volem calcular integrals de funcions racionals:

dxxq

xp )(

)( on )(xp i )(xq són polinomis.

14.4.1 Observació prèvia.

Considerarem només els casos en què el numerador és de grau inferior al denominador,

perquè si no ho fos, en efectuar una divisió entera dels polinomis, donaria lloc a la suma

d’un polinomi amb una fracció el numerador de la qual és de grau inferior al

denominador:

))(())(( xqgrauxpgrau Fem la divisió de polinomis:

)(xp )(xq

)(xR )(xQ

)()()()( xRxqxQxp amb ))(())(( xqgrauxRgrau

i per tant: )(

)()(

)(

)(

)(

)()(

)(

)()()(

)(

)(

xq

xRxQ

xq

xR

xq

xqxQ

xq

xRxqxQ

xq

xp

Exemple:

124

162122

23

xx

xxx

16212 23 xxx 124 2 xx 37)124)(23(16212 223 xxxxxxx

37 x 23 x

124

3723

124

37

124

)124)(23(

124

37)124)(23(

124

16212

2

22

2

2

2

2

23

xx

xx

xx

x

xx

xxx

xx

xxxx

xx

xxx

14.4.2 Integració de funcions racionals simples dxbxa

1

Es resolen mitjançant integració per substitució:

aubxau '

Cbxaa

Cua

duua

dxuua

dxbxa

a

abxa

ln1

ln111

'1111

14.4.3 Integració amb factors lineals. Exemples resolts.

1. 12x

dx

El denominador es pot escriure com a producte de dos factors lineals:

)1)(1(12 xxx

Per tant escrivim la fracció com a suma de fraccions simples :

1

2/1

1

2/1

1

1

per tant

2/1

2/1

1

0

)1)(1(

)(

)1)(1()1)(1(

)1)(1(

)1()1(

)1)(1(

)1(

)1)(1(

)1(

11)1)(1(

1

1

1

2

2

xxx

B

A

AB

BA

xx

ABxBA

xx

ABBxAx

xx

BBxAAx

xx

xBxA

xx

xB

xx

xA

x

B

x

A

xxx

*1

1

2

1

1

1

2

1

1

2/1

1

2/1

12

dx

xdx

xdx

xdx

xx

dx

Integrem per separat:

Cxduu

dxx

1ln1

1u'1xuFent

1

1

Cxduu

dxx

1ln1

1u'1xuFent

1

1

Cxx

Cxx

2

1ln1ln1ln

2

11ln

2

1*

2.

dx

xx

x3

3

)1(

1

Escrivim 3

3

)1(

1

xx

xcom a suma de fraccions simples:

3

23

3

23

322

2

3

3

322

2

3

3

323

3

)1(

)1()1()1(

)1(

)1()1()1(

1)1(1

)1(

)1()1(

)1(

)1(

)1(

1)1(1

)1(

)1()1(

)1(

)1(

)1(

111)1(

1

xx

DxxCxxBxxA

xx

DxxCxxBxxA

xx

Dx

xxx

xCx

xxx

xBx

xx

xA

xx

Dx

xxx

xCx

xxx

xBx

xx

xA

x

D

x

C

x

B

x

A

xx

x

DxxCxxBxxAx )1()1()1(1 233

Donant valors a la x:

1

2

224802480-1 xaPer

42C2B192D2C2BA92 xaPer

-1A-A10 xaPer

D21 xaPer

C

B

CBDCBA

Cxx

xx

Cxx

xx

dxx

dxx

dxx

dxx

dxx

dxx

dxx

dxx

dxxxxx

dxxx

x

xxxxxx

x

2

2

32

32

323

3

323

3

)1(

2

)1(

11ln2ln

)1(

12

)1(

11ln2ln)1(

1

12

1

1

1

12

1)1(

1

2

1

1

1

21

1

2

1

1

1

21

)1(

1

1

2

1

1

1

21

)1(

1

14.4.4 Integració amb factors no lineals.

Calcula la següent integral indefinida: dx

xx

x

222

222 xx no factoritza perquè 0222 xx no té solució real.

Ara bé, 1111222222 xxxxx per tant

dx

x

xdx

xx

x

1)1(22 22

Fent

1

1'1

ux

uxu

*1

1

11

1222

du

udu

u

udu

u

u

Per a la primera integral:

Cxx

Cxx

CxCuCpdpp

upupFent

duu

u

22ln2

1

112ln2

1

1)1(ln2

11ln

2

11ln

2

1

1

1

2

1

2'

1

2

2

22

2

2

Per a la segona integral:

CxCuduu

)1arctan()arctan(

1

12

Per tant

Cxxx )1arctan(22ln2

1* 2

Problemes PAU amb integració de funcions racionals.

1. Sigui 12

8)(

2

x

xxf . Trobeu l’àrea del recinte limitat per la gràfica d’aquesta funció,

l’eix OX i les rectes x = 0 i x = 2.

Solució PAU CAT TEC SET 2010 2.5

2. La gràfica de la funció x

xxf

3)( , des de x=1 fins a x=4, és la següent:

a) Calculeu l’equació de les rectes tangents a aquesta funció en els punts d’abscissa x= 1

i x= 3.

b) Dibuixeu el recinte limitat per la gràfica de la funció i les dues rectes tangents que

heu calculat.

c) Trobeu els vèrtexs d’aquest recinte.

d) Calculeu la superfície del recinte damunt dit.

Solució PAU CAT TEC JUNY 2009 4.5

3. Sigui la funció 2

4)(

x

b

xaxf .

a) Calculeu els valors de a i b, sabent que la recta 2x + 3y= 14 és tangent a la gràfica de

la funció f(x) en el punt d’abscissa x= 3.

Per a la resta d’apartats, considereu que a= –3 i que b= 4.

b) Trobeu els intervals de creixement i de decreixement de la funció f(x). Trobeu i

classifiqueu els extrems relatius que té la funció.

c) Calculeu els punts de tall de la funció f(x) amb l’eix OX.

d) Trobeu l’àrea del recinte limitat per la gràfica de la funció f(x), l’eix OX i les rectes

x= 1 i x= 3.

Solució PAU CAT TEC JUNY 2009 3.5 (Problema)

4. El gràfic de la funció 12

1)(

xxf , quan x>0, és com segueix:

a) Trobeu una primitiva de la funció f.

b) Calculeu l’àrea de la regió ombrejada.

Solució PAU CAT TEC SET 2006 4.2

5. Donada la funció 4

164)(

xxxf

a) Estudieu-ne la continuïtat.

b) Estudieu-ne els intervals de creixement i decreixement i els màxims i mínims locals.

c) Calculeu l'àrea limitada per la gràfica de la funció, l'eix OX i les rectes verticals x = 0

i x = 2. Solució PAU CAT TEC JUNY 1999 1.5

6. Trobeu el valor de k per tal que

k

k kx

dx2

13

PAU CAT TEC SET 1998 5.1

7. Trobeu l'àrea de la figura compresa entre la hipèrbola xy=1, les rectes x=1 i x=4 i

l'eix X, que està representada en el dibuix següent:

Solució: 3863.1)1ln()4ln( PAU CAT CCSS JUNY 1998 3.2

8. Calculeu la primitiva de la funció 1

2)(

2

x

xxf tal que la seva gràfica passi pel punt

de coordenades (1, 1).

Solució: )2ln(1,1ln)( 2 CCxxF

PAU CAT CCSS JUNY 1998 6.4

14.5 Substitucions trigonomètriques.

Fórmula de la superfície d’un cercle.

Podem calcular la superfície d’un cercle amb les eines del càlcul integral.

La circumferència de radi r queda determinada mitjançant l’equació 222 ryx

Per tant: 22 xry

Agafem la part superior de la circumferència (després multiplicarem per dos el resultat

obtingut): 22 xry .

Ens reduïm al cas radi 1r . *1 2 r

rdxxÀrea

Ja hem calculat abans (vegeu ....) la seva integral indefinida:

Cxx

xdxx

2

1)arcsin(

2

11

22

Per tant

24

2

22

1

22

1

2

11)1arcsin(

2

1

2

11)1arcsin(

2

1

2

1)arcsin(

2

1*

22

1

1

2

xxx

I per tant la superfície del cercle de radi 1 és .

14.5 Determinació d’àrees amb funcions en general.

1. Considerem la funció xxf 5)( i la funció x

xg4

)(

a) Fes un esbós de les dues gràfiques, indicant els seus punts de tall.

b) Determina l’àrea de la regió que delimiten.

2. Considera les funcions definides per 22)( xxf i xxg )(

a) Fes un esbós de les dues gràfiques, indicant els seus punts de tall.

b) Calcula l’àrea del recinte que delimiten.

3. Donada la funció IRf ),0(: definida per xxf ln)( , on ln és la funció

logaritme neperià, es demana:

a) Comproveu que la recta d’equació 21 exey és la recta normal a la gràfica de f

en el punt d’abscissa ex .

b) Calculeu l’àrea de la regió limitada per la gràfica f, l’eix d’abscisses i la recta normal

de l’apartat a.

4. Considerem la funció IRIRf : donada per 4)( 2 xxf .

a) Trobeu l’equació de la recta tangent a la gràfica de f al punt d’abscissa 1x .

b) Feu un esbós del recinte limitat per la gràfica de f, l’eix d’ordenades i la recta

d’equació 32 xy . Calcula la seva àrea.

5. Trobeu el valor de 0a sabent que l’àrea del recinte limitat entre la paràbola

xaxy 2 i la recta 0 xy val 36 unitats quadrades.

6. Considerem la funció IRIRf : definida per xxxf 2)(

a) Fes un esbós de la seva gràfica.

b) Calcula l’àrea del recinte limitat per la gràfica de f, l’eix d’abscisses i la recta

d’equació 3x .

7. Siguin IRIRgf :, les funcions definides per 32)( 2 xxxf i 12

1)( 2 xxg .

a) Fes un esbós de les gràfiques de f i g, i troba el seu punt de tall.

b) Calcula l’àrea del recinte limitat per les dues gràfiques i l’eix d’ordenades.

8. Sigui IRIRf : la funció definida per 1)( xxxf .

a) Fes un esbós de la gràfica de f.

b) Comprova que la recta d’equació xy és la recta tangent a la gràfica de f al punt

d’abscissa 0x .

c) Calcula l’àrea del recinte limitat per la gràfica de f i la tangent anterior.

9. Considera la corba d’equació xxy 33 .

a) Troba l’equació de la recta tangent a la corba al punt d’abscissa 1x .

b) Calcula l’àrea del recinte limitat per la corba donada i la recta 2y .

10. La corba 2

2

1xy divideix el rectangle de vèrtexs )0,0(A , )0,2(B , )1,2(C i

)1,0(D en dos recintes.

a) Dibuixa aquests dos recintes.

b) Troba l’àrea de cadascun d’ells.

11. Considera les funcions IRIRgf :, definides per xxf )( i 26)( xxg .

a) Dibuixa un esbós del recinte limitat per les seves gràfiques.

b) Calcula l’àrea d’aquest recinte.

12. Sigui IRf ),0(: la funció definida per )ln(1)( xxf , on ln és la funció

logaritme neperià.

a) Comprova que la recta d’equació xe

y1

1 és la recta tangent a la gràfica de f al

punt d’abscissa ex .

b) Calcula l’àrea del recinte limitat per la gràfica de f, l’eix d’abscisses i la recta tangent

de l’apartat a.

13. Considerem les funcions IRf ,0: i IRIRg : definides per

xxf 3)( i 2

3

1)( xxg

a) Fes un esbós de les seves gràfiques.

b) Calcula l’àrea del recinte limitat per les dues gràfiques anteriors.

14. Siguin les funcions IRIRgf :, definides per 1)( 2 xxf i 1)( xxg

Calcula l’àrea del recinte limitat per les seves gràfiques.

15. Les dues gràfiques del dibuix corresponen a la funció IRf ),0(: definida per

)ln(22

)( xx

xf i a la de la seva derivada IRf ),0(:' .

a) Indica, raonant la resposta, quina és la gràfica de f i quina és la de 'f .

b) Calcula l’àrea de la regió ombrejada.

16. Siguin IRIRf : i IRIRg : les funcions definides per

xxxf 2)( i 2)( xg

a) Determina els punts de tall de les gràfiques de f i g. Fes un esbós d’ambdues.

b) Calcula l’àrea del recinte limitat per ambdues gràfiques.

17. Calcula un nombre positiu a, menor que 4, per al qual el recinte limitat per la

paràbola d’equació 2xy i les dues rectes d’equacions 4y i ay tingui un àrea de

3/28 unitats quadrades.

18. Sigui IRIRf : la funció donada per xexf 2)( .

a) Justifica que la recta d’equació exy 2 és la recta tangent a la gràfica de f al punt

d’abscissa 2

1x .

b) Calcula l’àrea del recinte limitat per la gràfica de f, l’eix d’ordenades i la recta

tangent de l’apartat anterior.

19. Siguin IRIRf : i IRIRg : les funcions definides per 1)( 2 xxf i

22)( xxg

a) Fes un esbós de les gràfiques de f i g.

b) Calcula l’àrea del recinte limitat per ambdues.

20. Donades les funcions IRf ,0: i IRg ,0: definides per

xxf )( i 3)( xxg

Calcula l’àrea del recinte limitat per les gràfiques de f i g.

21. Sigui IRg ),0(: la funció donada per xxg ln)( (ln denota logaritme

neperià)

a) Justifica que la recta d’equació xe

y1

és la recta tangent a la gràfica de g al punt

d’abscissa ex .

b) Calcula l’àrea del recinte limitat per la gràfica de g, l’eix d’abscisses i la recta tangent

de l’apartat anterior.

22. Siguin IRIRgf :, les funcions definides mitjançant xxxf 4)( 3 i

63)( xxg

a) Determina els punts de tall de les gràfiques de f i g.

b) Calcula l’àrea del recinte limitat per aquestes gràfiques.

23. Siguin IRIRgf :, les funcions donades per 2)( xxf i axg )( (amb 0a ).

Sabem que l’àrea del recinte limitat per les gràfiques de les funcions f i g és 3/4 .

Calcula el valor de la constant a.

24. Sigui IRIRf : la funció definida per

26

2)(

xx

xxxxf

a) Fes un esbós de la gràfica de f.

b) Calcula l’àrea compresa entre la gràfica de f i l’eix d’abscisses.

25. Calcula e

dxxx1

2 )ln(

26. Sigui IRIRg : la funció definida per xxxxg 23

4

1)(

a) Fes un esbós de la gràfica de g.

b) Determina l’equació de la recta tangent a la gràfica de g al punt d’abscissa 2x

c) Calcula l’àrea del recinte limitat per la gràfica de g i l’eix d’abscisses.

15 Llistes de repàs general.

15.1 Exercicis de repàs general.

1. a) Determina la recta tangent a la funció xxf 3)( al punt d’abscissa 1x .

b) Determina la recta tangent a la funció anterior al punt d’ordenada 1y

2. Donada la funció )3ln()( 2 xxxf

a) Calcula la seva derivada.

b) Determina els punts de la gràfica de f en els què la recta tangent és paral·lela a la

recta 012 yx

3. Donada la funció 1

)(2

2

x

xxf

a) Determina les seves asímptotes.

b) Estudia el creixement, decreixement i determina els seus extrems relatius.

c) Estudia la curvatura i determina els seus punts d’inflexió.

4. Volem anar d’A a B passant per un punt C del riu. Determineu el punt C que fa

mínima la distància d’aquest viatge.

15.2 Llista de repàs d’integració i àrea.

1. Calcula l’àrea que determina la gràfica de la funció 107)( 2 xxxf entre els

punts 1x i 4x .

2. El polinomi baxxxp 2)( s’anul·la per a 2x i compleix

2

0 3

46)( dxxp .

Calculeu raonadament a i b.

3. (Opció A)

Donada la funció 24

)(x

xxf

,

a) Representa la seva gràfica (no cal fer un estudi complet)

b) Determina dxxf )(

c) Determina l’àrea entre la funció i la primera bisectriu xy . (Es demanen valors

exactes, no es poden presentar resultats decimals)

3. (Opció B)

Donada la funció 2)1(

)(x

x

e

exf

a) Determineu un punt de la seva gràfica en el què la recta tangent sigui paral·lela a

l’eix d’abscisses. Determineu aquesta recta tangent.

b) Determina dxxf )(

c) Determineu l’àrea delimitada per la gràfica d’aquesta funció, l’eix d’abscisses i les

rectes 0x i 5lnx .

4. Determina l’interval k,0 (k positiu) per al què l’àrea compresa entre la funció

xxxf 63)( 2 i l’eix d’abscisses en aquest interval sigui igual a 58 u2.

Les 21 funcions fonamentals Família Funció Funció inversa

Constant

axf )( 1 No en té

Identitat

xxf )( 2 xxf )(1

2

Polinòmiques

de grau 1

Proporcionalitat directa

axxf )( 3 x

axf

1)(1

3

Lineal general

baxxf )( 4 a

bxxf

)(1 4

Valor absolut (lineal a trossos)

xxf )( 5 (Només localment)

Part entera (Constant a trossos)

xxf )( 21 No en té

Polinòmiques de grau

>1

Quadràtica 2)( xxf 7

Arrel quadrada

xxf )(1 9

Cúbica 3)( xxf 8

Arrel cúbica 31 )( xxf ---

Racionals

Proporcionalitat inversa

xxf

1)( 6 x

xf1

)( 6

Exponencial general

xaxf )( 10 Logaritme general

)(log)(1 xxf a 12

Exponencials i

logarítmiques

Exponencial de base e xexf )( 11

Logaritme natural

)ln()(1 xxf 13

Exponencial de base 10 xxf 10)( ---

Logaritme en base 10

)log()(1 xxf 14

Sinus

)sin()( xxf 15 Arcsinus

)arcsin()(1 xxf 18

Trigonomètriques

Cosinus

)cos()( xxf 16 Arccosinus

)arccos()(1 xxf 19

Tangent

)tan()( xxf 17 Arctangent

)arctan()(1 xxf 20

1. Funció constant.

axf )(

Domini: ),(

Imatge: a

Període: No en té.

Intersecció X: No en té (si 0a ) o ),( si 0a

Intersecció Y: a

Creixement: No en té (horitzontal)

Mínims i màxims: No en té

Asímptotes verticals: No en té.

Asímptotes horitzontals: ay

Simetria: Respecte de l’eix Y.

Funció parell/senar: Funció parell.

Derivada: 0)(' xf

Funció inversa: No en té

Amb calculadora científica: ---

2. Funció identitat.

xxf )(

Domini: ),(

Imatge: ),(

Període: No en té.

Intersecció X: 0

Intersecció Y: 0

Creixement: creixent

Mínims i màxims: No en té

Asímptotes verticals: No en té.

Asímptotes horitzontals: No en té

Simetria: Respecte de l’eix Y.

Funció parell/senar: Funció parell.

Derivada: 1)(' xf (tipus 1)

Funció inversa: xxf )(1

Amb calculadora científica: ---

3. Funció de proporcionalitat directa.

axxf )(

Domini: ),(

Imatge: ),(

Període: No en té.

Intersecció X: 0

Intersecció Y: 0

Creixement: creixent si a>0, decreixent si a<0

Mínims i màxims: No en té

Asímptotes verticals: No en té.

Asímptotes horitzontals: No en té

Simetria: No en té.

Funció parell/senar: Funció senar.

Derivada: axf )(' (tipus 1)

Funció inversa: xa

xf1

)(1 (tipus 3)

Amb calculadora científica: ---

4. Funció lineal.

baxxf )(

Domini: ),(

Imatge: ),(

Període: No en té.

Intersecció X: a

b

Intersecció Y: b

Creixement: creixent si a>0, decreixent si a<0

Mínims i màxims: No en té

Asímptotes verticals: No en té.

Asímptotes horitzontals: No en té

Simetria: No en té.

Funció parell/senar: ---

Derivada: axf )(' (tipus 1)

Funció inversa: a

bxxf

)(1 (tipus 2)

Amb calculadora científica: ---

5. Funció valor absolut.

xxf )(

Domini: ),(

Imatge: ,0

Període: No en té.

Intersecció X: 0

Intersecció Y: 0

Creixement: creixent si x>0, decreixent si x<0

Mínims i màxims: No en té

Asímptotes verticals: No en té.

Asímptotes horitzontals: No en té

Simetria: Respecte de l’eix Y

Funció parell/senar: Funció parell

Derivada:

01

01)('

x

xxf

Funció inversa: Només localment.

Amb calculadora científica: ---

6. Funció de proporcionalitat inversa.

xxf

1)(

Domini: ),0()0,(

Imatge: ),0()0,(

Període: No en té.

Intersecció X: No en té

Intersecció Y: No en té

Creixement: Decreixent

Mínims i màxims: No en té

Asímptotes verticals: L’eix Y.

Asímptotes horitzontals: L’eix X

Simetria: Respecte de l’eix Y i també respecte de l’eix X

Funció parell/senar: Funció senar

Derivada: 2

1)('

xxf

Funció inversa: x

xf1

)(1 (ella mateixa)

Amb calculadora científica:

7. Funció quadràtica. 2)( xxf

Domini: ),(

Imatge: ,0

Període: No en té.

Intersecció X: 0

Intersecció Y: 0

Creixement: )0,( decreixent, ),0( creixent

Mínims i màxims: Mínim en x = 0

Asímptotes verticals: No en té

Asímptotes horitzontals: No en té

Simetria: Respecte de l’eix Y

Funció parell/senar: Funció parell

Derivada: xxf 2)('

Funció inversa: 0)(1 xsixxf (localment)

Amb calculadora científica:

8. Funció cúbica.

3)( xxf

Domini: ),(

Imatge: ),(

Període: No en té.

Intersecció X: 0

Intersecció Y: 0

Creixement: Creixent.

Mínims i màxims: No en té.

Asímptotes verticals: No en té

Asímptotes horitzontals: No en té

Simetria: Simetria central respecte a l’origen.

Funció parell/senar: Funció senar

Derivada: 23)(' xxf

Funció inversa: 31 )( xxf

Amb calculadora científica:

9. Funció arrel quadrada.

xxf )(

Domini: ),0[

Imatge: ),0[

Període: No en té.

Intersecció X: 0

Intersecció Y: 0

Creixement: Creixent.

Mínims i màxims: No en té.

Asímptotes verticals: No en té.

Asímptotes horitzontals: No en té.

Simetria: No en té.

Funció parell/senar: ---

Derivada: x

xf2

1)('

Funció inversa: 21 )( xxf

Amb calculadora científica:

10. Funció exponencial general.

)1,0()( aaaxf x

Domini: ),(

Imatge: ),0(

Període: No en té.

Intersecció X: No en té.

Intersecció Y: 1

Creixement: Creixent.

Mínims i màxims: No en té.

Asímptotes verticals: No en té.

Asímptotes horitzontals: Eix de les X

Simetria: No en té.

Funció parell/senar: ---

Derivada: )ln()(' aaxf x

Funció inversa: )(log)(1 xxf a

Amb calculadora científica:

11. Funció exponencial de base e.

xexf )(

Domini: ),(

Imatge: ),0(

Període: No en té.

Intersecció X: No en té.

Intersecció Y: 1

Creixement: Creixent.

Mínims i màxims: No en té.

Asímptotes verticals: No en té.

Asímptotes horitzontals: Eix de les X

Simetria: No en té.

Funció parell/senar: ---

Derivada: xexf )('

Funció inversa: )ln()(1 xxf

Amb calculadora científica:

12. Funció logaritme de base a.

)1,0()(log)( aaxxf a

Domini: ),0(

Imatge: ),(

Període: No en té.

Intersecció X: 1

Intersecció Y: No en té

Creixement: Creixent.

Mínims i màxims: No en té.

Asímptotes verticals: L’eix Y

Asímptotes horitzontals: Eix de les X

Simetria: No en té.

Funció parell/senar: ---

Derivada: xa

ex

xf a

1

)ln(

1log

1)('

Funció inversa: xaxf )(1

Amb calculadora científica: Mitjançant la fórmula )log(/)log()(log abba

13. Funció logaritme natural.

)ln()( xxf

Domini: ),0(

Imatge: ),(

Període: No en té.

Intersecció X: 1

Intersecció Y: No en té

Creixement: Creixent.

Mínims i màxims: No en té.

Asímptotes verticals: L’eix Y.

Asímptotes horitzontals: No en té.

Simetria: No en té.

Funció parell/senar: ---

Derivada: x

xf1

)(' ( tipus 6 )

Funció inversa: xexf )(1 ( tipus 11 )

Amb calculadora científica:

14. Funció logaritme de base 10.

)log()( xxf

Domini: ),0(

Imatge: ),(

Període: No en té.

Intersecció X: 1

Intersecció Y: No en té

Creixement: Creixent.

Mínims i màxims: No en té.

Asímptotes verticals: L’eix Y.

Asímptotes horitzontals: No en té.

Simetria: No en té.

Funció parell/senar: ---

Derivada: x

ex

xf1

)10ln(

1log

1)('

Funció inversa: xxf 10)(1 ( tipus 11 )

Amb calculadora científica:

15. Funció sinus.

)sin()( xxf

Domini: ),(

Imatge: 1,1

Període: 2

Intersecció X: n

Intersecció Y: 0

Creixement: Creixent en

2,

2

, decreixent en

2

3,

2

(periòdicament)

Mínims i màxims: Màxim en

1,

2

, mínim en

1,

2

3 i múltiples.

Asímptotes verticals: No en té.

Asímptotes horitzontals: No en té.

Simetria: Respecte a l’origen

Funció parell/senar: Funció senar

Derivada: )cos()(' xxf ( tipus 16 )

Funció inversa: )arcsin()(1 xxf (localment) ( tipus 18 )

Amb calculadora científica:

16. Funció cosinus.

)cos()( xxf

Domini: ),(

Imatge: 1,1

Període: 2

Intersecció X:

n2

Intersecció Y: 1

Creixement: Decreixent en ,0 , creixent en 2, (periòdicament)

Mínims i màxims: Màxim en 1,2 , mínim en 1, i múltiples.

Asímptotes verticals: No en té.

Asímptotes horitzontals: No en té.

Simetria: Respecte a l’eix Y

Funció parell/senar: Funció parell

Derivada: )sin()(' xxf ( tipus 15 )

Funció inversa: )arccos()(1 xxf (localment) ( tipus 19 )

Amb calculadora científica:

17. Funció tangent.

)tan()( xxf

Domini: Totes les

nx 2

Imatge: ),(

Període: 2

Intersecció X: n

Intersecció Y: 0

Creixement: Creixent

Mínims i màxims: No en té

Asímptotes verticals:

n2

.

Asímptotes horitzontals: No en té.

Simetria: Respecte a l’origen

Funció parell/senar: Funció senar

Derivada: )(tan1)(' 2 xxf

Funció inversa: )arctan()(1 xxf (localment) ( tipus 20 )

Amb calculadora científica:

18. Funció arcsinus.

)(arcsin)( xxf

Domini: 1,1

Imatge:

2,

2

Període: ---

Intersecció X: 0

Intersecció Y: 0

Creixement: Creixent

Mínims i màxims: Màxim en

2,1

, mínim en

2,1

Asímptotes verticals: N No en té.

Asímptotes horitzontals: No en té.

Simetria: Respecte a l’origen

Funció parell/senar: Funció senar

Derivada: 21

1)('

xxf

Funció inversa: )sin()(1 xxf ( tipus 15 )

Amb calculadora científica:

19. Funció Arccosinus.

)(arccos)( xxf

Domini: 1,1

Imatge: ,0

Període: ---

Intersecció X: 1

Intersecció Y: 2

Creixement: Decreixent

Mínims i màxims: Màxim en ,1 , mínim en 0,1

Asímptotes verticals: No en té.

Asímptotes horitzontals: No en té.

Simetria: No en té.

Funció parell/senar: ---

Derivada: 21

1)('

xxf

Funció inversa: )cos()(1 xxf ( tipus 16 )

Amb calculadora científica:

20. Funció Arctangent.

)(atan)( xxf

Domini: ),(

Imatge:

2,

2

Període: No en té

Intersecció X: 0

Intersecció Y: 0

Creixement: Creixent

Mínims i màxims: No en té

Asímptotes verticals: No en té.

Asímptotes horitzontals: 2

y i

2

y

Simetria: Simètrica respecte a l’origen

Funció parell/senar: Funció senar.

Derivada: 21

1)('

xxf

Funció inversa: )tan()(1 xxf ( tipus 17 )

Amb calculadora científica:

21. Funció part entera.

xxf )(

Domini: ),(

Imatge: Els nombres enters

Període: No en té

Intersecció X: 1,0

Intersecció Y: 0

Creixement: Creixent a salts.

Mínims i màxims: No en té

Asímptotes verticals: No en té.

Asímptotes horitzontals: No en té.

Simetria: ---

Funció parell/senar: ---

Derivada: 0)(' xf si x no és un nombre enter.

Funció inversa: No en té.

Amb calculadora científica: ---

Encara que aquesta funció s’anomena “part entera”, no és “eliminar els decimals”:

És el valor enter més petit menor o igual que x. Observa que si x és negatiu

31.2)1.2( f , 2)1.2( f

Solucions. 1.5 a) 1/4 b) 2

1.6 1. a) 4 b) 2

2 c) 4 d)

3

2 e) 2

2. a) 22

1 b) 32 c)

4

2 d) 0 e) 1/8 f) 28 g) -32

2.4

1. a) x = -3 b) 6.05

3

5040

30)0(

2

fy c) x = -1 , x = 5

d) Grau Numerador = 1, Grau Denominador = 2, per tant 0)(lim

xfx

2. a) 2 i 4 b) 1.6 c) –5 i 1 d) tendeix a una asímptota horitzontal y = 1

e)

3. a) 3 b) 9 c) –1

d) Tendeix a l’asímptota obliqua y = 6 x – 7

e)

4. a) )3)(2(

)3)(2)(1()(

xx

xxxxf b) 1, -2 , 3 c) –1

d) 2, -3 e) tendeix a l’asímptota obliqua 3 xy

f)

5. ( 4)( 3)( 8)

( )( 1)( 4)( 6)

x x xf x

x x x

6. ( 4)( 7)

( )( 2)( 3)( 5)

x xf x

x x x

7. ( 1)( 3)( 7)

( )( 2)( 5)

x x xf x

x x

-8 -6 -4 -2 2 4 6

-100

100

200

-1 1 2 3 4

-15

-10

-5

3,2,2

1. a)

b) Punt de tall amb l’eix Y: (0,1) , Punts de tall amb l’eix X: (-3,0) i (5,0)

c) Per a qualsevol 2x la funció és el polinomi x+3, i per tant contínua.

De la mateixa manera, si 12 x o si 1x

Si :2x

1)2( f

1)(lim2

xfx

1)(lim2

xfx

Els límits laterals coincideixen amb el valor de la funció en aquest punt per tant la funció és

contínua en aquest punt.

Si :1x

1)1( f

1)(lim1

xfx

451)(lim1

xfx

Els límits laterals són diferents per tant la funció la funció té una discontinuïtat en x=1.

2.

b)

Punt de tall amb l’eix Y: (0,2)

Punts de tall amb l’eix X: (-1,0), (1,0) i (2,0)

c)

Per a qualsevol 0x la funció és el polinomi 2x+2, i per tant contínua.

De la mateixa manera, si 0x , es tracta d’una funció polinomial 232 xx , perfectament

contínua.

Si :0x

2)0( f

220222lim)(lim22

xxfxx

2203023lim)(lim 22

22

xxxf

xx

Els límits laterals coincideixen amb el valor de la funció en aquest punt per tant la funció també

és contínua en aquest punt.

3.

Per a qualsevol 0x la funció és el polinomi x , i per tant contínua.

De la mateixa manera, si 0x , la funció és l’exponencial, contínua.

Si :0x

0)0( f

0)(lim0

xfx

1)(lim 0

0

exf

x

Els límits laterals no coincideixen en aquest punt per tant la funció té una discontinuïtat de salt.

4.

Per a qualsevol 1x la funció és 12 x , una composició de funcions contínues, i per tant

contínua.

Si 1x , la funció és polinòmica, i per tant contínua.

Per a 1x apliquem la definició de continuïtat:

0)1( f

0111lim)(lim 22

11

xxf

xx

021222lim)(lim11

xxfxx

Aquests tres valors coincideixen, i per tant en aquest punt la funció també és contínua.

5.

Observem que la funció té una discontinuïtat de salt en x=3

3.4.2 1. a) k=6 b) k=1 c) 2

1k

2.

3. a) a=4, b=1 b) a=3, b=2

4. a=13/6, b=19/9

5. h=2

5.1.1

1. a) α = -70.346 b) α = 63.435° c) α = 69.882° d) α = -36.87°

2. a) x = -2.3 b) x = 1.7 c) x = 2.103 d) x = 2.198

3. a) ( 1.25 , 1.125 ) b) ( 0.5 , 3.75 ) c) ( -1 , 4 ), ( 1.667 , -5.481 )

d) ( -1.667 , -3.481 ), ( 1 , 6 )

5.2.2 1. xxxxf 240848)( 23

Punts de tall amb els eixos:

Punts de tall amb l’eix X: (0,0)

Punt de tall amb l’eix Y: (0,0)

Màxims i mínims relatius:

Màxim relatiu: 208,2

Mínim relatiu: 100,5

Monotonia:

Creixent: ,52,

Decreixent: 5,2

Punts d’inflexió: )154,2/7(

Curvatura:

Convexa: )( ,2/7

Còncava: )( 2/7,

2. 78)( 24 xxxf

Punts de tall amb els eixos:

Punts de tall amb l’eix X: )0,7(),0,7(),0,1(),0,1(

Punt de tall amb l’eix Y: (0,-7)

Màxims i mínims relatius:

Màxims relatius: 9,2,9,2

Mínim relatiu: 7,0

Punts d’inflexió:

9

17,

3

2 ,

9

17,

3

2

3. xxxf 4)( 3

Punts de tall amb els eixos:

Punts de tall amb l’eix X: (-2,0) , (0,0) , (2,0)

Punt de tall amb l’eix Y: (0,0)

Màxims i mínims relatius:

Màxim relatiu: 9/316,3/32

Mínim relatiu: 9/316,3/32

Monotonia:

Creixent: ,3/323/32,

Decreixent: 3/32,3/32

Punt d’inflexió: (0,0)

Curvatura:

Convexa: )( ,0

Còncava: )( 0,

4. 924228)( 234 xxxxxf

Punts de tall amb els eixos:

Punts de tall amb l’eix X: (-3,0) , (-1,0)

Punt de tall amb l’eix Y: (0,9)

Màxims i mínims relatius:

Màxim relatiu: 1,2

Mínim relatiu: 0,1,0,3

Punts d’inflexió:

9

4,

3

36,

9

4,

3

36

5. 33)( xxxf

Punts de tall amb els eixos:

Punts de tall amb l’eix X: 0,3 , )0,0( , 0,3

Punt de tall amb l’eix Y: (0,0)

Màxims i mínims relatius:

Màxim relatiu: 2,1

Mínim relatiu: 2,1

Monotonia:

Creixent: 1,1

Decreixent: ,11,

Punts d’inflexió: (0,0)

Curvatura:

Convexa: )( 0,

Còncava: )( ,0

6. 424)( xxxf

Punts de tall amb els eixos:

Punts de tall amb l’eix X: 0,2 , )2,0(

Punt de tall amb l’eix Y: (0,0)

Màxims i mínims relatius:

Màxims relatius: 4,2 i 4,2

Mínim relatiu: 0,0

Monotonia:

Creixent: 2,02,

Decreixent: ,20,2

Punts d’inflexió: 9/20,3/6 , 9/20,3/6

Curvatura:

Convexa: )( 3/6,3/6

Còncava: )( ,66,

7. 34 2)( xxxf

Punts de tall amb els eixos:

Punts de tall amb l’eix X: 0,0 , )0,2(

Punt de tall amb l’eix Y: (0,0)

Màxims i mínims relatius:

Màxims relatius: No n’hi ha.

Mínim relatiu: 16/27,2/3

Monotonia:

Creixent: ,2/3

Decreixent: 2/3,

Punts d’inflexió: 0,0 , 1,1

Curvatura:

Convexa: )( ,10,

Còncava: )( 1,0

8.

Punt de tall amb l’eix Y: )0,0(000)0( 36 f

Punts de tall amb l’eix X:

)0,1(1

)0,0(00)1(00)( 3336 xxxxxxf

Monotonia.

794.02/1

00)12(336)('

3

3225

x

xxxxxxf

Creixent

Decreixent

Decreixent

),794.0(

)794.0,0(

)0,(

La funció té un mínim relatiu al punt 4/1,2/13

Curvatura.

585.05/1

00)15(6630)(''

3

34

x

xxxxxxf

Convexa

Còncava

Convexa

),585.0(

)585.0,0(

)0,(

La funció té punts d’inflexió a )0,0( i 25/4,5/13

Gràfica.

6.1

1.

a) 2

12

10)('

xxf

b) 2

5

2)('

xxf

c) 2)17(

7)('

xxf

d) 2)5(

10)('

xxf

e) 1) x(si 1)(' xf

f) 3

1

3

1)('

2

xxf

g) 22

2

)1(

1)('

x

xxf

h) 22

22

)4(

)12()('

x

xxxf

i) 22 )4(

8)('

x

xxf

6.2

1. Asímptota horitzontal y = 1

22 )14(

)12)(12(4)('

x

xxxf

5.0

5.00)('

x

xxf

2. 2

2 )1)(1)(1(3)('

x

xxxxf

1

10)('

x

xxf

3.

22

2

)1(

1)('

x

xxf

1

10)('

x

xxf

4.

32

2

)1(

314)('

x

xxf

3

1

3

1

0)('

x

x

xf

5.

32 )1(

1)6()('

xx

xxxf

1

00)('

x

xxf

6.

Asímptota obliqua: y = x + 3

2)1(

)2()('

x

xxxf

2

00)('

x

xxf

7.

22

2

)1(

1)('

x

xxf

1

10)('

x

xxf

32

2

)1(

)3(2)(''

x

xxxf

3

3

0

0)('

x

x

x

xf

8.

2

1

2

0)(

x

x

x

xf (Per Ruffini) 4

2

2

128)('

x

xxxf

2915.1

2915.90)('

x

xxf

9.

23

2

)1()2(

)56()('

xx

xxxf

5/6

00)('

x

xxf

6.3.4

1.

Punts de tall amb l’eix X: x = 2

Asímptotes verticals: x = -1

4

20)('

)1(

82)('

2

2

x

xxf

x

xxxf

3)1(

18)(''

xxf

2.

Punts de tall amb l’eix X: x = 5, x = 2

Asímptotes verticals : x = -3

325.3

325.90)('

)3(

316)('

2

2

x

xxf

x

xxxf

0)('')3(

80)(''

3

xf

xxf no té solucions reals.

3. Punts de tall amb l’eix X: no té.

Asímptotes verticals: x = -3 i x = 2

2

10)('

)6(

12)('

22

xxf

xx

xxf

0)('')6(

)733(2)(''

32

2

xf

xx

xxxf no té solucions reals.

4.

Punts de tall amb l’eix X : x = 2

Asímptotes verticals: x = -1 i x = 3

236.0

236.40)('

)32(

14)('

22

2

x

xxf

xx

xxxf

634.60)('')32(

)836(2)(''

32

23

xxf

xx

xxxxf (veieu la indicació)

5.

Punts de tall amb l’eix X : x = 0, x = 4

Asímptotes verticals: x = -2

828.2

828.2

0

0)(')2(

)8(3)('

2

22

x

x

x

xfx

xxxf

679.1

00)(''

)2(

)164(6)(''

3

23

x

xxf

x

xxxxf (veieu la indicació)

6.

a) Domini de definició. }1{

b) Punt(s) de tall amb l’eix X. )0,0(P

c) Punt de tall amb l’eix Y. )0,0(P

d) Asímptotes verticals. En x = 1

e) Comportament en l’infinit: Asímptotes horitzontals i obliqües.

Asímptota obliqua : 2 xy

f) Creixement i decreixement. Màxims i mínims.

3

2

3

2

4

2

4

322

2

3

2

23

)1(

3

)1(

2)1(3

)1(

2)1(3)1(

)1(

)1(2)1(3

)1(

)1(2)1(

3

x

xx

x

xxx

x

xxxx

x

xxxx

x

x

xx

xx

303

000)3(0

)1(

)3()('

2

2

3

2

xx

xxxx

x

xxxf

)0,( )1,0( )3,1( ),3(

05.0)1(' f 02)5.0(' f 04)2(' f 059.0)4(' f

f creixent f creixent f decreixent f creixent

Per tant la funció té un mínim relatiu en x = 3 , 75.6)3( fy

i observem que aquesta funció no té extrem relatiu en x = 0, tot i que 0)0(' f

g) Concavitat i convexitat. Punts d’inflexió.

3

2

)1(

)3()('

x

xxxf

444

22

46

2

6

223

3

2

23

2232

)1(

6

)1(

23

)1(

3233

)1(

)3()1)(2(3

)1(

)3()1)(2()1(3

)1(

)1(3)3()1)(2(3

)1(

)3(

)1(3)1(

)2(3633)3(

x

x

x

x

x

xxxxx

x

xxxxx

x

xxxxxx

x

xxxxxx

x

xx

xx

xxxxxxxx

00)1(

6)(''

4

x

x

xxf

)0,( )1,0( ),1(

0375.0)1('' f

048)5.0('' f 012)2('' f

f corbada cap avall f corbada cap amunt f corbada cap amunt

Per tant la funció té un punt d’inflexió en x = 0, 0)0( fy

Representació gràfica de la funció.

7.

a) 0

b) 0

c) No en té

d) La funció tendeix a una asímptota horitzontal y=1

e) 00)(')3(

6)('

22

xxf

x

xxf

0, decreixent, ,0 creixent, Per tant, la funció té un mínim en P = 0,0

f)

1

10)(''

3

)1(18)(''

32

2

x

xxf

x

xxf

1, corbada cap a baix, 1,1 corbada cap a dalt, ,1 corbada cap a dalt

g)

8.

a) Domini de definició. }5,6{

b) Punt(s) de tall amb l’eix X. x = -2 , x = 1

c) Punt de tall amb l’eix Y. )3/1,0(P

d) Asímptotes verticals. En x = -6 i x = 5

e) Comportament en l’infinit: Asímptotes horitzontals i obliqües.

Asímptota horitzontal y = 5

f) Creixement i decreixement. Màxims i mínims.

2/10)12(1400

30

)12(140)('

30

)12(140

30

140280

30

101515101502951510

30

12105530510

30

1055

1230

5101055

22

2222

22

2323

22

22

2

2

2

2

xxxx

xxf

xx

x

xx

x

xx

xxxxxx

xx

xxxxxx

xx

xx

xxx

xxx

)6,( )5.0,6( )5,5.0( ),5(

064.12)7(' f 021.1)3(' f 015.0)0(' f 011.3)7(' f

f creixent f creixent f decreixent f decreixent

Per tant la funció té un mínim relatiu en x = -0.5 , 37.0)3( fy

g) Concavitat i convexitat. Punts d’inflexió.

0)('')30(

)3133(280)(''

32

2

xf

xx

xxxf no té solucions reals

)6,( )5,6( ),5(

04.25)7('' f

032.0)5.0('' f 04.25)6('' f

f corbada cap a dalt f corbada cap a baix f corbada cap a dalt

f) Representació gràfica de la funció.

7.4

1. xxxf 64)(' 3

2. xxxf 218)(' 2

3. 5

23)('

2 xxxf

4. 21021)( 23

25

xxxf

5.23 2

1

3

1

2

3)('

xxxxf

6.25

2

)1()1(3)('

2

x

xxxf

7.xx

xf11

3

2)('

3

8. 310314)(' xxxxf

9. 192)(' 2 xxxf

10. 12266)(' 2 xxxf

11.22

22

)1(

)3()('

t

tttf

12. 2)3(

)4)(2()('

s

sssf

13.

22

234

2

1262)('

xx

xxxxxf

14. 328)(' 2 xxxf

15. 21)1(

1)('

xxxf

16.

23

22

2

1

41)('

xx

xxf

17.

3 22 13

12)('

xx

xxf

18.

2

3

11)('

xxf

19. xxf 2sin)('

20. xxxf 3sin3cos2)('

21. x

xfcos1

1)('

22. xxxxxf 3sin2sin33cos2cos2)('

23. tttf cos)('

24. ttttf 222 sincos3sin)('

25. xxf tan)('

26.x

xf2sin

2)('

27. xxf cot2)('

28. xxxf cossin)('

29. x

xfcos

1)('

30. x

xxf

)cos(ln)('

31. )cos(cossin)(' xxxf

32. 21

2)('

xxf

33. 3lnsin

cos2)('

2 xx

xxxf

34. 41

4)('

x

xxf

35. xx

xxf

2

12)('

36. 52

23)('

3

2

xx

xxf

37. 1ln)(' xxf

38. x

xxf

2ln3)('

39. 21

1)('

xxf

40. xx

xfln

1)('

41. 54

4)('

x

exf

42. axaxf x ln2)('2

43. 7ln7)1(2)(' 22 xxxxf

44. )21()(' 2xxexf x

45.

21

2)('

x

x

e

exf

46. xexf x cos)(' sin

47. xxexf x 2cos sincos)('

48. ))ln(sin(cot)(' xxexf x

49. )ln1()(' xxxf x

50. xx xxexf

x

)ln1()('

7.3

1. És contínua, però no és derivable en

x=1.

2. És contínua, però no és derivable en

2x b) 03 yx

3. És contínua per a 8k , però no és

derivable.

4. És contínua per a 1k , però no és

derivable.

7.6

2

10

3

1

)1(3

1)('

3 23 2

x

xxxf

)2/1,( ),2/1(

)(' xf + -

)(xf Creixent Decreixent

La funció té un màxim al punt

3 4,2

1

8.2.2

1. y = x y = -3x – 8 y =

5x

2. y = 0.5 x y = -0.64x – 0.12

3. y = x - 1 y = 0.31x - 0.5

y = -0.386x - 0.25

4. 32)( 23 xxxxP

8.3.1 1. )1,1(1 A ,

27

13,

3

12A

2. 279 xy , 59 xy

3. 2211 xyx ,

6232 xyx

4.

1

211

xy

xyx ,

27

1

3

9

26

3

3

12

xy

xy

x

8.3.4

a) Lines that are parallel to the x axis have slope

= 0. The slope of a tangent line to the graph of y

= x3 - 3x is given by the first derivative y ': y ' =

3x2 - 3

We now find all values of x for which y ' = 0:

3x2 - 3 = 0

Solve the above equation for x to obtain the

solutions: x = -1 and x = 1

The above values of x are the x coordinates of

the points where the tangent lines are parallel to

the x axis. Find the y coordinates of these points

using y = x3 - 3x

for x = -1 , y = 2 , for x = 1 , y

= -2

The points at which the tangent lines are parallel

to the x axis are: (-1,2) and (1,-2). See the graph

of y = x3 - 3x below with the tangent lines.

b) We need to determine two algebraic

equations in order to find a and b. Since the

point of tangency is on the graph of y = ax3 + bx

and y = -3x + 4, at x = 1 we have

a(1)3 + b(1) = -3(1) + 4

Simplify to write an equation in a and b: a +

b = 1

The slope of the tangent line is -3 which is also

equal to the first derivative y ' of

y = ax3 + bx at x = 1 y ' = 3ax

2 +

x = -3 at x = 1.

The above gives a second equation in a and b

3a + b = -3

Solve the system of equations a + b = 1 and 3a +

b = -3 to find a and b

a = -2 and b = 3.

See graphs of y = ax3 + bx, with a = -2 and b =

3, and y = -3x + 4 below.

c) The slope of a tangent line is given by the

first derivative y ' of y = aex + bx. Find y '

y ' = aex + b

To find the x coordinate of a point at which the

tangent line to the graph of y is horizontal, solve

y ' = 0 for x (slope of a horizontal line = 0) aex

+ b = 0

Rewrite the above equation as follows ex =

-b/a

The above equation has solutions for -a/b >0.

Hence, the graph of y = ae x + bx has NO

horizontal tangent line if -a/b <= 0

d) (2,2) and (-2,-2)

e) a = 2 and b = - 4

f) 4b2 - 12 ac = 0

8.4 1. y = -2x + 3, y = -2.83x +

7.07 y = -1.414x + 1.414

2. y = -1.65x - 0.22 y = -x + 1

y = - 4.48x - 6.50

3. y = 0.25x+1.875 y = x

y = 2.25x-2.71

8.5 1. 2cos)2(

1cos2)('

xxf ,

3)cos2(

)cos1(sin2)(''

x

xxxf

2. ))(sin1(

cossin2)('

2x

xxxf

,

9

6

9

4 xy

9.1 a) 2

)2()('

x

xexf

x 2x

b) 4

)3()('

x

xexf

x 3x

d) xe

xxxf

)3()('

2

3x (x=0 no és un extrem)

e) 2

)ln(1)('

x

xxf

719.2 ex

f) 3

)ln(21)('

x

xxf

64872.1 ex

g) 4

)ln(31)('

x

xxf

3956.13 ex

9.1.2 1.

a) M = ( 2 , 0.271 ) I = ( 1 , 0.368 )

b) M = ( 2 , 0.541 ) m = ( 0 , 0 ) I1 = ( 0.586 , 0.191 ) I2 = ( 3.414 , 0.384 )

c) m = ( 2.718 , 2.718 ) I = ( 7.3891 , 3.6945 ) (no apareix a la gràfica)

d) m = ( 1.649 , 5.437 )

e) m = ( 0.5 , 2.332 )

9.2 1. 001

2)('

2

x

x

xxf

)0,( ),0(

)(' xf - +

)(xf decreixent creixent

La funció té un mínim al punt (0,0)

1,101

)1(2)(''

2

2

xx

x

xxf

)1,( )1,1( ),1(

)('' xf - + -

)(xf Còncava Convexa còncava

Els punts )2ln,1( i )2ln,1( són punts d’inflexió.

2. 20)1(

2)('

x

xx

xxf , )4ln()2( f , el punt és 4ln,4

3. 00)1(

)1()('

3

x

e

eexf

x

xx

)0,( ),0(

)(' xf + -

)(xf creixent decreixent

La funció té un màxim relatiu al punt

4

1,0

4. 3

10

)1(

)13(4)('

32

2

x

x

xxf

3

1,

3

1,

3

1

,

3

1

)(' xf + - +

)(xf creixent decreixent Creixent

La funció té un màxim al punt

4

33,

3

1 i un mínim al punt

4

33,

3

1.

5. 22 ,10ln2)(ln)(' exxxxxf

2,0 e 1,2e ,1

)(' xf + - +

)(xf creixent decreixent Creixent

La funció presenta un màxim al punt 22 4, ee i un mínim al punt )0,1(

102ln2

)('' exxx

xxf

1,0 e ,1e

)('' xf - +

)(xf Còncava convexa

La funció presenta un punt d’inflexió a 11, ee

6. 1012

)('2

xe

xxxf

x, però 0)(' xf sempre, per tant la funció es sempre

decreixent, i no té ni màxims ni mínims relatius.

3,1034

)(''2

xxe

xxxf

x

1, 3,1 ,3

)('' xf + - +

)(xf Convexa Còncava convexa

7. 10)1(

)('2

xx

xexf

x

1,0 e ,1

)(' xf + -

)(xf creixent Decreixent

La funció té un màxim relatiu a 1x .

10.2 1.

xx

x

x

x

xxAplicant

x

xxx

x

xxx

x

xxxxxf

x

xxf

sin1

1

sin1

)1(sin

sin1

1sin*

1cossin

*sin1

)cos(sinsin

sin1

cossinsin

sin1

))(cos(cos)sin1)(sin()('

sin1

cos)(

22

22

2

22

2

22

2

2. Apliquem la regla de la cadena:

xxxxxfxxf sincos3)sin(cos3)('cos)( 223

Apliquem la propietat 000 BoABA

...,00sin

,...2

3,

20cos0cos

0sincos3

2

2

xx

xxxxx

10.4

1) a) y = 0.622x + 0.224 b) y = -0.942x + 2.973 c) y = 0.644x + 1.216

d) y = 2.06x -0.618 e) y = 1.298x + 0.103

2) a) P = ( 0.9 , 0.783 ) b) P = ( 1 , 0.54 )

13.3.3 1. 81/4 2. 5/2 3. 12 4. 32/27 5. 1.0833

14.2.3

1. Cx

6

6

2. Cxxx

3

2

2

2

3. Cxxx 2

10

16

4. Cxx 2

5

2

5. Cxxx

281

6. Cx 4 3

3

4

7. Ce x 5

5

1

8. Cx

5

5sin

9. Ca

ax

cos

10. Cx 2ln2

1

11. Cx

3

3cot

12. Cx

7

7tan

13. Cx 73ln3

1

14. Cx 1ln

5. Cx 25ln2

1

16. Cx 2cosln2

1

17. Cx

3

sin 3

18. Cx

4

cos 4

19. Cx 32 1

3

1

20. Cx 322

1 2

14.3.1

1.

xxgxxg

xfxxf

cos)(sin)('

1)(')(

Cxxx

dxxxxdxxxxdxxx

sincos

coscos)cos(1)cos(sin

2. dxex x

xx exgexg

xfxxf

)()('

1)(')(

Cexedxexedxex xxxxx 1

3. dxex x

2

xx exgexg

xxfxxf

)()('

2)(')( 2

*22 222 dxexexdxexexdxex xxxxx

i apliquem l’exercici anterior:

CxxeCexeexCexeex xxxxxxx 22222* 222

4. dxxx ln3

4/)()('

/1)('ln)(

43 xxgxxg

xxfxxf

Cxx

Cx

xx

Cx

xx

dxx

xx

dxx

xx

xdxxx

4

1ln

416ln

444

1ln

4

4ln

4

1

4ln

4ln

44444

34443

5. dxx ln

xxgxg

xxfxxf

)(1)('

/1)('ln)(

CxxCxxxdxxxdxxx

xxdxx 1lnln1ln1

lnln

14.5

1.

a)

b) 4ln42

15A

2.

a)

b) 3

7A

3.

a)

b) e

A2

11

4.

a) 32 xy

b)

3

1A

5.

5a

6.

a)

b) 3

8A

7.

a)

b) 3

4A

8.

a)

b) 1A .

9.

a) 2y

b)

c) 4

27A

10.

a)

b) 3

221 A ,

3

2262

A

11.

a)

b) 3

44A

12.

a)

b)

eeA

1

13.

a)

b)

3A

14.

2

9A

15.

a) f és “a” i 'f és “b”.

b) 92.33

83ln6 A

16.

a) 1x , 1x

b)

3

7A

17.

1a

18.

4

2

eA

19.

a)

b)

3

32A

20.

12

1A

21.

12

eA

22.

a) Els punts de tall són 2,1,3x

b)

4

131A

23.

1a

24.

3

32A

25. 9

12 3 e

26.

a)

b) 0y

c) 3

1A

15.1 1. a) 296.0296.3 xy b) 1099.1 xy

2. a) xx

xxf

3

32)('

2

b) )18ln,3(P (la solució x = -2 està fora del domini de la funció)

3. a) Asímptota horitzontal 1y . b) x = 0

c) 577.03

1x

4. La funció a minimitzar és

22 )12(6425)( xxxf

Derivem i igualem a zero la derivada:

2222 )12(64

12

25)12(64

)12(2)1(

252

2)('

x

x

x

x

x

x

x

xxf

62.413/60

2036006003903600600169208*

360060016924)12(25

20824)12(64

*)12(25)12(64)12(64

25

)12(

)12(64

25

12)12(64

25

)12(64

25

12

)12(64

12

250

)12(64

12

25

222

23422

23422

2222

2

2

2

2

2

22

2

2

2

2

2222

x

xxxxxx

xxxxxx

xxxxx

xxxxx

x

x

x

x

x

x

x

x

x

x

x

x

x

x

x

x

x

x

x

x

x

Descartem la solució 20x perquè no té sentit en el context en el què estem.

relatiu mínimun és 60/13x0049.0)5('

0082.0)4('

creixentf

decreixentf

15.2 1.

5

20107)( 2

x

xxxxf per tant la funció canvia de signe entre els

intervals d’integració i hem de calcular l’àrea per separat:

3/16)4(

6/41)1(

3/26)2(

102

7

3107)(

6

31

3

10

6

11

3

26

3

16

6

41

3

26)2()4()1()2()()(

23

2

4

2

2

1

F

F

F

xxx

dxxxxF

FFFFdxxfdxxfA

2. babap 24022)2(0 2

0)0(

223

822

23

2)2(

23)(

223

8)0()2()(

3

46

23

23

2

2

0

F

baba

F

bxxax

dxbaxxxF

baFFdxxp

Resolem el sistema

14)5(24

5)24(223

8

3

4624

223

8

3

46

240

b

aaaabba

ba

3. Opció A.

a)

b) Per canvi de variable:

xuxu

Cx

Cuduu

dxxx

dxx

x

2'4

2

)4ln()ln(

1

2

1)2(

4

1

2

1

42

2

22

c) Determinem els punts de tall:

3411

4

1

0

4 2

2

2xx

x

x

xx

x

2

)4ln(

2

0

2

)04ln(0

2

3

2

3

2

)34ln(3

22

)4ln(

44)(

6137.10.806922

)4ln(

2

32)0(32

42

2

22

22

3

0 2

F

F

xxdxxdx

x

xdxx

x

xxF

FFdxxx

xÀrea

3 Opció B.

2)1(

)(x

x

e

exf

xxx

xx

x

xx

x

xxxx

x

xxxxx

eee

ee

e

ee

e

eeee

e

eeeeexf

)1(2)1(

)1(

1

)1(

2)1()1(

)1(

)1(2)1()('

2

344

2

010)1(

1)('

3

xe

e

eexf x

x

xx

b)

4. La gràfica de la funció és la següent

Hem de tenir en compte que la funció canvia de signe: Entre 0 i 2 és negativa i la seva àrea és

0030)0(

4128232)2(

32

6

3

363)(

404)0()2(63

23

23

2323

2

2

0

2

1

F

F

xxxx

dxxxxF

FFdxxxA

Per tant, falten 54458 u2 més a la dreta del 2x , on la funció és positiva. El nostre

problema es redueix, doncs, a resoldre l’equació:

4)2(

3)(

43)4(3)2()(6354

23

2323

2

2

2

F

kkkF

kkkkFkFdxxxAk

505035443 2323 kkkkk

Per tant, l’interval buscat és 5,0

Sèrie “Matemàtiques curriculars”

Àlgebra Lineal 2n Batxillerat:

http://www.toomates.net/biblioteca/AlgebraLineal.pdf

Geometria Lineal 2n Batxillerat:

http://www.toomates.net/biblioteca/GeometriaLineal.pdf

Càlcul infinitesimal 2n Batxillerat:

http://www.toomates.net/biblioteca/Calcul.pdf

Programació Lineal 2n Batxillerat:

http://www.toomates.net/biblioteca/ProgramacioLineal.pdf

Compendium PAU TEC: Totes les Proves de la Selectivitat de Catalunya 1998-2019

http://www.toomates.net/biblioteca/Pautec.pdf

Compendium PAU CCSS: Totes les Proves de la Selectivitat de Catalunya 1998-2019

http://www.toomates.net/biblioteca/Pauccss.pdf

Àlgebra 1r Batxillerat:

http://www.toomates.net/biblioteca/Algebra1bat.pdf

Àlgebra 4t ESO:

http://www.toomates.net/biblioteca/Algebra4eso.pdf

Links d'interès

Recull de Proves PAU Extremadura 2000-2017:

http://www.vicentegonzalezvalle.es/documentos/Examenes_selectividad_A4.pdf